Sie sind auf Seite 1von 83

CUPRINS

Din partea redaciei


____________________________________
Andrei Ungureanu O.I.M. 2005
________________________
American Mathematic Contest
________________________
Note matematice
n legtur cu o serie de identiti cu parte ntreag _________
Problem de concurs cu soluii
__________________
Metode de determinare a prilor stabile __________________
Logic Fuzzy i paradoxuri
__________________
Triunghiuri echilaterale ntr-o reea laticeal
____________
Teme pentru grupele de performan
Clasa a V-a
Baze de numeraie
____________________________________
Algoritmul lui Euclid
____________________________________
Divizibilitatea
____________________________________
Clasa a VI-a
Rapoarte i proporii, probabiliti ________________________
Clasa a VII-a
Teorema bisectoarei
____________________________________
Clasa a VIII-a
Cercul lui Euler __________________________________________
Plan mediator
__________________________________________
Plan bisector
__________________________________________
Clasa a IX-a
Ecuaia lui Pell i aplicaii
______________________________
Inegaliti remarcabile ____________________________________
Funcia caracteristic a unei mulimi
__________________
Clasa a XI-a
Continuitate uniform ____________________________________
Clasa a XII-a
Formele integrale ale inegalitilor clasice __________________
Probleme propuse
Test bacalaureat M1
____________________________________
Test bacalaureat M1
____________________________________
Test bacalaureat M2
____________________________________
Testarea Naional
____________________________________
Test admitere
____________________________________
Testare clasa a V-a
____________________________________
Clasa a V-a
______________________________
Clasa a VI-a
______________________________
Clasa a VII-a
______________________________
Clasa a VIII-a
______________________________
Clasa a IX-a
______________________________
Clasa a X-a
______________________________
Clasa a XI-a
______________________________
Clasa a XII-a
______________________________
Petre Sergescu
____________________________________
Premianii concursurilor din 2005 ________________________
Rubrica rezolvitorilor
______________________________

-0-

1
3
4
5
8
9
11
15
17
19
22
24
27
30
33
34
37
39
42
44
48
52
54
56
58
59
61
62
63
63
64
65
66
67
69
70
76
81

EDITORIAL

SSM
H

DIN PARTEA COLECTIVULUI REDACIONAL LA AL CINCILEA


NUMR AL R.M.M.
Fideli promisiunii fcute n prefaa primului numr al revistei noastre
venim din nou n faa cititorilor notri cu prezentarea micrii matematice
din judeul nostru de la ultimul numr, RMM4 i pn n octombrie 2005.
Cum se tie deja, n ianuarie 2005 a avut loc Concursul interjudeean
de matematic Petre Sergescu. O prezentare a acestei manifestri organizat
de Colegiul Naional Traian i de Filiala Mehedini a Societii de tiine
Matematice din Romnia o vei gsi n paginile revistei noastre.
Pe 5.02.2005, Consiliul Naional al SSMR a hotrt ca societatea s se
reorganizeze n virtutea O.G.26/2000 modificat i completat cu
O.G.37/2003. n urma unor demersuri de loc facile, Filiala Mehedini a
SSMR a fost nscris cu personalitate juridic la Judectoria judeului
Mehedini, ncheierea nr.13 A din 3.08.2005 i posed Certificatul de
nscriere Nr.13 din 18 august 2005. Reamintim tuturor colegilor notri
profesori de matematic sau doar iubitori de matematic faptul c
deocamdat singurele posibiliti de sporire a patrimoniului financiar sunt
cotizaiile membrilor societii (i pe anul 2005 cotizaia a fost fixat de
Consiliul SSMR la 10 RON pe an pentru fiecare membru) i Revista
Mehedinean de Matematic pe care o dorim prezent pe mesele de lucru a
ct mai muli iubitori de performan matematic fie ei elevi sau profesori.
Ne bucur mult faptul c revista noastr a ajuns s fie cunoscut n
toat ara i nu numai, referine despre ea existnd deja n Gazeta
matematic Metodic 1/2005 i Journals of Mathematics Education and
Elementary Mathematics in Romania pg.64/2004.
Pe 1.02.2005 elevii Colegiului Naional Traian au participat pentru a
patra oar consecutiv la American Matematic Competition, obinnd
rezultate meritorii ce apar n paginile revistei noastre.de asemenea vei gsi
n acest numr premianii i elevii care au obinut meniuni la Olimpiada
Judeean de matematic pe 2005, precum i premii la alte concursuri de
matematic.
Ne folosim de paginile revistei noastre pentru a semnala c pe
8.X.2005 cu prilejul unei adunri festive desfurat n sala Heliade
Rdulescu a Academiei Romne s-au omagiat 110 ani de apariie
nentrerupt a Gazetei Matematice. Cu aceast ocazie s-a lansat forma
electronic a Gazetei 1895-2004 care va apare n curnd pe pia. Costul va
fi de aproximativ 110 euro i se poate comanda prin Filiala Mehedini a
SSMR. Cu aceast ocazie s-au decernat diplome colaboratorilor activi ai
Gazetei Matematice i ne face o deosebit plcere s anunm aici c din
Filiala Mehedini au primit diplome domnii : prof.dr.Gh.Ciniceanu,
prof.drd.Prajea Manuela, prof.drd.Stretcu Daniel, prof. Nedeianu Dan,
prof.Sceanu Victor, prof.Sitaru Daniel.
S mai semnalm c trecerea anilor a fcut ca nca doi membrii activi
ai Filialei noastre i ai micrii matematice mehedinene i nu numai, este
vorba de domnii profesori Chivu Mircea i Nnui Ion, s mplineasc vrsta
de pensionare. Ne folosim de prilej pentru a le ura pensie lung, sntate i

-1-

EDITORIAL
s i rugm s rmn alturi de noi n activitatea filialei cci experiena i
energia dnilor sunt foarte necesare.
NOT din partea colectivului redacional
Pentru ca revista noastr s-i ating obiectivul cel mai important , acela de
a crete nivelul de competitivitate al elevilor mehedineni, rugm pe
colaboratorii notri s in seama pe viitor cnd vor propune materiale
pentru grupele de performan de faptul c dintre temele suplimentare
pentru pregtirea olimpiadelor deja n numerele anterioare au aparut:
1. Principiul lui Dirichlet (al cutiei) [gimnaziu] prof: N. Bogdan i prof V. Grecu (2002)
2. Inegaliti [liceu] prof: C. Ptrcoiu, L. Roianu (2002)
3. Numrul divizorilor i suma divizorilor unui numr natural [gimnaziu] prof R. Antonie
(2002)
4. Probleme de numrare [clasa a VI a] prof: G. Ciniceanu (2002)
5. Calculul unor sume [gimnaziu] prof: G. Ciniceanu (2002)
6. Monotonia, mrginirea i convergena irurilor recurente definite de funcii continue
monotone [clasa a XI a] prof: M. Prajea (2002)
7. Structuri algebrice induse (transportul de structur) [clasa a XII a] prof: C.
Ptrcoiu (2003)
8. O aplicaie practic a punctului lui Torricelli [clasa a IX a, clasa a XI a] prof: V.
Bloi (2003)
9. Ridicarea la putere a unor matrici diagonalizabile [clasa a XI a] prof: L. Grecu (2003)
10. Ultima cifr a unei puteri [clasa a V a] prof: R. Popescu, D. Stretcu (2003)
11. Complemente de geometrie (ceviene, izogonale, punctual lui Torricelli, teorema lui
Ptolemeu) [clasa a VII a] prof: G. Ciniceanu, elev: A. Caragea (2003)
12. Aplicaii ale formulelor de calcul prescurtat n probleme de teoria numerelor [gimnaziu]
prof: A. Lupu (2003)
13. Discontinuitile funciilor reale [clasa a XI a] prof: G. Ciniceanu (2003)
14. Despre izomorfisme [clasa a XII a] prof: D. Sitaru (2003)
15. Ptrate perfecte [clasa a VI a] prof: V. Sceanu (2004)
16. Teorema lui Pitagora generalizat [clasa a VII a] prof: M. Constantin, P. Ttucu
(2004)
17. Funcia parte ntreag, funcia parte fracionar [clasa a IX a] prof: P. Tupan (2004)
18. Asupra unor funcii cu proprietatea Darboux [clasa a XII a] prof: O. Ticui (2004)
19. Ecuaii exponeniale i logaritmice neelementare [clasa a X a]> prof: D. Nedeianu
(2004)
20. Numere raionale pozitive [clasa a VI a] prof: R. Popescu, D. Stretcu (2004)
21. Criterii de divizibilitate cu 7,11 i 13 [clasa a V a] prof: G. Gagea (2004)
22. Exploatarea inegalitilor [clasa a IX a] prof: G. Ciniceanu (2004)
23. Valori proprii. Vectori proprii. Teorema lui Cayley Hamilton. Teorema lui Frobenius
[clasa a XI a] prof: M. Prajea (2004)
24. Proprieti ale primitivelor unei funcii continue periodice [clasa a XII a] prof: D.
Nedeianu, L. Giugiuc (2004)
25. Reducerea unor recurene neliniare la recurene liniare [clasa a XI a] prof: V.
Preneanu (2004)
26. Teoreme despre patrulatere circumscribile [clasa a VIII a] prof: A. Ptruescu (2004)

-2-

SSM
H

EDITORIAL

Andrei Ungureanu
-unul din cei mai valoroi tineri matematicieni din lumeprof. Manuela Prajea
Medalia de Aur i Premiul I la Olimpiada Internaional de Matematic
desfurat n perioada 10-20 iulie 2005 -Marena - Mexic obinut de
olimpicul internaional Andrei Bogdan Ungureanu l situeaz pe tnrul
matematician pe primele locuri pe mapamond i confirm nc o dat
valoarea incontestabil a acestuia pe plan internaional. Performana unic
n judeul Mehedini pn la ora actual (medaliat cu aur la olimpiada
internaional), rezultatul lui Andrei Bogdan Ungureanu se contureaz dup
o carier olimpic de succes nceput cu muli ani n urm.
Premiul I la toate olimpiadele judeene i naionale (ncepnd cu cls. a
V-a i pn n prezent actualmente este elev n clasa a XI-a) ar fi suficiente
pentru a-l situa pe Andrei pe treapta cea mai de sus a olimpiadelor
naionale.
Dar rezultatele nu se opresc aici pentru c n spatele aparenelor
avem un talent nativ, o inteligen remarcabil susinut cu succes de un
caracter de nvingtor, de o performan uimitoare, de putere i druire.
i pentru c gloria nu se ctig fr sacrificii, avem multe ore de
munc, mult ambiie i hotrre n dorina de birui.
Evoluia lui Andrei pe scara internaional a nceput gradat, treapt
cu treapt, baraj cu baraj (pentru selectarea lotului olimpic naional se
susin n fiecare an alte 6 baraje), medalie dup medalie la fiecare
participare internaional.
n anul 2002, elev pe atunci n clasa a VIII-a, Andrei Bogdan
Ungureanu obine Medalia de Aur la Olimpiada Balcanic de Matematic
pentru juniori, Trgu Mure-Romnia.
Urmeaz apoi trecerea la liceu, alte nceputuri, alte standarde, pe
undeva o alt matematic: arie extins ca volum i subtilitate, probleme
adnci, acuitate maxim, ingeniozitate n creaie i construcie.
n ciuda terenului nebttorit, reuim s doborm obstacolele, s ne
ridicm deasupra greutilor i Andrei se calific n lotul pentru seniori al
Romniei ( concureaz acum cu elevii claselor IX-XII pe acelai tip de
subiecte) obinnd Medalia de Bronz la Olimpiada de Matematic
Pluridisciplinar din Iacuia-Rusia.
Trecerea la seniori i selecia n lotul Romniei n fiecare an au fost
probe foarte dure att din punct de vedere tiinific ct i psihologic:
rezisten la efort, emoii, presiuni, concentrare.
n clasa a X-a, anul 2004, a obinut Medalia de Argint la Olimpiada
Balcanic de Matematic Plevna- Bulgaria i Medalia de Argint la
Olimpiada Internaional de Matematic Atena- Grecia succes rsuntor
pentru prima participare la olimpiada internaional i pentru vrsta
fraged a lui Andrei.
n clasa a XI-a, anul 2005, situaia se complic ntruct curriculum-ul
pentru olimpiada naional este altul dect cel pentru internaionale i
Andrei reuete n paralel s le parcurg pe amndou la cote maxime.

-3-

EDITORIAL
Premiul I la Olimpiada Naional de Matematic, calificarea n lotul
Romniei urmate apoi de rezultatele internaionale: Medalia de Bronz la
Olimpiada Balcanic de Matematic Iai- Romnia i n sfrit mplinirea
visului din copilrie, succes i lacrimi de bucurie Medalia de Aur la
Olimpiada Internaional de Matematic escaladarea olimpic ajuns la
apogeu, Andrei a reuit s transforme visul n realitate.
Andrei a obinut primul loc i la concursurile americane American
Mathematics
Competitions
i
American Invitational
Mathematics
Examination (echivalentele olimpiadelor locale i judeene din Romnia),
fiind singurul calificat pentru faza final U.S.A.M.O. (United States
American Mathematics Olympiad). Punctajul foarte bun obinut l-a remarcat
n S.U.A. drept pentru care a primit o burs pentru olimpici cu durata de un
an din partea unei instituii americane de selecie a performerilor: Exeter
Academy New Hampshire SUA. Andrei nu a onorat ns aceast burs
ntruct a considerat c pregtirea n Romnia l va propulsa foarte bine n
ierarhia valorilor mondiale.
Elev eminent la toate disciplinele, Andrei a avut din clasa a V-a pn
n prezent, media general 10, a participat i la olimpiadele judeene de
fizic i informatic n clasa a VIII-a i a IX-a, obinnd numeroase premii I.
Colegiul Naional Traian a fost dintotdeauna o coal a elevilor, a
performanelor, a olimpicilor iar succesele obinute de Andrei Ungureanu ne
determin s-l declarm ca fiind cel mai bun elev al acestuia de la
nceputuri i pn n prezent i totodat cel mai valoros olimpic al tuturor
timpurilor pentru coala matematic mehedinean.
Elev fiind, am fost i eu olimpic la rndul meu (premiul I, II, III la
olimpiadele judeene, naionale i interjudeene), dar nu am beneficiat de o
atenie aparte n pregtirea mea. Cnd mi-am ales s fiu profesor, am tiut
c voi sprijini cu orice pre i sacrificii elevii cu nclinaii deosebite cci
numai aa succesele pot fi maxime. Am creat mai muli olimpici naionali
dar din prima clip cnd l-am ntlnit pe Andrei (n clasa a VIII-a) am tiut
c aceasta este o ans unic n via.
mi amintesc de prima impresie a primei ntlniri: rapiditatea,
ingeniozitatea i naturaleea cu care rezolva problemele (n demersul a dou
ore ajunsesem deja la olimpiadele naionale de clasa a X-a n mod firesc)
mi-au umplut sufletul de bucurie i efortul era cu att mai mare cu ct nu
puteam s-i art acest lucru pentru c eram abia la nceput de drum.
A urmat apoi o munc de apostolat fr ezitri i renunri, leciile
noastre durau 4-5 ore n fiecare duminic, le pregteam cu eforturi
(dificultate, concentrare, nvare, timp) pentru c este destul de greu s
lucrezi la acest nivel.
S-a depus mult suflet n acest vis, satisfaciile sunt pe msur i
avand in vedere proiectele de viitor ale lui Andrei, intrucat el se apropie de
finalizarea liceului,ii doresc mult succes in continuare.

-4-

NOTE MATEMATICE

SSM
H

n legtur cu o serie de
identiti cu parte ntreag
Prof. Gabriela & Lucian Bondoc
La nceputul o s trecem n revist cteva proprieti ale prii ntregi :
1. [x + y ] [x ] + [ y ] x, y R
[x ] x
2. = , n z * , x R
n n
1
2
n 1

3. [x ] + x + + x + + .... + x +
[nx] = 0 (Hermite)
n
n
n

Demonstraie 3.
1
2
n 1

Notm cu g(x) = [x ] + x + + x + + .... + x +


[nx ] . Este evident
n
n
n

1
1
c funcia g este periodic de perioad . Pentru x 0, avem g(x)=0.
n
n
Dac f:R R o funcie periodic de perioad principal T astfel nct f(x)=0
x [0, T ] , atunci f(x)=0 x R . Avnd n vedere observaia fcut g(x)=0
x R de unde identitatea cerut.
n continuare voi prezenta cteva identiti care folosesc proprietiile
elementare ale prii ntregi.
1. S se calculeze expresia:
n + 2k
n + 1 n + 2 n + 4
+
+
+
....
+
(O.I.M. Anglia)
2 k +1 .....k N
2 4 8

Demonstraie:
1

Pentru n=2 n identitatea lui Hermite obinem : x + = [2 x ] [x ] .


2

Expresia se mai scrie sub forma:


1
n 1 n 1 n 1
n
n n n
2 + 2 + 4 + 2 + 8 + 2 + ... + 2 k +1 + 2 + .... = [n] 2 + 2 4 ..... = [n ] = n .
2. S se arate c :
n
n
(
1) n ( 1) 1
k
( 1) k =
+

2
4
k =1
Demonstraie:

[ ]

n2

( 1) [
n2

k =1

n=2m

(Matematica n coala)

k =
1+

24
+2
2
+32 + 3142
3 + ...
3 + ... + ( 1) (n 1) + ... + (1) n .n
1
4
214
31 + 214
424
43
14444244443
n 1

3termeni

5 termeni

( 1) [
n

k = ( 1 + 2) + ( 3 + 4) + ... + [ (2m 1) + 2m ] = m

( 1) [
n2

k =1

( 2 n +1) termeni

k =1

n=2m+1

7 termeni

k = ( 1 + 2) + (3 + 4 ) + ... + (1 2m) + 2 m ( 2m + 1) = (m + 1)

-5-

NOTE MATEMATICE

[ ] [ ]

[ ]

3. Sa se arate c : n + 3 n + ... + n n = [log 2 n ] + [log 3 n ] + ... + [log n n ] (Kvant)


Demonstraie:
Observm c pentru a 1 , [a] reprezint numrul de numere naturale
mai mici sau egale cu a. Deci k n -1 este acel numr natural y 2 pentru

[ ]

([ ] ) [ ]

[ ]

care y k n adica ykn. Deci suma


n 1 + ( 3 n 1) + ... + ( n n 1) , este
numrul s al tuturor perchilor (x,y) de numere naturale, mai mari ca 1, ce
verific inegalitatea y x n . Analog se procedeaz cu partea dreapt;
[log m n] 1 este acel numr natural x 2 astfel ca x log m n , adica m x n .

De aici ([log 2 n ] 1) + ([log 3 n] 1) + ... + ([log n n] 1) = s . Observm c pentru m>n,


y 2 inegalitatea m y n nu are loc. Deci ambii membri sunt egali cu s+n-1.
4. S se arate c pentru n, k N (n,k)=1 avem identitiile:
n 2n
(k 1)n (n 1)(k 1)
a)
k + k + ... + k =
2
n 2n
(k 1)n n 1
b)
+ + ... +
=
2
k k
n
Demonstraie::
p
r
a) Plecm de la
= c + (teorema mpririi cu rest) i deducem
k
k
p r
p
k = c = k k
k 1
n (k 1) 1 k 1
in k 1 in k 1 r n k 1 1 k 1
De aici = i = i ri =
ri cu ri resturile de la
k i =1
k i =1
2
k i =1
i =1 k
i =1 k
i =1 k
n 2n
(k 1)n . Cum k i n sunt prime ntre ele, resturile
mpririle: , ,...,
k k
k
mpririlor vor fi distincte dou cte dou i fac parte din mulimea
k 1
k (k 1)
{1,2,,k-1}. Deci ri =
, de aici relaia cerut.
2
i =1
b) Rezult imediat din definiia prii fracionale.
n cele ce urmez voi prezenta cteva rezultate referitoare la identiti
de forma
n + n + 1 + ... + n + k 1 = an + b . unde a, b, k N * , k 2
a
Lema 1. Dac sirurile (an)n, (bn)n au limita + i [bn ] = [a n ] atunci lim n =1
bn
Demonstraie::
[bn ] = [an ] [bn ] an [bn ] + 1 de unde
bn 1 [bn ] a n bn + 1
a
de aici cu criteriul cletelui avem lim n =1

bn
bn
bn
bn
bn
Obs.1. Dac (bn )n i (a n)n sunt iruri pentru care [an]=[bn], convergente la 0
a
atunci lim n nu este neaprat 1 aa cum arat exemplul urmtor .
bn
1
2
, bn =
, n numr natural, i limita este 1/2.
an =
n+3
n+3

-6-

] [

NOTE MATEMATICE

] [

Obs.2. Dac n + n + 1 + ... + n + k 1 = an + b


a, b, k N * k 2 a=k2.
Demonstraie::
a
n + n + 1 + ... + n + k 1
k
lim n =lim
=
=1 a = k2
bn
an + b
a

SSM
H

[
] [ 16n + 21] = [ 16n + 22 ] = [ 16n + 23]
b) [ 9n + 7 ] = [ 9n + 8 ]
c) [ 4n + 1] = [ 4n + 2 ] = [ 4n + 3 ]
a) 16n + 20 =

Lema 2.

Demonstratie:
Artm prima identitate celelalte demonstrndu-se analog.
2
16n + 20 = m N atunci
m 2 16n + 20 < (m + 1)
Din 16n+20=(m+1)2-r, r N * rezult (m+1)2 r(mod 4) deci r 0 sau 1 (mod4)
Dar 16n+20=(m+1)2-1 16n+20=m2+2m, adic m=2t ,t N de unde avem
4n+5=t2+t egalitate imposibil deoarece cei doi termeni au pariti diferite.
Rmne m 2 16n + 20 ( m + 1) 2 4 de unde:

m 2 + 3 16n + 23 (m + 1) 2 1 m < m 2 + 3 16n + 23 < m + 1

[ 16n + 23 ] = m = [ 16n + 20 ]
de unde rezult [ 16n + 20 ] = [ 16n + 21] = [ 16n + 22 ] = [ 16n + 23 ]
deci

Proprozitie. Urmtoarele egaliti sunt verificate pentru orice n N*


a) n + n + 1 = 4n + 1 = 4 n + 2 = 4n + 3

[ n+
c) [ n +
b)

] [

n +1 +

] [
] [
]
n + 2 ]= [ 9n + 7 ] = [ 9n + 8 ]
n + 2 + n + 3 ]= [ 16n + 20 ] = [ 16n + 21] = [ 16n + 22 ] = [ 16n + 23]

n +1 +
Demonstratie:
c) Fie an= n + n + 1 + n + 2 + n + 3 bn= 16n + 20
cn= 16n + 23
Prin ridicare la ptrat se demonstreaz c 2( n + n + 3) 16n + 20
Analog se arat c

(1)

2( n + 1 + n + 2) 16n + 20

(2)

Din (1) i (2) avem an 16n + 20 pentru orice n.


Funcia f:[0, ] R, f(x)= x concav n + n + 1 + n + 2 + n + 3 < 4

4n + 6
4
(Jensen)

Dac 16n + 23 =m N * , atunci


m 2 16n + 23 < (m + 1) 2 m 2 < 16n + 23 (m + 1) 2 1.
4n + 6
= 16n + 24 m + 1
4
Am obinut [bn ] bn a n < m + 1 dar [bn ] = [cn ]=m i n consecin [an ] = m = [cn ] .

Deci an< 4

Deoarece bn < 16n + 21 < 16n + 22 < cn rezult identitatea cerut.


Bibliografie:
[1] Mircea Ganga Teme si probleme de matematica
[2] Gazeta matematica - Asupra unei probleme din gazeta matematica
Dorinel Anca

-7-

NOTE MATEMATICE
Problem de concurs cu soluii
elev. Ungureanu Andrei Bogdan
C.N.T. clasa a XII-a

S se demonstreze inegalitatea [( x + y )( y + z )( z + x)]4


unde x, y, z R*+.

16 3
( x + y + z) 3 x 3 y 3 z 3
27

Soluia 1.
y( x + y + z)
xy 3 z ( x + y + z )3
+ xz 44
3
33
Am folosit inegalitatea mediilor pentru 4 numere.
( x + y)( y + z ) = xy + xz + yz + y 2 = 3

( x + y )( x + z ) 4 4

analog obinem

x 3 yz ( x + y + z )3
33

xyz 3 ( x + y + z )3
33
nmulim inegalitile i ridicm la ptrat obinem:
( x + z )( y + z ) 44

163
x+ y+z
[( x + y )( y + z )( z + x)]
( x + y + z)3 x 2 y 2 z 2
xyz
27
3

x+ y+z
dar folosind
xyz (inegalitatea mediilor pentru 3 numere) rezult
3

ceea ce trebuia demonstrat.

Soluia 2.
Fie x = p a, y = p b, z = p c, unde p este semiperimetrul unui triunghi
iar a,b,c sunt laturile sale. Fie S aria triunghiului considerat, R raza cercului
circumscris, iar A, B, C unghiurile triunghiului.
163 3
Inegalitatea de demonstrat devine [abc]4
p ( p a )3 ( p b )3 ( p c ) 3
(1)
27
Folosind relaiile S2 = p(p a)(p b)(p c), 4RS = abc
inegalitatea (1) este echivalent cu
16S2 27R4
(2)
Este cunoscut egalitatea: 2S = R(sinA).nlocuind n relaia (2) rmne de
demonstrat c (sinA)227/4. Funcia sin este convex pe intervalul [0, ],
3
aplicnd inegalitatea lui Jensen obinem (sinA)/3 sin (A/3)=

2
(sinA)227/4, ceea ce trebuia demonstrat.

-8-

NOTE MATEMATICE
Metode de determinare a prilor stabile
pentru unele legi de compoziie

SSM
H

prof. Buzatu Carmen Victoria


Se d legea de compoziie: x * y = abx aby + axy + ab 2 + b, x, y G , a, b
Evident, x * y = a ( x b )( y b) + b, x, y G .
n 1
n
*
Prin inducie, 1
x *4
x2
* ...
4*3x = a ( x b) + b, x G, n N .
de n ori x
( n)

Notm 1
x *4
x2
* ...
4*3x = x .
de n ori x

De asemenea, x ( n) * y = [a( x b)]n ( y b) + b, x, y G , n N * .


Definiie. Numim parte stabil maximal n raport cu o lege de compoziie
cea mai mare mulime parte stabil.
Problem S se determine prile stabile G n raport cu operaia *, dac:
a) G = [c, ), c , a > 0
b) G = [c, d ], a * , G parte stabil maximal
c) G Z , G finita ; a, b Z
Soluie
a) x, y [c, ) x * y [c, ) lim ( x * y) [c, ), y [c, )
x

a >0

lim[a ( x b)( y b) + b] [c, ), y [c, ) a sgn( y b) + b [c, )


x

y b 0, y [c, ) y b x b, x, y [c, ) [c, ) [b, ).


Dac c = b x, y [b, ) x b, y b [0, ).
Cum x * y = a( x b)( y b) + b x * y b x * y [b, ) [b, ) este parte stabil.
Dac c b x, y [c, ) (b, ). Deoarece x * y [c, ) a ( x b)( y b) + b c, x,
c >b

y [c, ).Pentru x = y = c a (c b) 2 + b c (c b)(ac ab 1) 0 ac ab + 1.


bc

Deci, dac x * y [c, ), x, y [c, ) ac ab + 1. Reciproc, verificm c [ c , )


( b , ) este parte stabil atunci cnd ac ab + 1.
b < c x 0 < c b x b
0 < (c b ) 2 ( x b )( y b ) a > 0

Avem: b < c y 0, c b y b
0 < a (c b ) 2 a ( x b )( y b ) b < a (c b ) 2 + b a ( x b )( y b ) + b (1)
(1)
1 a >0
Dar ac ab + 1 : a > 0 c b + c b > 0. Deci, (c-b)(ac-a b-1 ) 0 c a (c b) 2 + b
a
a ( x b )( y b) + b, x, y [c, ) c x * y, x, y [c, )
Deci, dac b = c [c, ) = parte stabil n raport cu operaia *.
Dac b c [ c, ) = parte stabil ac ab * 1. Deci, [ b , ) este parte stabil
maximal n rapot cu operaia *.
1
b) x [c, d ] x ( n) [c, d ], n N * [a ( x b)]n + b [c, d ], x [c, d ], n N *
a
1
n
lim[ [ a ( x b)] n + b] [c, d ]. Dac x [c, d ] a.. a ( x b) > 1 a ( x b)
n a
n
(n)
x [c, d ], fals.

-9-

NOTE MATEMATICE
Deci,

x [c , d ] a ( x b ) 1 x b

x [c, d ] [c, d ] [b

1
1
1
b
x b +
,
a
a
a

1
1
, b + ].
a
a

1
1
1
1
1
1
, b + ] x, y [b , b + ], x b si y b ( x b)( y b)
a
a
a
a
a
a
1
1
1
1
1
1
2 a( x b)( y b) a( x b)( y b) b x * y b + , x, y
a
a
a
a
a
a
1
1
1
1
1
1
1
1
[b , b + ] x * y [b , b + ], x, y [b , b + ] [b , b + ] este
a
a
a
a
a
a
a
a
parte stabil maximal n raport cu operaia *.
c) G finit G marginit. Conform ( b ) a ( x b ) 1 , x G
1
1
1
1
x [b , b + ], x G G [b , b + ], a, b Z , a 0.
a
a
a
a
1
1
Dar, G Z G [b , b + ] Z G are maxim 3 elemente, i anume,
a
a
Dac [c, d] = [b

G = {b 1, b, b + 1}, dac a = 1; altfel G={b}.


Cazuri particulare:
1. S se determine prile stabile cu proprietile date pentru urmtoarea
lege de compoziie: x * y = ( x 3)( y 3) + 3, x, y [c, )
Soluie: x * y = ( x 3)( y 3) + 3 lim ( x * y ) = sgn( x 3) [c, ), x [c, )
y

x 3 0 x 3, x [c, ) [c, ) [3, ) [3, ) este parte stabil maximal n


raport cu operaia *. De asemenea, pri stabile sunt toate mulimile
[ c , ) cu ac ab + 1 . Dar a = 1, b = 3 c 4.
2. S se determine prile stabile maximale [c, d ] la operaia *:
x * y = 2( x + 3)( y + 3) 3, x, y [c, d ].
Soluie: x ( n ) = (2) n 1 ( x + 3) n 3, n N * lim x ( n ) [c, d ] 2( x + 3) 1, x [c, d ]
n

1
1
7 5
7 5
7 5
3 x 3 + , x [c, d ] x [ , ], x [c, d ] [c, d ] [ , ] [ , ]
2
2
2 2
2 2
2 2
este parte stabil maximal n raport cu operaia *.
3. S se determine prile stabile finite G Z la operaia:
x * y = 2( x 1)( y 1) + 1, x, G Z .

Soluie: x ( n ) = 2 n 1 ( x 1) n + 1, n N * , x G . Cum x G x ( n ) G , n N *
1
1 3
1 3
[ 2( x 1)] n + 1 G 2( x 1) 1, x G x [ , ], x G G = [ , ] Z
2
2 2
2 2
G = {0}.

- 10 -

NOTE MATEMATICE
Probleme propuse:
S se determine prile stabile n raport cu legile de compoziie:
a ) x * y = 4( x 7 )( y 7 ) + 7, x, y [c, d ], c, d

SSM
H

b) x * y = 3( x 2)( y 2) + 2, x, y [c, ), c
c) x * y = 5( x 4)( y 4) + 4, x, y (a, ), a
d ) x * y = 4( x 5)( y 5) + 5, x, y [c, d ] Z , c, d
e) x * y = xy 2 x 2 y + 6, x, y [c, ), c
Bibliografie:
[1] Gheorghe Andrei, Constantin Caragea, Viviana Ene
Algebr. Culegere de probleme pentru examene i olimpiade colare
[2] Colecia Gazeta Matematic
[3] Revista de Matematic, Timioara

LOGIC FUZZY i PARADOXURI

Constantin Ptrcoiu
Universitatea din Craiova - FIMST

Logica binar opereaz cu propoziii care sunt false sau adevrate nu


i adevrate i false simultan. Cu alte cuvinte n aceast logic o propoziie
este : ori alb ori neagr sau altfel spus ori bun ori rea . Se utilizeaz
deci o scar de adevr format din dou valori, de obicei aceast scar se
consider a fi {0,1}.
In realitate nu avem doar alb i negru, exist o infinitate de tonuri de
gri, nu avem doar bun sau ru, avem i aproape bun , mai puin bun ....
Logica fuzzy utilizeaz o scar de adevr cu mai multe valori, de
obicei se alege intervalul [0,1]. In aceast logic o afirmaie are un anumit
grad de adevr de obicei cuprins ntre 0 i 1; deci n aceast logic o
afirmaie poate fi de exemplu 0,6 (60%) adevrat. Interesant este c logica
fuzzy furnizeaz i soluii la probleme clasice nerezolvate cum ar fi unele
paradoxuri.
In continuare vom prezenta cteva paradoxuri clasice, noiuni legate
de mulimi i logic fuzzy, urmnd ca n funcie de interesul cititorilor s
dezvoltm dac este cazul acest subiect.
PARADOXUL AVOCATULUI
Protagoras a pregtit pe Euathlus s devin avocat, convenind ca plata s o
primeasc dup ce Euathlus ctig primul proces.
Euathlus a intrat n politic i nu a profesat avocatura.
Protagoras l d totui n judecat pe Euathlus ca s-i plteasc pregtirea.
Protagoras spune:
1. Dac ctig procesul, instana l oblig s-mi plteasc, deci voi primii
banii.
2. Dac pierd procesul, atunci el ctig primul proces, deci conform
conveniei voi primii banii.

- 11 -

NOTE MATEMATICE
Concluzie: ori cum ar fi, primesc banii.
Euathlus spune:
1. Dac ctig procesul, instana nu m obliga s pltesc , deci nu
pltesc.
2. Dac pierd procesul, nu am ctigat nici un proces, deci conform
conveniei nu pltesc.
Concluzie, oricum ar fi, nu pltesc.
Cine are dreptate ?
PARADOXUL CROCODILULUI
Un crocodil prinde un copil i i spune mamei lui :
Dac ghiceti ce voi face, i voi da copilul; dac nu, l voi mnca
-l vei mnca, strig mama disperat.
Concluzie:
Dac crocodilul mnnc copilul, mama sa a ghicit i deci crocodilul
ar trebui s-i dea copilul.
Dac crocodilul nu mnnc copilul, mama s-a nelat i crocodilul ar
trebui s mnnce copilul.
Cum se poate ieii din aceast dilem ?
Lewis Carroll a propus o soluie pragmatic:
Dac crocodilul mnnc copilul, mama a spus adevrul i crocodilul
nu se ine de cuvnt
Dac crocodilul nu mnnc copilul, mama s-a nelat i crocodilul
nu se ine de cuvnt (s-l mnnce).
Oricum ar face crocodilul nu-i poate ine cuvntul.
Neavnd nici o ans s-i pstreze onoarea inndu-i cuvntul, nu
exist nici o ndoial c va aciona n acord cu natura sa.
PARADOXUL FRIZERULUI
Pe ua unui frizer dintr-un mic ora a fost afiat urmtorul anun:
Eu brbieresc pe toi oamenii din ora care nu se brbieresc singuri i
numai pe aceia
ntrebare: Cine brbierete frizerul ?
PARADOXUL DUBLRII
Presupunnd c ntr-o noapte toate lucrurile din lume i dubleaz
dimensiunile, exist vreun mijloc de a ne da seama ?
PARADOXUL MINCINOSULUI
Epimenide, un orator din Creta a spus :
Toi locuitorii din Creta sunt mincinoi
Este adevrat sau fals aceast afirmaie ?
Mulimi i logic fuzzy
A decide cnd un obiect x are proprietatea p (cum ar fi mic , subire ,
luminos etc.) este acelai lucru cu a spune n ce msur este adevrat c
x are proprietatea p.
O soluie rezonabil ar fi utilizarea unei scri pentru diverse grade de
adevr al lui p(x). Dup cum tim, logica binar utilizeaz o scar cu dou

- 12 -

NOTE MATEMATICE
valori de adevr {0,1}. Logica fuzzy folosete de obicei o scar cu mai multe
valori, uneori cu o infinitate de valori i anume intervalul [0,1]
Fie U universul obiectelor x.
O mulime fuzzy A este caracterizat de o funcie
A:U[0,1]
care asociaz fiecrui element xU, un numr real A(x)[0,1], numit
gradul de apartenen al lui x la A.
Funcia A se mai numete funcia de apartenen a mulimii A.
Explicit, vom scrie mulimea fuzzy {x/A(x) xU}, unde perechea x/A(x)
nseamn elementul x aparine lui A cu gradul de apartenen A(x),
A(x)[0,1].
Valoarea A(x) exprim gradul de adevr ca x s aparin lui A.
Observaie: Dac n locul intervalului [0,1] drept scar a valorilor alegem
mulimea {0,1}, regsim mulimile clasice, ntruct dac A(x) = 0 putem
spune c x nu aparine mulimii A, iar dac A(x) = 1 putem spune c x
aparine lui A (A este n acest caz funcia caracteristic a mulimii A).

SSM
H

Mulimea fuzzy vid este {x/0 xU}.


Dac A,B sunt mulimi fuzzy atunci:
AB A(x) B(x);
AB=C, unde C(x)= A(x) B(x), unde nseamn maxim;
AB=D, unde D(x)= A(x) B(x), unde nseamn minim;
Complementara mulimii A este mulimea CA, unde CA(x) = 1 -A(x)
Lsm cititorilor plcerea de a constata c proprietile operaiilor cu
mulimile fuzzy sunt cele deja cunoscute:
A B = B A
A B = A B
A B = B A
A B = A B
( A B) C = A ( B C )
( A B) A = A
( A B) C = A ( B C )
A ( B C ) = ( A B) ( A C )
A ( B C ) = ( A B) ( A C )

( A B) A = A
A A = A

A A = A
n logica fuzzy pq p max q, pq p min q, non p 1- p.
Dac de exemplu n logica binar tabelul de adevr pentru disjuncia
propoziiilor
poate fi scris cu ajutorul aa-numitului
tabel Cayley
n logica fuzyy n care utilizm trei valori de adevr 0;
0.5; 1, tabelul Cayley pentru disjuncie este :
Iniial, logic fuzzy era considerat orice logic care utiliza mai mult de dou
valori de adevr. Dup faimoasa carte a lui L.A. Zadeh The concept of
linguistic variable and its applications to approximate reasoning , logica
fuzzy a inclus nc dou domenii: Teoria raionamentului aproximativ i
Teoria logicii lingvisticii.

- 13 -

NOTE MATEMATICE
In logica binar (ntruct propoziiile pot fi doar adevrate sau false)
apare urmtorul paradox :
- Cu un pai de gru nu se face un snop. Nici cu dou, nici cu trei, nici cu
patru
In consecin nu exist snopi.
Fie n un numr natural i poziia p(n): n este un numr mic
p(0) este adevrat.
p(n) p(n+1) ?
Deci, dac n este un numr mic atunci n +1 este un numr mic ?
Observm c nu putem aplica inducia aa cum o tim atunci cnd este
vorba de proprieti vagi.
Verificarea faptului c n este mic nu implic acelai efort n verificarea
faptului c n+1 este mic. De exemplu dac constatm c 100 este un numr
mic numrnd o sut de liniue, verificarea faptului c 101 este mic mai
presupune n plus un efort (ntr-adevr mic, dar efort) i anume numrarea
nc a unei liniue.
Deci, n logica fuzzy implicaia p(n) p(n+1) este adevrat numai ntr-un
anumit grad s spunem 1-, cu >0. Aadar n logica fuzzy paradoxul
precedent dispare ntruct nu este pe de-a ntregul adevrat c adunnd
succesiv paie de gru nu putem obine snopi.
Bibliografie.
1. Kaufman, A. Introduction to the theory of fuzzy sets. Academic
Press, New York,1975.
2. Kruse, R. Gebhardt, J. Klawon, F. Foundations of Fuzzy Systems
Wiley, Chichester, 1994.
3. Zadeh, L.A The concept of linguistic variable and its applications to
approximate reasoning . Information Sciences 8, 1975.
4. Zadeh, L.A Making computers think like people, IEEE Spectrum,
1984
5. Zadeh, L.A. Fuzzy algoritms. Info&Ctl.,Vol. 12, 1998
6. Zimmermann, H.J. Fuzzy set theory and its aplications, Kluwer
Boston, 1991.

- 14 -

NOTE MATEMATICE
Triunghiuri echilaterale ntr-o reea laticeal

SSM
H

Iuliana Radu, drd.Rutghers University SUA


Irina Zman, prof. Liceul ieica

S se demonstreze c un triunghi ale crui vrfuri sunt puncte de


coordonate ntregi, nu poate fi echilateral.
Solutions
1. First, prove that any triangle drawn on an "infinite geoboard" must have a
rational area. To that end, you can draw vertical lines through the left-most
and right-most vertices of the triangle, and horizontal lines through the
highest and lowest vertices. They will intersect to form a rectangle whose
sides are of integer length, so its area is an integer. The area of the triangle
inside is the area of this rectangle minus the sum of the areas of 3 little
triangles that are inside the rectangle but outside the initial triangle. Integer
minus rational gives rational.
Now, the formula for the area of an equilateral triangle is (side)2 * sqrt(3)/4.
Because the coordinates of the triangle's vertices are integers, (side)2 is an
integer as well. So, (side)2*sqrt(3)/4 cannot be rational, therefore this
triangle does not exist.
2. The answer to the question is no. Here's an informal proof.
Proof (by contradiction): Assume we want an equilateral triangle with integer
coordinates on the Cartesian Plane. Without loss of generality, let one point
be on the origin. Let the other points be (a,b) and (c,d).
Further, suppose a, b, c, and d are not all even (if they were, keep dividing
by 2 to find a similar triangle with integer coordinates which are not all
even). Note that a2 + b2 = c2 + d2 = (a-c)2 + (b-d)2, since our triangle is
assumed equilateral. For abbreviation, let k = a2 + b2.
Case 1: a and b cannot both be odd.
If a, b odd, k is congruent to 2 mod 4. This means c and d must both also be
odd (if one is odd but not the other), that gives c2+d2=odd number.
If c, d even, c2 + d2 is divisible by 4. But then (a-c)2 + (b-d)2=(even)2 + (even)2
and is divisible by 4. This is a contradiction, so a and b are not both odd.
Case 2: a and b cannot be odd and even. WLOG, assume a odd, b even.
Then either c is odd or d is even or viceversa (Similar argument to above). If
c is odd and d is even, (a-c)2 +(b-d)2 = (even)2 + (even)2 = divisible by 4. But k
is odd, since a is odd and b is even.
Likewise, if c-even and d-odd, (a-c)2+(b-d)2=odd2+odd2=even, but k is odd.
Case 3: a and b cannot both be even. If a, b even, using the same arguments
earlier in the proof, c and d would both need to be even, contradicting our
assumption that a,b,c, and d were not all even.
3. Proof by contradiction: assume you can build an equilateral triangle
having integer coordinates for all vertices: A(a1, a2), B(b1, b 2), C(c1,c 2).
Case 1: one of the sides is perpendicular on Ox. WLOG, assume the triangle
is in the first quadrant, and AC is perpendicular on Ox: A is the "higher"

- 15 -

NOTE MATEMATICE
point and B to the left of AC. AC (the length) = a2-c2, which is an integer.
Also, the height corresponding to B has integer length (a1-b1).
But in an equilateral triangle, h = side * sqrt(3)/2. If the side has integer
length, then the height cannot have integer length - contradiction.
Case 2: none of the sides is orizontal or vertical. Assume the triangle looks
like something obtained from the triangle in Case 1 when rotated clockwise
(A pointing toward North-East). Let AB intersected with Ox be D, and AC
intersected with Ox be E.
tg(ADE) = (a2-b2)/(a1-b1), tg(AEx) = (a2-c2)/(a1-c1)
(note that in Case 2 a1<>c1 and a1<>b1).
In ADE, AEX is an exterior angle so m(AEX)=m(DAE)+m(ADE). Apply tg to
both sides: tg(AEX)=[tg(60)+tg(ADE)]/[1-tg(60)*tg(ADE)]. Then
(a2-c2)/(a1-c1)=[sqrt(3)+(a2-b2)/(a1-b1)]/[1-sqrt(3) * (a2-b2)/(a1-b1)]
Perform cross multiplication here and move everything containing sqrt(3)
to one side - you'll get a contradiction.
4. Assume that O(0,0), A(a,b), B(c,d) are the vertices of the equilateral
triangle (all coordinates being integers). Let r be the square of the length of
the triangle's side. Then a2+b2=r, c 2+d2=r, r=(b-d)2+(c-a)2
r =b2+d2+c 2+a2-2bd-2ac r=r+r-2(bd+ac) bd+ac=r/2
Then (a2+b2)(c2+d2)-(bd+ac)2=r2-r2/4=(3r2)/4
When simplyfing the left side, it turns out to be (ad-bc)2
Then (ad-bc)2=3r2/4 2(ad-bc)/r=sqrt(3), contradiction.
5. Assume there is such a triangle, and let A(a),B(b),C(c) be its vertices.
In this notation, a=a1+a2*i, etc. (and a1, a2, b1, b2, c 1, c2 are integers)
(a-b)/(c-b) =cos60+i*sin60=1/2+i* sqrt(3)/2.
(a1-b1)+i*(a2-b 2)/(c1-b1)+i*(c2-b2) =1/2+i*sqrt(3)/2
Bringing everything to one side and separating the imaginary term from
the real one, we get: (a1-b1)-1/2(c1-b1)+(c2-b2)*sqrt(3)/2=0
(a2-b2)-1/2(c2-b2)+(c1-b1)*sqrt(3)/2=0
All "letters" are integers, so c2-b2 must be 0.
Now, (a1-b1)-1/2(c1-b1)=0. Using similar reasoning, c1-b1 must be 0, etc
So, a1=b1=c1 and a2=b2=c2 which means A=B=C-therefore ABC cannot be a
triangle.

- 16 -

NOTE MATEMATICE

SSM
H

Tem pentru grupa de performan la clasa a- V -a


Baze de numeraie
prof. Adriana Moclea
Sistemul de numeraie n baza zece s-a impus civilizaiei moderne
datorit avantajelor lui practice. De-a lungul istoriei, au fost folosite i alte
baze de numeraie, de exemplu baza douzeci sau baza aizeci.
Cel mai economic sistem de numeraie este cel care folosete numai
doua cifre: 0 i 1. Acest sistem se numete binar, sau baza 2.
Un numr ntr-o baz oarecare B se scrie sistematic astfel:
(a n a n -1a n -2 a 2 a 1a 0 ) (B) = a n B n + an -1 B n -1 + a n -2 B n- 2 + + a 2 B 2 + a 1B1 + a 0
Efectund calculele, se obine numrul transformat n baza 10.
Trecerea unui numr din baza 10 intr-o baz oarecare B se face prin
algoritmul mpririlor succesive.
Pentru o mai buna nelegere, iat cteva probleme pregtitoare pentru
olimpiade i concursuri de matematic, specifice acestei teme:
1. S se verifice egalitile:
33115 - 11335 = 44225 - 22445
44227 - 22447 = 66447 - 44667
apoi s se arate n ce alte sisteme de numeraie rmn adevrate. Indicii
indic bazele de numeraie .
prof. Adriana Moclea
Soluie: scrise n baza de numeraie x egalitile devin identiti
2x3 + 2x2 - 2x - 2 = 2x3 + 2x2 - 2x - 2
Aceast identitate are loc n orice sistem de numeraie cu baza x 7.
2. S se gseasc cifrele corespunztoare literelor pentru a avea egalitatea:
1a5 6 = b035
prof. Adriana Moclea
Soluie:
62 + 6a + 5 = 52 b + 3 6a + 38 = 25b
(i) dac 25b se termin cu 0, atunci 6a se termin cu 2
a) deci a =2, b =2 ;
b) dac a =7, atunci nu exist b.
(ii) dac 25b se termin cu 5, atunci 6a se termin cu 7, dar nu exist cifra
a cu aceast proprietate.
3. S se gseasc elementele mulimii A ={a, b}, tiind c are loc egalitatea:
abba bbba = aaa numerele fiind ntr-o baz de numeraie mai mic dect 10.
prof. Adriana Moclea
Soluie: scriem egalitatea n baza x i apoi trecnd n baza 10 se obine
relaia
(a - b)(x -1)(x2 + x + 1) = a (x2 + x + 1)
sau
(a - b)(x -1) = a
(i)
x = 9 7a = 8b A ={8, 7}
(v)
x = 5 3a = 4b A ={4, 3}
(ii)
x = 8 6a = 7b A ={7, 6}
(vi)
x = 4 2a = 3b A ={3, 2}
(iii)
x = 7 5a = 6b A ={6, 5}
(vii) x = 3 a = 2b A ={2, 1}
(viii) x = 2, imposibil.
(iv)
x = 6 4a = 5b A ={5, 4}

- 17 -

CERCUL DE MATEMATICA
4. S se arate c numrul abcd 35 , unde a, b, c sunt cifre impare i d cifra
par, nu este ptrat perfect .
prof. Adriana Moclea
4
3
2
Soluie: (5 a + 5 b +5 c) + (5d + 3), prima sum are ultima cifr 5, a doua
sum are ultima cifr 3, deci numrul se termin cu cifra 8, deci nu poate fi
ptrat perfect.
5. Se d fracia F = (35x - 3x)/31x , unde x este baza de numeraie.
S se arate c F este ireductibil.
prof. Adriana Moclea
Soluie: n baza 10, F devine (3x + 2)/(3x + 1), dac fracia se simplific prin
d, atunci d|(3x + 2), d|(3x + 1), deci d divide diferena, deci d|1, rezult c
F este ireductibil.
6. Dac 301b = 28, c este ultima cifr a numrului 20052006 , iar a = (3n + 2;
2n + 1), n

N, calculai a b + b c + c a

prof. Adriana Moclea


Soluie: 3b2 + 1 = 28, deci b =3; 20052006 are ultima cifr 5, deci c = 5;
a|2(3n + 2) si a|3(2n + 1), deci a|2(3n + 2) - 3(2n + 1) = 1, deci a =1
nlocuind a, b, c se obine 249.
7. Aflai numerele naturale x, y, z care verific egalitile :
a) 23x + 2 + 23y + 22z + 1 = 352
b) 24x + 1 + 23y + 2 + 2z + 1 = 552
prof. Adriana Moclea
Soluie:
a) x =y =z =2
b) x =z =2, y =1
8. Se dau numerele A = 10 2 + 120 3 ; B = 506 1; C = 12 4 + 108
a) Fr a extrage rdcina ptrat s se ordoneze numerele.
b) S se arate c : [ AC ( B 2 18) 137 112 ] : (187 128 + 112 ) = 1 .
Soluie: a) B<A<C
9. S se gseasc toate numerele din sistemul zecimal care se scriu n
sistemul binar cu dou cifre, cu trei cifre, cu patru cifre.
GM.nr7/1978
Soluie:
ab 2 = 2a + b , unde a = {1}; b= {0,1}. Atunci numerele vor fi 2 i 3.
abc2 = a 22 + b 2 + c = 4a + 2b + c , unde a={1}; b={0,1}; c={0,1}.
Atunci numerele vor fi : 4, 5, 6, 7.
abcd 2 = 8a + 4b + 2c + d , unde a={1} ; b={0,1} ; c={0,1} ; d={0,1}.
Atunci numerele vor fi : 8, 9, 10, 11, 12, 18, 14, 15.
10. Se consider numerele N 1 = ( aaa
aaa
12...
3a ) b si N 2 = ( 1
2...
3a ) b , n N*.
3n

a) S se determine ctul mpririi N1 : N2.


b) Utiliznd rezultatul obinut, s se afle ctul mpririi 5555555552 : 5552.
G.M. nr. 6/1982
Soluie:
a) N2 =a( bn-1 + bn-2 ++1)
N1=a( b3n-1 + b3n-2 ++ b2n + b 2n-1 + b 2n-2 + + bn + bn-1 + bn-2 ++1)
N1=ab2n(bn-1 + bn-2 ++1) +abn(b n-1 + b n-2 + +1) +a(bn-1 + bn-2 ++1)
N1= a( bn-1 + b n-2 ++1)( b2n + b n ++1) ,
Atunci N1 : N2 =b2n +b n ++ =100 00100 001b , 2n+1 cifre n baza b.
b) 5555555557 : 5557 = 10010017

- 18 -

CERCUL DE MATEMATICA
Tem pentru grupa de performan la clasa a- V -a

SSM
H

Algoritmul lui Euclid


Prof. Pi-Rada Ionel-Vasile CNT
Prof.Pi-Rada Marica c.Gen.5
Enunul problemei: Se dau dou numere naturale a, b. S se
determine cel mai mare divizor comun i cel mai mic multiplu comun
pentru cele dou numere.
Prezentm algoritmul lui Euclid pentru calcularea cmmdc a dou numere.
Se depun n cutiile x (demprit) i y (mpritor) valorile din cutiile a
i respectiv b. Utilizm i cutia r (rest). Euclid ne nva urmtorul algoritm:
- ct timp y>0 (mpritorul nu este zero i deci se poate efectua
mprirea) efectum urmtoarele operaii
i. r primete restul mpririi dintre x i y
ii. x primete valoarea din y
iii. y primete valoarea din r
- cmmdc este valoarea din cutia x (dempritul)
- cmmmc se poate apoi calcula cu formula cmmmc=a(b/cmmdc)
n continuare voi prezenta doua implementri n limbajele de programare
Pascal si C++:
Programul in limbaj Pascal
Programul in limbaj C++
Program cmmdc_si_cmmmc;
#include<iostream.h>
Var a,b,x,y,r,cmmdc,cmmmc:
unsigned long a, b, x, y, r, cmmdc,
longint;
cmmmc;
Begin
int main(void){
repeat
do {
Write(a=); readln(a);
cout<<a=; cin>>a;
Write(b=); readln(b);
cout<<b=; cin>>b;
Until (a>0)and(b>0);
} while(!((a>0)&&(b>0)));
x:=a; y:=b;
x=a; y=b;
while (y>0) do
while (y>0)
begin
{
r:=x mod y; x:=y; y:=r;
r=x%y; x=y; y=r;
end;
}
cmmdc:=x; cmmmc:=a*(b div
cmmdc=x; cmmmc=a*(b/cmmdc);
cmmdc);
cout<<cmmdc=<<cmmdc<<endl;
writeln(cmmdc=,cmmdc);
cout<<cmmmc=<<cmmmc<<endl;
writeln(cmmmc=,cmmmc);
return 0; }
end.
Observaia 1. Algoritmul lui Euclid este unul dintre cei mai vechi algoritmi
cunoscui. Interesant este faptul c dei este destul de performant este i
simplu n acelai timp. Corectitudinea i finitudinea (se termin dup un
numr finit de pai) se bazeaz pe observaiile:
1. cmmdc(x,y) = cmmdc(y,r) unde r este restul mpririi lui x la y;
- conform teoremei mpririi ntregi avem x=yc+r, unde c este ctul;
- avem deci r=x-yc;
- dac d este divizor pentru x i y, atunci el divide i pe r, deci d este
divizor comun att pentru perechea (x,y) ct i pentru perechea (y,r);

- 19 -

CERCUL DE MATEMATICA
dac presupunem c d este cmmdc pentru (x,y) dar nu i pentru (y,r),
atunci rezult c exist h>d i h este cmmdc pentru (y,r);
- ABSURD cci atunci ar rezulta c h divide y i h divide r, i deci h
divide yc+r=x adic h este divizor comun pentru (x,y) i fiind mai mare
dect d ar rezulta c d nu este cmmdc pentru (x,y)
2. un ir strict descresctor de numere naturale nu poate avea infinit de
muli termeni pentru c se termin odat cu apariia numrului zero.
3. restul mpririi ntregi a dou numere naturale este strict mai mic dect
mpritorul
Observaia 2. Practic, pe parcursul algoritmului, se parcurge un ir de
perechi de numere naturale. Exemplu: (288,376), (376,288), (288,88),
(88,24), (24,16), (16,8), (8,0) cmmdc=8.
Observaia 3. Pe parcursul algoritmului lui Euclid pentru calcularea
cmmdc nu intereseaz cturile mpririlor ci doar resturile acestora.
Observaia 4. Algoritmul lui Euclid poate fi folosit i la determinarea unei
soluii (x,y) cu numere ntregi pentru ecuaia ax+by= cmmdc(a,b). Aceast
variant de algoritm este cunoscut sub numele de
Algoritmul extins al lui Euclid:
Presupunem c a=bc+r. La nceput putem scrie:
a=1a+0b
i b=0a+1b
Putem observa apoi c pentru fiecare mprire, dac avem dempritul x
scris sub forma x=u1a+v1b i respectiv mpritorul y scris sub forma
y=u2a+v2b, atunci restul r=x-yc se va putea scrie sub forma r=u3a+v3b,
unde u3=u1-cu2 i v3=v 1-cv2. Deci toate numerele care apar la demprit se
pot scrie sub forma ua+vb , adic i cmmdc.
Exemplu pentru a=288 si b=376 :
t = u a + v b
a = 1 a +
0 b
b = 0 a +
1 b
288 =
1 288 +
0 376
376 =
0 288 +
1 376
288 =
1 288 +
0 376
88 = -1 288 +
1 376
24 =
4 288 + -3 376
16 = -13 288 + 10 376
8 = 17 288 + -13 376
Pas 1 (u1 ,v1 )=(1,0),
Pas 2 (u2 ,v2 )=(0,1),
Pas 3 (u3 ,v3 )=( u1-c 1 u2, v 1- c1 v2) ,
Pas 4 (u4 ,v4 )=( u2-c 2 u3, v 2- c2 v3) ,
Pas 5 (u5 ,v5 )=( u3-c 3 u4, v 3- c3 v4) ,
.
Pas k (uk,vk)=(,)
Pas k+1: (uk+1,vk+1)=(,)
.
Pas n+2: (un+2, vn+2)=(un-c nun+1,vn-cnvn+1)
Algoritmul se oprete cnd un+2a+vn+2b=0 si atunci un+1a+
vn+1b=cmmdc sunt numerele cutate. Algoritmul extins se poate scrie astfel
Programul in limbaj Pascal
Program cmmdc_si_cmmmc;
-

- 20 -

CERCUL DE MATEMATICA
Var a,b,x,y,r,cmmdc,cmmmc:
longint;
Var c,u1,u2,u3,v1,v2,v3:longint;
Begin
repeat
Write(a=); readln(a);
Write(b=); readln(b);
Until (a>0)and(b>0);
x:=a; y:=b;
u1:=1; v1:=0;
u2:=0; v2:=1;
while (y>0) do
begin
r:=x mod y;
c:=x div y;
u3:=u1-c*u2;
v3:=v1-c*v2;
u1:=u2; u2:=u3;
v1:=v2; v2:=v3;
x:=y; y:=r;
end;
cmmdc:=x;
cmmmc:=a*(b div cmmdc);
writeln(cmmdc=,cmmdc);
writeln(cmmmc=,cmmmc);
writeln((u,v)=(,u1,,,v1,));
end.

#include<iostream.h>
long int a,b,x,y,r,cmmdc,cmmmc;
long int c,u1,u2,u3,v1,v2,v3;
int main(void) {
do {
cout<<a=; cin>>a;
cout<<b=; cin>>b;
} while( !((a>0)&&(b>0)) );
x=a; y=b;
u1=1; u2=0;
v1=0; v2=1;
while (y>0)
{
r=x%y;
c=x/y;
u3=u1-c8u2;
v3=v1-c*v2;
u1=u2; u2=u3;
v1=v2; v2=v3;
x=y; y=r;
}
cmmdc=x;
cmmmc=a*(b/cmmdc);
cout<<cmmdc=<<cmmdc<<endl;
cout<<cmmmc=<<cmmmc<<endl;
cout<<(u,v)=(<<u1<<,<<v1<<)<<
endl;
return 0;

SSM
H

Programul in limbaj C++


Probleme propuse:
Exerciiul 1. Arborele cu fracii: Se pornete de la fracia 1/1 care se va
numi rdcina. Din fiecare fracie i/j se obin dou fracii i/(j+i) i (i+j)/j.
Gsii pentru fracia 510511/30031 numrul de fracii care se afl pe lanul
de fracii ce duce la rdcin (se vor numra i capetele lanului).
Exemplu: Pentru fracia 67/23 traseul se obine din aproape n aproape
astfel: 44/23, 21/23, 21/2, 19/2, 17/2, 15/2, 13/2, 11/2, 9/2, 7/2, 5/2,
3/2, 1/2, 1/1 deci numrul de fracii de pe traseu este 15.
Exerciiul 2. Determinai o pereche de numere ntregi (x,y) soluie pentru
ecuaia 70x+51y=7.
Exerciiul 3. Determinai cea mai mic valoare pozitiv pe care o poate avea
expresia 203x+311y, unde x si y sunt numere ntregi.
Exerciiul 4. Dovedii c orice fracie din arborele de la exerciiul 1 este
ireductibil.
Exerciiul 5. Verificai c numrul de fracii de pe traseul dintre o fracie
din arbore i rdcina arborelui este egal cu suma cturilor care se obin
prin algoritmul lui Euclid de calculare a cmmdc aplicat valorilor de la
numrtorul i numitorul fraciei .

- 21 -

CERCUL DE MATEMATICA
Exerciiul 6. Justificai c algoritmul urmtor este corect i se poate folosi
pentru calcularea cmmdc a dou numere naturale. Implementai-l ntr-un
limbaj de programare!
Pas 1 : citete a,b
Pas 2 : x a
Pas 3 : y b
Pas 4 : ct timp (x diferit de y) execut
daca x>y
atunci x x-y
altfel y y-x
sfarsit daca
sfarsit cat timp
Pas 5 : scrie x
Pas 6 : stop

Tem pentru grupa de performan la clasa a- V -a


Divizibilitatea
- instrument principal in studiul numerelor naturale -

prof. Gimoiu Iuliana

Definitie Fie a si b doua numere naturale, a0. Spunem ca a divide


pe b si scriem a|b, daca exista un numar natural c, astfel incat ac=b.
Daca a divide pe b, spunem ca este un divizor al lui b sau b este un
multiplu al lui a.
Studiul divizorilor este principalul mod de clasificare a numerelor naturale.
O prima clasificare a numerelor naturale este: numar par, numar impar.
Divizibilitatea prin 2, cea mai simpla, furnizeaza primul criteriu de
triere a numerelor naturale.
Exista numere care sunt divizibile prin 2, numerele pare, si cele care
nu sunt divizibile prin 2, numerele impare.
Alternanta par-impar jaloneaza succesiunea numerelor naturale.
Eleganta regula a paritatii, permite un acces rapid si surprinzator la
solutionarea unor probleme interesante din nebanuite domenii.
Iata o problema:
Numerele narurale de la 1 la 18 sunt scrise in linie. Putem
pune intre numere semnele + sau astfel incat suma lor sa fie 0?
Dupa oricate incercari facute, vom constata ca nu putem obtine suma 0.
Dar la acelasi rezultat putem ajunge observand ca suma numerelor cu
semnul + trebuie sa fie egala cu suma numerelor cu semnul , deci ar
trebui ca suma 1+2+3++17+18 sa poata fi partajata in doua sume egale,
lucru imposibil deoarece 1+2+3++17+18=171 adica numar impar.
Determinarea numarului de divizori ai unui numar natural a permis
matematicienilor o noua clasificare a numerelor naturale:
numarul 1;
numere care au exact doi divizori (numere prime);
numere care au si alti divizori (numere compuse).

- 22 -

CERCUL DE MATEMATICA
Teorema Fiecare numar compus se descompune in produs de puteri
de numere prime intrun singur mod (exceptand ordinea factorilor).
Asadar cunoasterea numerelor prime devine o probleme capitala pentru
descompunerea in factori primi a unui numar natural dat.
Sirul numerelor prime este: 2,3,,7,11,13,
Pentru a verifica daca un numar natural este prim sau nu, este
cunoscut faptul ca se imparte numarul dat la numerele prime din sirul de
mai sus, pana cand catul devine mai mic decat impartitorul. Daca nici una
din impartiri nu sa facut exact, numarul dat este prim, in caz contrar el
este numar compus.
Pentru reducerea calculelor se vor folosi criteriile de divizibilitate cu
2,3,5 prezentate in toate manualele de clasa a Va si cunoscute si
recunoscute cu usurinta de catre elevi.
Prezentam in continuare un criteriu de divizibilitate cu 7:
Un numar natural se divide cu 7 daca si numai daca diferenta
dintre numarul scris fara ultima cifra si dublul ultimei cifre este
divizibil cu 7.
Demonstratie Fie N un numar natural care are ultima cifra a; deci
N= xa unde x este un numar natural oarecare (nu neaparat cifra).

SSM
H

Demonstram ca N7 (x2a) 7.
Calculand 2N+(x2a)=2 xa +x2a=
=2(10x+a)+x2a=
=20x+2a+x2a=
=21x, obtinem un nunar divizibil cu 7.
Daca N7 si 2N+(x2a)7 atunci (x2a) 7 iar daca (x2a)7 si
2N+(x2a) 7, atunci N7.
Am folosit in demonstratie urmatoarea proprietate foarte cunoscuta a
relatiei de divizibilitate:
Fie a,m,n numere naturale, a0.
Daca a|m si a|n, atunci a|(mn).
Exercitii propuse:
1. Care este primul numar divizibil cu 7 din sirul: 9,99,999,9999,?
2. Un elev pretinde ca orice numar de forma aba se divide cu 7, daca a+b=7.
Este adevarat?
3. Demonstrati ca suma 5+5 3+55++547 se divide prin 126.
4. Demonstrati ca 7 divide numarul 3638+4133.
Bibliografie:
[1] Ioan Dancila, Divizibilitatea numerelor,Editura Sigma, Bucuresti
2001.
[2] Gh. Eckstein & colectiv, Olimpiade si concursuri de matematica
V-VIII, Editura Birchi, 2005

- 23 -

CERCUL DE MATEMATICA
Tem pentru grupa de performan la clasa a- VI -a
Rapoarte i proporii, probabiliti

prof. Adrian Lupu


Colegiul Tehnic Decebal

n cele ce urmeaz ne propunem s prezentm cteva chestiuni legate


de tematica anunat care sperm s fie utile n pregtirea pentru
performan a elevilor de clasa a VI-a. Demersul nostru vizeaz elemente de
teorie care sunt mai puin sau chiar deloc prezentate la clas precum i
exerciii n legtur cu acestea, prin care sperm s cultivm i mai mult
apetitul pentru matematic al micilor matematicieni.
O aplicatie imediata a notiunii de raport este data de reprezentarea la
scara (prin scara unui plan elevii intelegand raportul dintre distanta din
planul reprezentarii si distanta in realitate , avand drept notiune asociata pe
cea de scar a unei harti).Uzual, acest raport are numratorul egal cu
unitatea.Este esenial de precizat ca lungimile ce alcatuiesc raportul trebuie
sa fie exprimate prin aceeasi unitate de masura.
Exemplu:
Sa se determine lungimea gardului ce delimiteaza
terenul de sport reprezentat in figura urmatoare,
1
realizata la scara
, unde ABCD este dreptunghi
100
cu dimensiunile AD=20 cm , AB=40 cm,
Soluie: n realitate pentru terenul de sport limea
are 20 cm x100=2000cm = 20 m si lungimea 40cm x100 = 4000cm = 40m
deci perimetrul de 120 m .
La nivelul clasei a VI-a notiunea de probabilitate este prezentata ca
raportul dintre numarul cazurilor favorabile realizarii unui eveniment si
numarul cazurilor egal posibile ale experientei , punctandu-se cazurile
limita : probabilitatea evenimentului imposibil este 0 , iar cea a
evenimentului sigur este 1.
1. Un numar avind trei cifre se obtine aranjand in ordine aleatoare cifrele 2,
4, 6. Care este probabilitatea ca numarul :
a) sa fie divizibil cu 6
b) sa fie divizibil cu 4
c) sa fie prim.
Solutie: Cazurile egal-posibile sunt 246, 264, 462, 426, 624, 642.
Dintre acestea , cazurile favorabile vor fi :
a) toate , caci oricare dintre cele 6 numere se divide cu 2 si 3 deci P=1
2 1
b) 264, 624 deci P= =
6 3
c) Nici unul , caci toate cele 6 se divid cu 2.
2. Dintr-o echipa de elevi cu 6 baieti si 4 fete, alegem la intamplare 2
copii.Care este probabilitatea ca ambii copii sa fie baieti?
9
Solutie: Numarul cazurilor egal-posibile este 10x =45
2
5
1
Numarul cazurilor favorabile este 6x = 15, P=
2
3

- 24 -

CERCUL DE MATEMATICA
3. Care este probabilitatea ca aruncand 2 zaruri , suma punctelor sa fie
solutie a ecuatiei in numere naturale : n-3n+2=0?
Solutie: Cazuri egal-posibile sunt 36. Ecuatia are solutiile n=1 si n=2
Dar 1 nu se poate scrie ca suma de doua punctaje la aruncare iar 2=1+1
1
este singurul caz favorabil.P=
.
36
Un alt tip de probleme cu caracter special este al asa-numitelor
m
probleme de aliaj. Titlul unui aliaj este dat de formula:T=
unde m este
M
masa metalului pretios din componenta aliajului iar M este masa intregului
aliaj. In aceeasi problematica se incadreaza si exercitiile in legatura cu
m
concentratia unei solutii: c= d unde md este masa substantei dizolvate iar
ms
ms este masa solutiei. Daca este exprimata procentual, se foloseste formula
m
C= d x 100
ms
Exista si probleme corelate cu aceleasi idei dar in care nu sunt folosite
neaparat pentru exemplificare metale in aliaje ,dar suportul teoretic
matematic este acelasi.Sunt asa numite probleme de amestec
Exemple
1. Sa se afle titlul unui aliaj aur-cupru care contine 200g aur si 800g cupru
200
Solutie m=200g , M=1000g => T=
=0,2 .
1000
Observatie Ca si la scara unei reprezentari, e important ca unitatile de
masura sa fie aceleasi.
2. Sa se afle concentratia unei solutii de acid-sulfuric dupa ce la cele 300g
apa in care au fost dizolvate 200g substanta se mai adauga 500g apa
200
200
Solutie C=
=
=0,2
200 + 300 + 500 1000
Concentratia procentuala va fi de 20%
3. Sa se afle pretul unui Kg de cafea din amestecul a 3 Kg de cafea cu
100.000 ROL/Kg cu 5 Kg la 120.000 ROL/Kg
3 100.000 + 5 120.000 900.000
Solutie P=
=> P=112.500 ROL
=
3+ 5
8
Observatie Elevii pot intui ca la o astfel de problema in esenta este vorba
despre media ponderata, in care ponderile sunt date de cantitatile partiale .
In ceea ce priveste problemele legate de procente, proportii , proportii
derivate, siruri de rapoarte egale, proportionalitate directa si inversa ,
suportul teoretic necesar pentru abordarea problemelor de concurs este
suficient de bine reprezentat in curriculum-ul de clasa.
Pentru final prezentam cateva probleme in legatura cu subiectul in
discutie.
1. Daca pe o harta a Romaniei realizata la scara 1:800000 distanta dintre
Craiova si Drobeta-Turnu Severin este de 12,5 cm , sa se afle distanta
reala dinte cele doua orase.
2. Numarul ab se construieste astfel incat cifrele a si b sa fie numere prime.
Sa se afle probabilitatea ca numarul ab sa fie prim .

SSM
H

- 25 -

CERCUL DE MATEMATICA
(Adrian Lupu,Minimath)
3. Fie n un numar natural mai mic sau egal cu 2005. Sa se afle
probabilitatea ca numarul (n+1851)(n+1907)(n+1642) sa se divida cu 3.
(Adrian Lupu,Minimath)
4. Sa se arate ca daca amestecam p1 Kg de cafea la pretul x1 pentru un Kg
cu p2 Kg de cafea la pretul x2 pentru un Kg se obtine un amestec in care
p x + p2 x2
pretul unui Kg este P= 1 1
. Generalizarea la n cantitati partiale
p1 + p 2
p1,..pn, n ,n 2 cu preturile x1,..,xn.
5. Intr-un pahar se dizolva 10g sare in 90g apa.Intr-un alt pahar se dizolva
tot sare in 210g apa.Se amesteca apoi continuturile celor doua pahare.Sa
se afle cata sare s-a dizolvat in cel de-al doilea pahar stiind ca in final
concentratia solutiei obtinute este de 0,1.
a+b b+c c+a
6. Fie a,b,c N*.Aratati ca daca
=
=
atunci a=b=c. (***)
b+c c+a a+b
ab bc ca
7. Aratati ca daca a,b,c N* si
=
=
atunci a=b=c.
(***)
c
a
b
8. Fie a,b,c numere rationale strict pozitive. Aratati ca daca
2a b 2b c 2c a
=
=
atunci oricare dintre numere este media aritmetica
c
a
b
a celorlalte doua.
(***)
a1 + a 2
a 2 + a3
a 2005 + a1
9. Aratati ca daca
unde
=
= ............... =
a 2 + 2005 a 2 a 2 + 2005 a 3
a 2005 + 2005 a1
a1,,a2005 Q *+ atunci a1= a2=.=a2005
(***)
a d
10. Fie a,b,c N* astfel incat
= .Sa se demonstreze ca a+b+c+d este
c b
numar compus
(***)
11. a )Determinati numerele naturale a,b,c stiind ca sunt direct
proportionale cu 0,25 ; 0,(3) si 0,5 si ab+bc+ca=54
b)Cat la suta reprezinta numarul mai mic din suma celorlalte doua ?
(Etapa judeteana,Gorj,1985)
12. Suma a trei numere este 87. Daca se mareste primul numar cu 150%
din el,al doilea se micsoreaza cu 25% din el, iar al treilea se micsoreaza
cu 5,atunci numerele sunt egale.Sa se afle numerele.
(Etapa judeteana,Hunedoara,1994)
13. Sa se afle numarul rational pozitiv care dupa ce creste cu 16%, iar
rezultatul scade cu 16%, se micsoreaza cu 16.
(Etapa judeteana,Dolj,1991)
14. Numarul a reprezinta 25% din numarul b.Cat la suta reprezinta
numarul b din a+b?
(Etapa judeteana,Covasna,1992)
Bibliografie:
1) V.Pop a - Teme pentru grupele de excelenta la matematica clasa a
VI-a editura Dacia Cluj 2004
2) C.Hrabor Probleme din concursurile de matematica clasele V-VIII
editura, Scorpion, Bucuresti 1995
3) A.Blauca Probleme pentru clasa aVI-a editura, Axa,Botosani 1998

- 26 -

CERCUL DE MATEMATICA
Tema pentru grupa de performanta la clasa a- VII -a

SSM
H

Teorema bisectoarei

prof. Mariana Draga Ttucu

Teorema bisectoarei unghiului interior

Fie ABC si D (BC ) . [AD este bisectoarea unghiului < BAC daca si numai
BD AB
daca :
=
.
DC AC
Demonstraie:
Fie [AD bisectoarea unghiului BAC. Ducem prin C paralela la AD care
intersecteaz pe AB in E.
BD AB
ADCE
=
(1)
DC AE
Dar < AEC < BAD unghiuri corespondente
< DAC < ACE alterne interne
AEC-isoscel AE=AC(2)
BD AB
Din(1) si(2)
=
.
DC AC
BD AB
Fie
=
(1) si D (BC) astfel incat
DC AC
[AD bisectoarea
BD' AB
unghiului < BAC =>
=
( 2)
D' C AC
BD BD'
BD
BD'
Dar din (1) si (2)
=
=>
=
=>
DC D' C
BD + DC BD'+ D' C
BD BD'
=
=> BD = BD' ; D, D' ( BC ) => D = D'.
BC BC
Observaie: Teorema de mai sus se poate formula si astfel: Bisectoarea unui
unghi al unui triunghi determina pe latura opusa unghiului segmente
proporionale cu laturile si reciproc.

Teorema bisectoarei unghiului exterior

Fie ABC, D BC , D ( BC ) . Atunci [AD este bisectoarea unuia din unghiurile


BD AB
cu vrful in A si exterioare triunghiului, daca si numai daca:
=
.
DC AC
Demonstraie:
Fie C (BD) si [AD
bisectoarea < B' AD .
Ducem
CEAD
< ECA < CAD coresp.

< AEC < B ' AD alt. int .


AEC-isoscel.

- 27 -

CERCUL DE MATEMATICA
ECAD

BD AB
BD
AB
D BC , D ( BC ) astfel nct
=
; AE = AC =>
=
DC AE
DC AC.

BD
AB
=
DC AC.
Fie D BC , D' ( BC ) astfel
incat [AD bisectoarea
BD' AB
=
D' C AC

< B' AC
BD AB
=
DC AC
BD' BD
BD' D' C BD DC
BC
BC
=
=>
=
=>
=
=> D' C = DC => D = D'.
D' C DC
D' C
DC
D' C DC
Teorema: Fie [AD si [AD bisectoarele interioara respectiv exterioara ale
unghiului < BAC al triunghiului neisoscel ABC.Locul geometric al punctelor
MB AB
M din planul triunghiului ABC pentru care
=
este un cerc care
MC AC
trece prin punctele A,D si D(Acest cerc se numete cercul Apoloniu al
laturii [BC]).
Demonstraie:
Din teorema bisectoarei interioare rezulta AB/AC = BD/DC, de unde rezulta
/\

ca MB/MC = BD/DC, deci [MD este bisectoarea interioara a unghiului BMC .


Analog, folosind teorema
bisectoarei exterioare si relaia
din enun, se obine ca [MD
este bisectoarea exterioara
/\

pentru unghiul BMC .Rezulta


/\

ca m( DMD') = 90 0 si cum D si D
sunt fixe, M aparine cercului
de diametru [DD].
Reciproc, se arata ca orice punct M al cercului de diametru [DD] satisface
relaia MB/MC=AB/AC .
Se observa mai intai ca din teoremele bisectoarelor interioara si exterioara
se obine egalitatea DB/DC = DB/DC.
Se construiesc paralelele BE, BE, la MD, respectiv la MD.Din teorema lui
Thales in triunghiul EBC
pentru paralela MD si
triunghiul DMC pentru
paralela EB, se obin
relaiile ME/MC = DB/DC
(1) , si ME/MC = DB/DC
(2) ME = ME si cum
triunghiul EBE este
dreptunghic in B, se obine
ME = ME = MB.

- 28 -

CERCUL DE MATEMATICA
nlocuind in relaia (1) pe ME cu MB si in relaia (2) pe ME cu MB, se obine
ca:MB/MC = DB/DC = DB/DC = AB/AC. Analog se obin cercurile lui
Apoloniu ale laturilor [AB] si[AC].
Aplicaie:
Fie ABC, [AD bisectoarea interioara a < BAC , D ( BC ). Atunci
4bc
AD 2 =
p( p a); unde p-semiperimetrul triunghiului, AB=c, AC=b, BC=a.
(b + c ) 2
Rezolvare:
c BD
c + b BD + DC
ab
=
=>
=
=> DC =
(1)
b DC
b
DC
b+c
Din teorema bisectoarei
ac
BC = a DC => BD =
(2)
b+c
Din relaia lui Stewart
DA 2 a = c 2 DC + b 2 BD a DB DC
, nlocuim DC si BD cu (1)
si (2)
bc
AD 2 =
[(b + c) 2 a 2 ].
2
(b + c)

SSM
H

Probleme propuse:
1) Fie triunghiul ABC in care AD BC, D (BC), M mijlocul lui (BC), si [AD
bisectoarea unghiului BAM, iar [AM bisectoarea unghiului DAC, determinai
msura unghiului BAC.
2) In triunghiul ABC, a > b si a > c, unde AB = c, AC = b, BC = a, fie linia
mijlocie [EF], EF BC, unde E [AB] si F [AC], care intersecteaz
bisectoarele BB si CC in M, respectiv in N, M [BB], N [CC].Sa se arate ca
MB NC a + b a + c
:
+
=
+
.
MB ' NC ' a b a c
3) In triunghiul ABC se construiesc inaltimea AD, bisectoarea AA, M
mijlocul segmentului [AA], N mijlocul segmentului [AC].Daca
m( < ACB)= 22o 30, B este mijlocul segmentului[DA], m( < BDM)= 330 45,iar
BD
MN
= k , sa se calculeze
.
AC
DC
Bibliografie:
[1] Liviu Nicolae
Vladimir Boskoff
[2] Gheorghe Titeica

Probleme practice de matematica;


Ed.tehnica 1990 Bucuresti
Probleme de geometrie
Ed.Apollo1992 Craiova

- 29 -

CERCUL DE MATEMATICA
Tema pentru grupa de performanta la clasa a- VIII -a
CERCUL LUI EULER

elev Tigora Andrei


prof.dr.Ciniceanu Gheorghe

1 DATE BIOGRAFICE
Leonhard Euler (1707-1783), matematician elveian ale crui lucrri
majore au fost realizate n domeniul matematicii pure. Euler s-a nscut n
Basel i a studiat la Universitatea din Basel, obinndu-i diploma de
absolvire chiar de la vrsta de 16 ani. n 1727, la invitaia Ecaterinei I,
mprteasa Rusiei; devine membru al Academiei de tiine din Sankt
Petersburg. n 1730 a fost numit profesor de fizic, iar apoi n 1733 a
devenit profesor de matematic. La insistenele regelui prusac Frederick cel
Mare a devenit profesor de matematic la Academia de tiine din Berlin.
Euler s-a ntors la Sankt Petersburg n 1766 rmnnd aici pn la sfritul
vieii. Dei a suferit de deficiene ale vederii nc de la vrsta de 20 de ani
orbind aproape complet spre finalul vieii,a fcut descoperiri importante n
domeniul matematicii printre care se numr si Dreapta lui Euler si
Cercul lui Euler.
2 PRINCIPALUL REZULTAT
Teorema 1 ntr-un triunghi, cu notaiile obinuite (O centrul cercului
circumscris, H ortocentru, I centrul cercului nscris) punctele O, G, H sunt
coliniare i avem GH=2GO. (Dreapta lui Euler)
Demonstraie Vom sugera aici doar folosirea proprietii liniei mijlocii, i
asemnarea triunghiurilor.
Teorema 2 ntr-un triunghi oarecare mijloacele laturilor, picioarele
perpendicularelor i mijloacele segmentelor cuprinse ntre vrfuri i
ortocentru sunt nou puncte aflate pe acelai cerc. (Cercul lui Euler) (fig. 1)
Demonstraie
Fie A, B, C mijloacele [BC], [CA] respectiv [AB]
A, B, C picioarele nlimilor din A, B respectiv C
H ortocentrul ABC
O centrul cercului circumscris ABC
A, B, C mijloacele [AH], [BH] respectiv [CH]
[BC] linie mijlocie n ABCBCBC
(1)
[AB] linie mijlocie n ABCAB=1/2AB
(2)
AAB dreptunghic n A
[AC] median AC = 1/2AB
(3)
(1), (2), (3) AACB trapez isoscel
A,B,C,A conciclice
(4)
analog
A,B,C,B conciclice
(5)
A,B,C,C conciclice
(6)
(4),(5),(6) A,B,C,A,B,C conciclice
(a)
Fie O mijlocul [OH] O se afl pe mediatoarele segmentelor [AA],[BB],
[CC] O este centrul cercului pe care se afl punctele A, B, C, A, B, C .
n ACH, [AB] linie mijlocie ABCH
(7)
n ABC, [BA] linie mijlocie BA AB
(8)

- 30 -

CERCUL DE MATEMATICA
CHAB
(7), (8), (9) ABBA
n ABH, [AC] linie mijlocie ACBH
n ABC, [CA] linie mijlocie CACA
BHCA
(10), (11), (12) ACCA
(a), (b) m(ABA)+m(ACA)=180 ABAC inscriptibil
i A, A diametral opuse
Analog
BCBA patrulater inscriptibil
CACB patrulater inscriptibil
i B, B diametral opuse - C, C diametral opuse
(a), (e), (f), (g) A, B, C, A, B, C, A, B, C conciclice.

(9)

SSM
H

(b)
(10)
(11)
(12)
(c)
(e)
(f)
(g)

B''

A''
C'

B'
O
O'

H
C'
C''

B'''

A''

A'

fig.1

COROLAR
1) Centrul cercului lui Euler, O, este mijlocul segmentului [OH].
2) Raza cercului lui Euler este jumtate din raza cercului circumscris ABC.
Demonstraie Le considerm ca fiind bune exerciii pentru cei doritori de a-i
nsui temeinic tema prezentat aici.
3 APLICAIE
Fie I centrul cercului nscris n ABC i A1B1C1 triunghiul su
complementar. S se demonstreze c centrele cercurilor lui Euler ale
triunghiurilor BIC, CIA, AIB sunt situate pe bisectoarele unghiurilor
triunghiului A1B1C1. (fig. 2)
Soluie
Fie O1 centrul cercului circumscris BIC, H1 ortocentrul BIC,
[BI1 i [CI1 bisectoare exterioare ale unghiului B i unghiului C
I1[BI1d(I1,AB)=d(I1,BC)
(1)
I1 [CI1d(I1,AC)=d(I1,B
(2)

- 31 -

CERCUL DE MATEMATICA

fig. 2
I
O'
G1

A1

O1

(1), (2) d(I1,AB)=d(I1,AC) A, I, I1 coliniare


[BI1 bisectoarea exterioar a unghiului B
(3)
[BI bisectoarea interioar a unghiului B
(4)
[CI1 bisectoarea exterioar a unghiului C
(5)
[CI bisectoarea interioar a unghiului C
(6)
(3), (4) m(pIBI1)=90
(7)
(5),(6) m(pICI1)=90 (8)
(7), (8) O1 mijlocul [II1]
n BIC aplicm Teorema dreptei lui Euler
O1, G1, H 1 coliniare i [IA1] median [AA1][O1H1]={G1}
O1A1 mediatoarea [BC] O1A1BC
(9)
H1I1BC
(10)
(9), (10) O1A1H 1I1
Din Teorema fundamental a asemnrii avem
GIH1GA1O1 2A1O1=IH1
Fie [A1P bisectoarea pC1A1B1 A1PAI
A1PIO1
(11)
PIA1O1
(12)
(11), (12) O1A1P1I paralelogram PI=A1O1 H 1P=A1O
(a)
PH1A1O1
(b)
(a), (b) O1A1H1P paralelogram A1PO 1H1={O}, O mijlocul [O1H 1]
dar mijlocul [O1H1] este centrul cercului lui Euler pentru BIC.
Deci centrul cercului lui Euler n BIC apartine bisectoarei A1B1C1
Analog se demonstreaz c i centrele cercului lui Euler pentru CIA i AIB
aparin bisectoarelor A1B1C1.
Definiie Se numete omotetie de centru O i raport kR, funcia
f : P P dat de f(P) = P, OP' = k OP .

- 32 -

CERCUL DE MATEMATICA
1
i centru O (centrul
2
cercului circumscris) a cercului circumscris in orice triunghi.
Aplicaia 2. Centrul cercului lui Euler este pe dreapta lui Euler.

SSM
H

Aplicaia 1. Cercul lui Euler este o omotetie de raport

Bibliografie
[1] Gheorghe ieica - Probleme de geometrie, Editura Tehnica Bucuresti
[2] Vladimir Boscoff , Liviu Nicolescu - Probleme practice de geometrie

Tema pentru grupa de performanta la clasa a- VIII -a


Plan mediator al unui segment

prof. Prejneanu Vasile Doru


Liceul Pedagogic tefan Odobleja

Definiie Fie [AB] un segment, M mijlocul segmentului [AB], se


numete plan mediator al segmentului [AB] planul perpendicular pe dreapta
suport a segmentului AB i care o intersecteaz n punctul M.
Teorem Condiia necesar i suficient ca un punct P din spaiu s
se afle n planul mediator al unui segment [AB] este ca d (P,A)= d (P,B) sau
PA=PB, unde d (P,A)=PA i d (P,B)=PB lungimile segmentelor AP i PB.
Demonstraie Fie {M}= [AB]. Dac P , atunci PM AB. Aplicnd
teorema lui Pitagora n triunghiurile PMA i PMB avem c PA2= PM2 + MA2 i
PB2= PM2 +MB2, dar este plan mediator lui [AB], deci MA=MB, deci avem
c PA2 = PB2, deci PA=PB.
Vom demonstra acum i reciproc, presupunem PA=PB i fie plan,
AB, presupun , fie {N}= [AB] din teorema lui Pitagora n PAN i
PBN, avem PA2= PN2 + NA2 i PB2= PN2 + NB2, dar cum PA=PB, avem c
PA2=PB2 => NA2=NB2 => NA=NB, deci N mijlocul [AB] i = este chiar
planul mediator al segmentului [AB].
Aplicaii
1. S se determine locul geometric al punctelor din spaiul egal deprtate de
trei puncte A, B, C necoliniare.
Demonstraie Notm L={MMA=MB=MC; A,B,C necoliniare}
L- locul geometric cutat, vom demonstra c L=d , unde d (ABC) i d
(ABC) = {O}, O cercul circumscris ABC.
Vom determina pe d i anume fie 1 plan mediator pentru [AB] i 2
plan mediator pentru [BC] i 1 [AB]= {O1} i 2 [BC]= {O2}; 1 2=d,
fie {O}=d (ABC). n aceste condiii AO1=BO1 i BO2= CO 2 i OA=OB=OC,
deoarece O se afl n planele mediatoare ale segmentelor [AB] i [BC], deci O
este centrul cercului circumscris ABC, i deci OAB i OBC isoscele i
deci OO1 AB i OO2 BC. Cum 1 plan mediator pentru [AB]=> AB 1
i d include 1=> AB d(1).
Cum 2 plan mediator pentru [BC]=> BC 2 i d include 2=> BC d (2).
Din (1) i (2) avem c d (ABC), deci am construit dreapta d. Acum fie M
L, atunci MA=MB=MC, i fie O centrul cercului circumscris ABC, atunci
AO=BO=OC, cum MO=MO=MO, atunci din cazul (LLL) MAO MBO

- 33 -

CERCUL DE MATEMATICA
MCO=>m (BMO)=m (CMO)=m (AMO)= 90 grade => MO (ABC), deci M d ,
deci L include d (3).
Fie M d, {d}= 1 2, 1 i 2 plane mediatoare pentru [AB] i [BC],
atunci MA=MB=MC, deci d include L (4). Din (3) i (4) L=d.
2. Planele mediatoare a segmentelor determinate de trei puncte necoliniare
au o dreapt comun ce reprezint perpendiculare pe planul triunghiului
determinat de acela trei puncte ridicat n centrul cercului circumscris
acestuia.
Demonstraie Fie punctele A,B,C necoliniare i O centrul cercului
circumscris ABC, i fie d dreapta de intersecie a planelor 1 mediator
segmentului [AB] i 2 mediator segmentului [BC], despre care am artat n
aplicaia 1 c este perpendicular pe planul (ABC) n punctul O. Vom arta
c d se gsete i n planul mediator segmentului [AC]. Fie 3 planul
mediator segmentului [AC], 3 [AC] = {O3}, cum OA=OC, OAC isoscel i
deci OO3 AC, fie M d, d (ABC) OO3 AC, atunci din teorema celor trei
perpendiculare avem c MO3 AC=> O3MA O3MC cazul catet catet i
deci MA=MB, deci MO3 include d, dar MO3 include 3, deci d include 3,
deci d = 1 2 3.
3. Dac A,B,C,D sunt patru puncte necoplanare, s se demonstreze c
exist un punct O egal deprtat de ele.
Demonstraie Fie d dreapta pe care se gsesc punctele egal deprtate
de punctele B,C,D ca n aplicaia 2, care este perpendicular pe planul
(BCD) n centrul cercului circumscris BCD, fie planul mediator al
segmentului [AB]. Dac d, atunci AB include (BCD) contrar ipotezei, deci
exist un punct {O}= d , dar cum O , O este egal deprtat de A i B,
dar i de B,C,D pentru c se gsete pe dreapta d, deci O este punctul
cutat.

Tema pentru grupa de performanta la clasa a- VIII -a


Planul bisector
prof. Bloi Valeria
Liceul Pedagogic tefan Odobleja
Prin intersecia a dou plane i se obin patru unghiuri diedre,
oricare dou dintre ele avnd msuri egale sau suplementare.
Notm
cu
cu
1 , 2 , 1 , 2
semiplanele n care dreapta d mparte
planul i . Fie M d i dreptele
MM1 , MM2 ,MM1 d, MM2 d.
Definim unghiul diedru al celor 2
plane, unghiul a crui msur 90 0 .
Dac m( M 1 M M 2 ) 90 0 , atunci unghiul
diedru dintre planele i este unghiul
format de semiplanele 1 si 1 .

- 34 -

CERCUL DE MATEMATICA
Definiie Se numete plan bisector al diedrului determinat de
semiplanele 1 si 1 , planul cu proprietatea c unghiurile diedre
determinate de 1 si , respectiv 1 si cu aceeai msur.
Planul se mai numete i plan bisector interior analog cu
bisectoarea interioar a unghiului plan.
Planul ' perpendicular pe planul i cu care se intersecteaz dup
dreapta d se numete planul bisector exterior al diedrului format de
semiplanele 1 si 1 .
Teorema1 Oricare ar fi un punct M din planul bisector al diedrului
format de semiplanele 1 si1 , distana de la acest punct la cele dou
semiplane este aceeai.
Demonstraie:
Fie M i notm cu
M1=pr1 (M)
M2=pr1 (M)
M= prd (M)
Conform reciprocei teoremei celor
perpendiculare, MM1 d i MM2 d .

SSM
H

)
)
Din ipotez m(M 1 M ' M ) = m(M 2 M ' M ), m( M 1 ) = m(M 2 ) = 90 0 , [MM] latur comun

=> conform cazului (catet, catet) c MMM1 MMM2


Prin urmare [MM1]
[MM2] i deci d(M, 1 )=d(M, 1 ). Se poate
demonstra i reciproc, adic dac dac M are proprietatea c distanele la
cele dou semiplane sun egale, atunci M aparine planului bisector.
Teorema 1 se poate formula ca loc geometric : Locul geometric al punctelor
din spaiu egal deprtate de feele unui unghi diedru este planul bisector al
unghiului diedru.
Obs. Planul bisector interior i exterior sunt dou plane perpendiculare,
proprietate analoag cu cea referitoare la bisectoarele a dou unghiuri plane
suplimentare.
Teorema2 (Teorema planului bisector)
Planul bisector al unui unghi diedru al unui tetraedru, mparte
muchia opus n dou segmente ale cror lungimi sunt direct proporionale
cu ariile feelor diedrului.
Mai exact, fie tetraedrul ABCD i planul bisector al diedrului format de
feele (ABC) i (BCD). Dac E este intersecia planului bisector cu AD,
atunci:
AE A[ ABC ]
=
.
ED A[ BCD]
Demonstraie:
Fie F proiecia lui E pe planul (ABC) i G proiecia
lui E pe planul (BCD). Cum E aparine planului
bisector, [EF] [GE].
Fie A proiecia lui A pe planul (DBC) i D proiecia
lui D pe planul ABC. Cum EF||DD`=> punctele

- 35 -

CERCUL DE MATEMATICA
F,E,D,D` sunt coplanare i (FED) I (ABC)=AD`. Analog se demonstreaz c
D,G,A` sunt coliniare. Aplicnd teorema fundamental a asemnrii =>
AE
FE
EG ED
=
i analog
=
.
Dar EG=EF
AD DD '
AA' AD
AE DD ' ED AA'
AE AA'

=
AE DD ' = ED AA '
=
AD
AD
ED DD '
Scriem acum volumul tetraedrului n dou moduri:
3V
3V
DD ' A[ ABC ] AA' A[ BCD ]
V=
=
de unde AA' =
i DD' =
3
3
A[ BCD]
A[ ABC ]
3V
AE A[ BCD ]
AE A[ ABC ]
Deci
.
=

=
3
V
ED
ED A[ BCD]
A[ ABC ]
Aplicaii:
)
)
AB
2
1. Fie tetraedrul ABCD cu m ( ABC ) = 600 ; m ( DBC ) = 450 i
=
. Fie
BD
3
E (AD). S se arate c dac E se afl n planul bisector al diedrului format
de planele (ABC) i (BCD), atunci E este mijlocul lui AD.
Soluie
E fiind n planul bisector
)
AE A[ ABC ]
AE AB BC sin ABC

=
)
ED A[ BCD ]
ED BD BC sin DBC
0

AE AB sin 60
AE
=

=
0
ED BD sin 45
ED

AE
=
ED

2
3

3
2

3
2 2

3
2
2

AE
= 1 AE = ED E
ED

este mijlocul al lui AD.


2. Fie tetraedrul ABCD cu AB=AC i BD=DC. Considerm planul bisector
al diedrului format de planele (ABC) i (BCD), care intersecteaz AD n E.
)
Dac M mijlocul lui BC, s se arate c ME este bisectoarea unghiului AMD .
Soluie:
Cum E aparine planului bisector
AE A[ ABC ]
AE BC AM
AE AM
=

=
.
ED A[ BCD ]
ED BC DM
ED DM
Conform reciprocei teoremei bisectorei n plan =>
(ME este bisectoarea AMD deoarece mparte latura
AD n pri proporionale cu laturile unghiului
)
AMD .

- 36 -

CERCUL DE MATEMATICA

SSM
H

Tema pentru grupa de performanta la clasa a-IX-a


Ecuaia lui Pell i aplicaii
eleva Coand Oana clasa a X-a CNT
eleva Cucu Irina clasa a X-a CNT
prof. dr.Ciniceanu Gheorghe CNT
1 INTRODUCERE
Ecuaiile diofantice sunt o categorie de probleme matematice foarte
studiate de-a lungul istoriei. Ecuaiile diofantice sunt ecuaiile ale cror
soluii se caut n mulimea numerelor ntregi.
Iat cteva exemple de astfel de ecuaii:
1. Rezolvai n Z *Z
ecuaia: 2x+3y=5
2. Rezolvai n Z *Z
ecuaia: xy-3x+2y=17
3. Rezolvai n Z *Z
ecuaia: x2-y2=7
4. Rezolvai n Z *Z
ecuaia: x2-2y2=1
S remarcm pentru nceput c ecuaiile 1, 2, 3 sunt elementare,
variante ale lor fiind ntlnite adesea n culegerile adresate elevilor de
gimnaziu. Cea de-a patra ecuaie se incadreaz n categoria general a
ecuaiei lui Pell i vom da n finalul acestei note forma soluiei generale.
1. y=5-2x/3 5-2x M3 2x trebuie s dea restul 2 la mprirea
cu 3, adic x=3k+1. Deci S={(3k+1,-2k+1)/ kZ }
2. xy-3x+2y=17 xy-3x+2y-6=11 (y-3)(x+2)=11
Soluiile vor fi date de rezolvarea sistemelor:
y-3 = -11
y-3= -1
y-3= 1
x+2= -1
x+2= -11
x+2= 11

y-3=11
x+2= 1

deci (x,y) {(-3,-8); (-13,2); (9,4); (-1,14)}


3. Este rezolvabil prin acelai procedeu cu ecuaia numrul 2
(x-y)(x+y)=7
x-y= -1
x-y= -7
x-y= 1
x-y= 7
x+y= -7
x+y= -1
x+y= 7
x+y= 1
deci (x,y) {(-4,-3);(-4,3);(4,3);(4,-3)}
2 PRINCIPALUL REZULTAT
Teorem. Ecuaia x 2-my2= 1, m nu este ptrat perfect, (ecuatia lui
Pell) are o infinitate de soluii. Dac (x1,y1) este soluia cu valorile cele mai
mici, diferit de (1,0) i (xn,yn) este soluia general, atunci
xn+1= x1xn+my 1y n
yn+1=y1xn+x 1y n
Demonstraie
Se poate gsi n [1].
Soluia problemei 4.
S ne folosim de Teorem pentru soluionarea problemei 4
Se observ c (3,2) este soluie. Deci xn+1=3xn+4yn; yn+1=2xn+3yn de unde
x2=3x1+4y1=17; y2=2x1+3y1=12 i ntr-adevr 172-2*122= 289-288 =1.
Folosind Teorema, deducem c forma general a soluiei ecuaiei 4 este:
xn+1=3xn+4y n
yn+1=2xn+3y n

- 37 -

CERCUL DE MATEMATICA
3 APLICAII
1. S se demonstreze c exist o infinitate de triunghiuri avnd laturile
numere naturale consecutive, iar aria exprimat prin numr natural.
Soluie: Fie x-1,x,x+1 laturile. S N x nu e impar deci x=2y i x2-4=3u2.
Deci 4y2-4=3u2 deci u e par u=2v. Atunci y2-3v 2 = 1 deci ecuaia Pell.
Are soluia (2,1) i deci
xn=1/2[(2+3)n+(2-3)n]
yn=1/23[(2+3)n+(2-3)n], n 1
Lungimile laturilor sunt 2xn-1, 2xn , 2xn+1 iar aria S=3xnyn .
2. Demonstrai c ecuaia (x - y)3 + (y z)3 + (z x)3 = 30 nu are soluii n Z.
Soluie. Se factorizeaz partea stng a ecuaiei i se obine:
(x - y)(y - z)(z - x) = 10:
Se observ c (x - y) + (y - z) + (z - x) = 0. Scriem toi divizorii numrului 10,
adic 1; 2; 5; 10. Se verific nemijlocit c suma oricror trei divizori ai
numrului 10, al cror produs este 10, este diferit de 0.
3. S se rezolve n Z ecuaia x2 + 1 = 3y.
Soluie. Fie x, y care verific ecuaia. Atunci x2 + 1 = 0(mod 3). Examinm
cteva cazuri, n dependen de restul mpririi numrului x prin 3.
a) Fie x 0(mod 3). Atunci x2 + 1 1(mod 3) i deci x2 + 1 0(mod 3).
b) Fie x 1(mod 3). Atunci x2 + 1 2(mod 3) i deci x2 + 1 0(mod 3).
c) Fie x 2(mod 3). Atunci x2 + 1 2(mod 3) 0(mod 3).
n consecin, congruena x2 + 1 0(mod 3) nu are soluii, i astfel ecuaia
la fel nu are soluii ntregi.
4. S se arate c ecuaia x(x + 1) = 4y(y + 1) este incompatibil n Z+ .
Soluie. Ecuaia iniial este echivalent cu ecuaia x2+ x + 1 = (2y + 1) 2
De aici rezult c x2 < (2y + 1)2 < (x + 1)2 sau x < 2y + 1 < x + 1. Contradicia
obinut implic incompatibilitatea n N* a ecuaiei iniiale.
5. S se arate c ecuaia x2 y3 = 7 nu este compatibil peste N*.
Soluie. Dac y par x2 3(mod 4), dar ultima congruen nu are loc
pentru nici un x Z. Dac presupunem c y este impar, adic y=2k+1,
atunci x2 + 1 = y3 + 8 = (y + 2)((y + 1)2 + 3) = (y + 2)(4(k + 1)2+ 3);
i deci, numrul x2 + 1 are divizori de forma 4n + 3, de unde rezult c x2+1
are divizor prim de forma 4n + 3. ntr-adevr, dac toi divizorii primi ai
numrului 4(k + 1)2 + 3 sunt de forma 4n + 1, atunci i numrul 4(k+1)2+3
va fi de forma 4n + 1.
Bibliografie:
[1] D.Andrica,T.Andreescu - Ecuaii diofantiene, Editura Gill Zalau, 2004.

- 38 -

CERCUL DE MATEMATICA

SSM
H

Tema pentru grupa de performanta la clasa a-IX-a


Inegalitati remarcabile
prof. MAGDALENA CONSTANTIN
prof. PORFIREL DRAGA TATUCU

In acest scurt articol va vom prezenta doua inegalitati remarcabile


cunoscute si aplicate de elevii din ciclurile gimnazial si liceal, carora
rezolvarea exercitiilor este astfel mult facilitata.
1) Inegalitatea mediilor.
Minim(a,b,c,.) Media armonica Media geometrica Media
aritmetica Media proportionala Maxim(a,b,c,.), unde:
n
Media armonica=
Media geometrica = n a1 * a2 * ....a n ;
;
1
1
1
+
+ .... +
a1 a 2
an
Media aritmetica =

a1 + a 2 + ... + a n
;
n

Media proportionala =

a1 + a 2 + ... + a n
n

Exercitii rezolvate:
1) Aratati ca daca a, b, c R cu a,b,c 5 si 6a+4b+3c=7, atunci:
a + 5 + 3 2b + 10 + 4 6c + 30 9 .
Aplicam inegalitatea mediilor. Adica Mg Ma.
1 + ( a + 5)
a + 5 poate fi scris ca : 2 1 * ( a + 5)
(1)
2
1 + 2 + (b + 5)
3 2(b + 5) = 3 1 * 2 * (b + 5)
( 2)
3
1 + 2 + 3 + ( c + 5)
4 6(c + 5) = 6 1 * 2 * 3 * (c + 5)
(3)
4
a + 6 b + 8 c + 11
1(a + 5) + 3 1 * 2(b + 5) + 6 1 * 2 * 3(c + 5)
+
+
=
2
3
4
6a + 36 + 4b + 32 + 3c + 33 6 a + 4b + 3c + 101
=
=
dar..6 a + 4b + 3c = 7
12
12
7 + 101
a + 5 + 3 2b + 10 + 4 6c + 30
=9
12
(Etapa locala - Alba )
2) Fie a, b, c numere naturale nenule. Sa se demonstreze inegalitatea:
a2 b2 c2 a b c
+
+
+ +
b2 c2 a2 c a b
Folosim inegalitatea mediilor(Ma Mg)
a2 b2 1
a2 b2
a2 a
a2 b2
a
2 + 2 *
*
=
=

+ 2 2
2
2
2
2
c
c
c 2
b c
c
b
c
b

b2 c 2 1
2 + 2 *
a 2
c

b2 c 2
b
b 2 c/ 2 b
*
=

+ 2 2
2
2
2
a
a
c/
a
c
a

- 39 -

CERCUL DE MATEMATICA
c2 a2 1
c 2 a/ 2 c
b2 c2
c
2 + 2 *
*
=

+ 2 2
2
2
2
b
b
b 2
a/
b
c
a
a
Adunam cele trei relatii obtinute:
a2 b2 b2 c2 c2 a 2
a2 b2 c2 a b c
a b c
+
+
+
+
+

2
+
+

+
+
+ +

b2 c 2 c 2 a2 a2 b2
b2 c2 a2 c a b
c a b
3) Fie a1, a2, .. ,an (0,1)
Daca a1a2an =1 aratati ca (1+ a1)(1+ a 2).(1+ an) 2 n
Folosim inegalitatea mediilor: Ma Mg
1 + a1
1 + a2
1 + an
a1 ;
a 2 ; ...
an
2
2
2
(1+ a1)(1+ a2).(1+ an) 2 n a1 * a2 * .... * an (1+ a1)(1+ a2).(1+ an) 2 n
1442443
=1

(Cluj etapa locala)

Exercitii propuse:
1) Fie a, b, c, x, y, z R care verifica conditiile:
a+b+c=1 si x 2 y 2 + z 2 = 1
Aratati ca : a(x+b) + b(y+c) + c(z+a) 1
(Concursul Nicolae Paun - 2003 )
2) Aratati ca pentru orice a, b, c >0 si k natural nenul are loc inegalitatea:
1
1
1
a+b+c
+ k
+ k

k
a + 2k 1 a + 2k 1 a + 2k 1 2k abc
(Concursul Alexandru PapiuIlarian-2003)
3) Daca a, b, c, d sunt numere reale strict pozitive, demonstrati ca :
a+b
bc
c+a
4(a + b + c)
+
+

c+d
a+d
b+d a+b+c+d
(Concursul Gheorghe Dumitrescu - 2003)
2) Inegalitatea lui Cauchy-Buniakowski-Schwartz (C.B.S):
2

n 2 n
a i xi a i xi , egalitate daca aixj = ajxi , i j ,i, j 1, n
i =1

i =1
i =1
2
2
2
2
2
2
2
sau: (a1 x1 + a 2 x 2 + ...a n x n ) a1 + a 2 + ....a n x1 + x 2 + ... + x n .
n

)(

Exercitii rezolvate:
1) Fie a, b, c>0. Sa se arate ca

a
b
c
2
(a + b + c) 2
+
+

(b + c) 2 (a + c) 2 (a + b) 2 3 (a + b)(b + c)(c + a )

Aplicam C.B.S si obtinem


a
b
c
(a + b + c) 2
+
+

(b + c) 2 (a + c) 2 (a + b) 2 a (b + c) 2 + b(a + c) 2 + c(a + b ) 2
Aratam ca :

- 40 -

2
(a + b + c ) 2
(a + b + c) 2

3 (a + b)(b + c)(c + a ) a (b + c) 2 + b(a + c) 2 + c(a + b ) 2

3
( a + b)(b + c)(c + a ) a (b + c) 2 + b (a + c) 2 + c( a + b) 2
2

CERCUL DE MATEMATICA

SSM
H

3(a + b)(b + c)(c + a) 2a(b + c) 2 + 2b(a + c) 2 + 2c(a + b) 2


3( ab + b 2 + ac + bc)(c + a ) 2a (b 2 + 2bc + c 2 ) 2b( a 2 + 2ac + c 2 ) 2c( a 2 + 2ab + b 2 ) 0
3( abc + b 2 c + ac 2 + bc 2 + a 2 b + a 2 c + abc) 2ab 2 4abc 2ac 2 2ba 2 4acb 2cb 2 0
6abc + 3b 2 c + 3ac 2 + 3bc 2 + 3a 2 b + 3ab 2 + 3a 2 c 2ab 2 12abc 2ac 2 2a 2 b 2c 2 b
2 ac 2 2b 2 c 0
6abc + b 2 c + ac 2 + bc 2 + a 2 b + ab 2 + a 2 c 0
Avem: c 2 a + ab 2 2 abc + b 2 c + ca 2 2abc + a 2 b + c 2 b 2abc 0
a( c b) 2 + b( a c) 2 + c(b a ) 2 0

relatie adevarata
a, b, c > 0

a
b
c
2
(a + b + c) 2

+
+

(b + c) 2 (a + c) 2 (a + b) 2 3 (a + b )(b + c)(c + a )

2) Demonstrati inegalitatea (1!) 2 + (2!) 2 + .... + (n!) 2


Avem: [(1!) 2 + ( 2!) 2 + .... + ( n!) 2 ]

6[(n + 1)!1]
, ( )n 1
n (n + 1)(2 n + 1)

n( n + 1)( 2n + 1)
[( n + 1)!1]
6

n(n + 1)(2n + 1)
= 12 + 2 2 + ....n 2
6
[(1!) 2 + ( 2!) 2 + .... + ( n!) 2 ] * [12 + 2 2 + ....n 2 ] > 1 [(n + 1)!1]
Aplicam C.B.S pt relatia din stanga si comparam rezultatele:
[(1!) 2 + ( 2!) 2 + .... + ( n!) 2 ] * [12 + 2 2 + ....n 2 ] (1!*1 + 2!*2 + ... + n!*n) = [(n + 1)!1] 2
Cum: [(n + 1)!1] 2 [(n + 1)!1]
6[(n + 1)!1]
, ( )n 1
inegalitatea: (1!) 2 + (2!) 2 + .... + (n!) 2
n (n + 1)(2 n + 1)
Dar

Probleme propuse:

y
x
z
+
+
< 3
1+ x 1+ x + y 1 x + y + z
(Concursul Cristian Calude 2003)
2) Sa se determine cea mai mica si cea mai mare valoare a expresiei:
2
2
2
2
A= x1y1+ x2 y2 daca x1 +x 2 2 si y1 + y 2 4
1) Fie x, y, z 0. Demonstrati ca

3) Demonstrati ca daca x, y, z R, atunci ( x + y + z ) 3(x 2 + y 2 + z 2 ).


2

- 41 -

CERCUL DE MATEMATICA
Tema pentru grupa de performanta la clasa a-IX-a
Funcia caracteristic a unei mulimi
1Introducere
Definiie. Fie x . Pt. A P(x), definim f A : x {0,1},

Prof. Giugiuc Leonard


C.N.Traian

1, x A, unde A = X \ A. A numeste functie


f A ( x) =

0, x A caracteristica a lui A

Vom da n continuare, cteva proprieti ale funciilor definite mai sus.


P1 : A B f A f B
P4 : f A B = f A f B
P5 : f A B = f A + f B f A f B

P2 : A = B f A = f B

P3 : f X = 1, f = 0
P6 : f A + f A = 1
Lsm cititorului plcerea descoperirii demonstraiilor acestor proprieti

2 Principalele rezultate
Teorema 1. Fie A, B P(x). Definim AB = ( A \ B ) (B \ A)(diferenta simetrica ).
S se arate c este asociativa pe P(x).
Demonstraie Avem de artat c ( AB )C = A (BC ) f ( AB )C = f A ( BC )

f AB = f ( A\ B )( B \ A ) = f (A B )(B A) = f A B + f B A f A D B A = f A f B + f B f A 0 = f A (1 f B ) +
1
1 1

+ f B (1 f A ) = f A + f B 2 f A f B = 2 f A f B +
(1)
2
2 2

folosim (1) i avem:



1
1 1
1
1 1 1
1 1

f ( AB )C = 2 f AB f C + = 2 2 f A f B + f C + =
2
2 2
2
2 2 2
2 2

1
1
1 1

= 4 f A f B f C +
2
2
2 2

1
1
1 1

Analog artm c f A ( BC ) = 4 f A f B f C + .
2
2
2 2

Deci, f ( AB )C = f A ( BC ) ( AB )C = A ( BC ).

Teorema 2. ( ) A, B, C P( x), A (BC ) = ( A B )( A C ).


Demonstraie
f A( BC ) = f A f BC = f A ( f B + f C 2 f B f C ) = f A f B + f A f C 2 f A f B f C .

f ( A B ) ( AC ) = f A B + f AC 2 f A B f AC = f A f B + f A f C 2 f A2 f B f C =

= f A f B + f A f C 2 f A f B f C f A2 = f A
Din (1) i (2) rezult A (BC ) = ( A B ) ( A C ) .
3 APLICATII
1. Fie A1 , A2 ,......., An P( x). Calculai f A1 A2 ......... An

- 42 -

(1)
(2)

CERCUL DE MATEMATICA
Soluie. f A1 A2 = f A + f B f A f B = ( f A 1)( f B 1) + 1. Utiliznd P.I.M., artm uor

SSM
H

c f A1 A2 ... An = ( 1) n 1 ( f A1 1)...( f An 1) + 1.
2. Fie A1, A2, ..., An P(x). Calculai f A1A2 ...An
1
1 1

Soluie. f A1A2 = f A1 + f A2 2 f A1 f A2 = 2 f A1 f A2 + .
2
2 2

1
1
1 1

Cu ajutorul P.I.M., obinem c f A1A2 ...An = (2)n 1 f A1 f A2 ... f An +


2
2
2 2

3. Rezolvai ecuaia AY = B, unde x = [o, ] , A = [0,2], B = [1,3].


Soluie. Am demonstrat cu ajutorul funciilor caracteristice c ""
asociativ. Este uor de artat c AB = BA, A = A, AA = , ( )A, B P(x).
Deci, AY = B A( AY ) = AB ( AA)Y = AB Y = AB Y = AB.
Evident, AB = [0,1] [2,3], deci Y = [0,1] [2,3].
4. Fie A1 = [0,2], A2 = [1,3], A3 = [ 2,4]. Calculai A1 A2 A3
Soluie. Avem x A1A2 A3 f A1A2A3 ( x) = 1.

1
1
1 1

Din problema 1), f A1A2 A3 = 4 f A1 f A2 f A3 + .


2
2
2 2

Deci, f A1A2A3 ( x) = 1 {f A1 ( x) = 0, f A2 ( x) = 0, f A3 ( x) = 1 } sau

{f
{f

} sau {f A ( x) = 0, f A ( x) = 1, f A ( x) = 0 } sau
A ( x ) = 1, f A ( x ) = 1, f A ( x ) = 1 } {x A1 , x A 2 , x A3 } sau {x A1 , x A 2 , x A 3 }
sau {x A1 , x A2 , x A 3 } sau {x A1 , x A2 , x A3 } x (3,4] sau x [0,1) sau
x sau x {2}. Deci, A1A2 A3 = [0,1) {2} (3,4]
A1

( x) = 1, f A2 ( x) = 0, f A3 ( x) = 0
2

Metoda 2: Din problema 5),


1
1 1

A1A2 = [0,1) (2,3] A1 A2 A3 = ( A1A2 ) A3 f A1A2 A3 = 2 f A1A2 f A3 +


2
2 2

x A1 A2 A3 x A1A2 si x A3 , sau x A1A2 si x A 3 x {2} {3,4}

Sau x [0,1) x [0,1) {2} (3,4].

4 Probleme Propuse
1. Fie A1 = [0,2], A2 = [1,3], A3 = [3,4], A4 = [3,5]. Calculai: A1A2 A3 A4 .
2. Fie AK = [K 1, KM ], K = 1, M . Calculai: A1A2 ...An .

3. Calculai: ( A1 A2 )( A2 A3 )( A3 A4 ).
4. S se arate c A B=(A B) \ (A B).
5. Dac A, B sunt mulimi finite, demonstrai formula
card(A B) = card(A) + card(B) - 2card(A B)
6. Dac A1,, An sunt mulimi finite, s se gseasc o formul pentru
calcularea card(A1 A2 An), i s se demonstreze.
Bibliografie:
[1] Mihail Megan, Traian Ceauu
INTRODUCERE N ANALIZA MATEMATIC, Editura Sidona 2002
[2] D.Busneag, V.Maftei
Teme pentru cercurile de matematica, EDP Bucuresti 1983

- 43 -

CERCUL DE MATEMATICA
Tema pentru grupa de performanta la clasa a-XI-a
Continuitatea uniform a funciilor

prof. Ticui Ovidiu

Vom prezenta pentru nceput cteva noiuni teoretice i o serie de


rezultate utile n studiul uniform continuitii funciilor.
Definiia 1 O funcie f : E R R este uniform continu pe mulimea E
dac > 0, = ( ) > 0 astfel nct, x, x E cu proprietatea c x x < ,
s avem c f ( x ) f ( x ) < .
Exemplul 1 Fie funcia constant f : R R, f(x)=c, c R, x R.Atunci
funcia f este uniform continu pe R.
ntr-adevr x , x R , obinem c: f ( x ) f ( x ) = c c = 0

> 0, = 1 > 0 astfel nct

x, x R cu proprietatea c x x < , avem

c f ( x ) f ( x ) = 0 < funcia f este uniform continu pe R.


Exemplul 2 Fie aplicaia identic f : R R, f(x)=x. Atunci funcia f este
uniform continu pe R.
ntr-adevr x , x R , avem c f ( x ) f ( x ) = x x > 0, = ( ) = > 0 ,
astfel nct

x , x R cu proprietatea c x x < , avem c

f ( x ) f ( x ) = x x < funcia f este uniform continu pe R.

Teorema 1 (Heine) Fie E R o mulime i f : E R o funcie. Atunci funcia


f este uniform continu pe E [ ( xn )n 0 , ( y n )n 0 E dou iruri cu proprietatea
c x n y n 0 f ( x n ) f ( y n ) n
0]

Demonstraie
[] Presupunem c funcia f este uniform continu pe mulimea E
[ > 0, = ( ) > 0, x , x E : x x < f ( x ) f ( x ) < ] (*).Fie irurile

( x n )n 0 , ( y n )n 0 E

cu proprietatea c x n y n n
0 N N astfel nct

(*)

x n y n < , n N f ( xn ) f ( y n ) < , n N f ( x n ) f ( y n ) n
0

[]

Presupunem prin reducere la absurd c funcia f nu este uniform


continu pe E > 0, > 0, x , x E : x x < i
1
, n 1 astfel nct
n
x n y n < n n
0 (**) i f ( xn ) f ( y n ) (***).Pe de alt parte, din (**)

f ( x ) f ( x ) ( x n )n0 , ( y n )n0 E i n =

f ( x n ) f ( y n ) n
0 , ceea ce contrazice relaia (***).n concluzie, obinem

c funcia f este uniform continu pe mulimea E.


Teorema 2 Orice funcie f : E R R uniform continu pe mulimea E
este continu pe mulimea E.
Demonstraie Fie x0 E i f : E R o funcie uniform continu pe
mulimea E > 0, = ( ) > 0 astfel nct, x, x E cu proprietatea c
x x < , s avem c f ( x ) f ( x ) < . n particular, pentru

- 44 -

CERCUL DE MATEMATICA
x = x E , x = x 0 E

SSM
H

> 0, = ( ) > 0 astfel nct, x, x0 E cu

proprietatea c x x0 < , s avem c f ( x) f ( x0 ) < f este continu n


x0 , x 0 E f este continu pe mulimea E.
Observaia 1 Reciproca acestei teoreme nu este adevrat.
Contraexemplu Fie funcia f : R R, f ( x) = x 2 . Evident c funcia f este
continu pe R i vom demonstra c funcia f nu este uniform continu pe R.
Presupunem prin reducere la absurd c funcia f este uniform
continu pe R i fie = 1 > 0 astfel nct x, y E : x y < avem c

f ( x) f ( y) = x 2 y 2 < 1 2 x + 2 = x 2 ( x + ) < 1, x R
2

1
(1 + 2 ), x R (absurd) f nu este uniform continu pe R
2
Teorema 3 O funcie continu pe o mulime compact(nchis i mrginit)
este uni-form continu pe aceast mulime.
Demonstraie Fie E mulime compact i f : E R funcie continu pe E.
Presupunem prin reducere la absurd c funcia f nu este uniform continu
pe mulimea E > 0, > 0, x , x E : x x < i f ( x ) f ( x ) .
x <

Fie n =

1
1
, n 1 ( xn )n1 , ( x n )n1 E astfel nct x n x n < i f ( x n ) f ( xn ) (*)
n
n
Lema

( )

Mulimea E fiind mrginit, (x n )n1 este un ir mrginit x n p


subir conver-gent al irului ( x n )n 1 , cu proprietatea c

Cesaro

p 0

un

x n p
x0 E = E (avnd n vedere c mulimea E este o mulime nchis).
p
x n p xnp <

1
0 xn p xnp
0 xnp = x n p ( x n p x np )
x0 0 =
p
p
n p p

x0 x n p
x 0 , x np
x0 f ( x n p )
f ( x 0 ), f ( x np )
f ( x0 )
p
p
p
p

f ( xn p ) f ( x np )
0,
p
eea ce contrazice relaia (*) f este uniform continu pe mulimea E.
Definiia 2 Spunem c funcia f : E R R este lipschitzian pe mulimea
E, dac M > 0 astfel nct f ( x ) f ( x ) M x x , x , x E .Aceast
inegalitate se numete condiia lui Lipschitz.
Teorema 4 Dac funcia f : E R R este lipschitzian pe mulimea E,
atunci fucia f este uniform continu pe mulimea E.

Demonstraie Fie > 0 i alegem = ( ) =


> 0 x, x E , cu proprietatea
M

c x x < =
,avem c: f ( x ) f ( x ) M x x < M = M
=
M
M
f este uniform continu pe mulimea E.
Observaia 2 Dac f i g sunt funcii uniform continue pe mulimea E,
atunci funciile f +g i f ( R) sunt uniform continue pe mulimea E, prin
urmare mulimea funciilor uniform continue pe mulimea E mpreun cu
operaiile de adunare a funciilor i respectiv de nmulire a unei funcii cu
un scalar, formeaz o structur algebric de spaiu vectorial.

- 45 -

CERCUL DE MATEMATICA
Observaia 3 Produsul a dou funcii uniform continue pe o mulime nu
este,n gene-ral, o funcie uniform continu pe acea mulime.
Exemplul 3 Fie funciile f,g: R R, f ( x) = g ( x) = x .Avem evident c fiecare
dintre funcii este uniform continu pe mulimea R, n timp ce produsul
celor dou funcii, adic funcia fg: R R, ( fg )( x) = f ( x) g ( x) = x 2 nu este
uniform continu pe mulimea R.
Teorema 5 Orice funcie f: R R periodic i continu pe mulimea R este
uniform continu pe mulimea R
Demonstraie Funcia f fiind periodic T > 0 : f ( x + T ) = f ( x), x R.(*)
Avem evident c [0, T ] este o mulime mrginit i nchis, deci compact i
cum funcia f este continu pe R, deci pe [0, T ] f este uniform continu
(*)
pe [0, T ] > 0, = ( ) > 0, x, y [0, T ], x y < , avem c f ( x) f ( y ) <
2

f ( x) f ( y ) < , x, y [nT , (n + 1)T ], x y < , n Z. Fie x, y R astfel nct


2
x y < .Avem c n Z astfel nct x, y [nT , (n + 2)T ]
f ( x) f ( y ) f ( x) f ((n + 1)T ) + f ((n + 1)T ) f ( y )


+ = f este uniform
2 2

continu pe R.
Teorema 6 Fie E R o mulime mrginit i f : E R o funcie uniform
continu pe mulimea E. Atunci f poate fi prelungit n mod unic la o funcie
uniform continu g : E R.
Demonstraie Fie z E \ E E fixat g ( z ) = lim f ( x ) R. Dac z E atunci
xz

definim g(z)=f(z) i astfel obinem o funcie g : E R care prelungete pe f.


Avem c funcia f este uniform continu pe mulimea E
[ > 0, = ( ) > 0, x, y E : x y < f ( x) f ( y ) < ](*)
Considerm z , z E \ E , z z < (x n )n0 , ( y n )n0 E astfel nct
x n n
z , y n n
z . Deoarece lim x n y n = z z <

(*)

N 1 astfel nct x n y n < , n N f ( x n ) f ( y n ) , n N


g ( z ) g ( z ) g este uniform continu pe mulimea E.

Aplicaii rezolvate
1. S se arate c funcia f : (0,1] R, f ( x) = x sin

1
este uniform continu.
x

1
x x
0 f poate fi prelungit prin
0
x
1

x sin , x (0,1]
continuitate la [0,1] prin funcia g : [0,1] R, g ( x) =
. Evident
x
0, x = 0
Soluie: Avem c 0 f ( x) = x sin

avem c g este continu pe [0,1] , care este o mulime compact g este


uniform continu pe [0,1] f este uniform continu.

- 46 -

CERCUL DE MATEMATICA
2. S se arate c orice funcie de gradul I este o funcie uniform continu.

Soluie: Fie f :R R, f(x)=ax+b, a R , b R i > 0 .Alegem = > 0


a

SSM
H

x, y R, cu x y < , avem c: f ( x) f ( y) = a x y < a = f este


uniform continu.
3. Fie f :R R o funcie astfel nct f ( x + y ) f ( x) + f ( y ), x, y R i
f (0) = 0. Atunci f este uniform continu f este continu n x=0.
Soluie: [ ] Presupunem f este uniform continu pe R f este continu pe
R f este continu n x=0.
f ( x) f ( y ) f ( x y )
[] f ( x) = f ( x y + y ) f ( x y ) + f ( y )
f ( y) f ( x) f ( y x)
f ( x) f ( y ) max{ f ( x y), f ( y x)} (*).Deoarece f este continu n x=0

> 0, > 0 astfel nct f ( x) = f ( x) f (0) < , x R,


(*)

x < f ( x) f ( y ) < , x, y R, cu x y < f este uniform continu pe R.

4. S se arate c funcia f : (0,1) R, f ( x) = sin


Soluie: Fie irurile x n =

, yn =

nu este uniform continu.


x

x n y n n
0 i

1
3
+ 2n
+ 2n
2
2

3
f ( x n ) f ( y n ) = sin( + 2n ) sin(
+ 2n ) n
2 0 f nu este uniform

2
2
conti-nu pe (0,1).
Aplicaii propuse
1. Artai c funcia f : (0,1] R, f ( x) = x cos

1
este uniform continu
x

1
sin x este uniform continu
x
1
3. Artai c funcia f : (0,1] R, f ( x) = arcsin x este uniform conti-nu.
x

4. Artai c funcia f : (0,1) R, f ( x) = cos nu este uniform continu


x
5. Studiai uniform continuitatea urmtoarelor funcii f , g , h : R R,
x
x
f ( x) = 1 + x 2 , g ( x) =
, h( x ) = e .
2
1+ x

2. Artai c funcia f : (0,1] R, f ( x) =

Bibliografie:
[1] Gh.Sirechi - Calcul diferenial i integral vol.1,2 Editura tiinific i
Enciclopedic, Bucureti,1985
[2] M.Niculescu,N.Dinculeanu,S.Marcus Manual de analiz matematic
vol.1, Editura Didactic i Pedagogic, Bucureti,1962.

- 47 -

CERCUL DE MATEMATICA
Tema pentru grupa de performanta la clasa a-XII-a
Formele integrale ale inegalitilor clasice

prof. Daniel Sitaru


C.N.Economic Theodor Costescu

Inegalitile guverneaz matematica, egalitile sunt doar un caz


special al acestora. Scopul acestei note matematice este acela de a prezenta,
n paralel, inegalitile clasice n form elementar i n form integral.
1. Inegalitatea modulelor
- forma elementar:
Dac a1, a2, .,anR, nN* atunci
a1 + a 2 + ... + a n a1 + a 2 + ... + a n
- forma integral:
Fie f:[a,b] R o funcie continu. n aceste condiii:
b

f ( x)dx f ( x)dx
2. Inegalitatea mediilor
- forma elementar:
Dac a1, a2, .,anR*, nN* atunci
n

a
i =1

2
i

a
i =1

2
i

a
i =1

n
n

i =1 ai

m;

M = max a i , m = min ai
i =1, n

i =1, n

- forma integral:
Fie f:[a,b] [m,M] o funcie continu. n aceste condiii:
1

ln f ( x )dx
ba
b a
1 b
.m b
e a

f ( x )dx M
baa
dx
a f (x )

3. Inegalitatea Cauchy-Buniakovski
- forma elementar:
Dac a1, a2, .,an, b1, b2, .bnR*, nN* atunci

(a1b1 + a2b2 + ............ + anbn )2 (a12 + a22 + ............. + an2 )(b12 + b22 + ............. + bn2 )

forma integral:
Fie f, g : [a,b] R funcii integrabile. n aceste condiii:
b

- 48 -

b
f ( x )g ( x ) dx f
a

b 2

(x )dx g (x )dx .
a

CERCUL DE MATEMATICA

SSM
H

4. Inegalitatea Holder
- forma elementar:
1 1
Dac p>1 i + = 1 atunci:
p q
1

n
q
n
p p
q
a i bi a i bi

i =1
i =1
i =1

- forma integral:
Fie f, g : [a,b] R funcii integrabile. n aceste condiii:
n

.
a

b
pb
q
p
q
f ( x )g ( x ) dx f ( x ) dx g (x ) dx
a
a

5. Inegalitatea Minkovski
- forma elementar:
Dac a1, a2, .,an , b1, b2, .bnR, nN* , pN ; p2 atunci:
1

n
n
n p p
p p
p p
a i + bi a i + bi
i =1

i =1

i =1

- forma integral:
Fie f, g : [a,b] R funcii integrabile i p1. n aceste condiii:
1

b
p b
p b
p
f ( x ) + g ( x ) p dx f ( x ) p dx + g (x ) p dx .

n cazul n care p=2 avem:

2
2
2
[ f (x ) + g (x )] dx f (x )dx + g (x )dx

6. Inegalitatea Cebev
- forma elementar:
Dac a1, a2, .,an, i b1, b2, .bn sunt dou iruri de numere reale,
ambele cresctoare, atunci:
a1 + a 2 + ....... + a n b1 + b2 + ........ + bn a1b1 + a 2 b2 + ............ + a n bn

n
n
n
Dac a1, a2, .,an, i b1, b2, .bn sunt dou iruri de numere reale,
monotone de sens contrar, atunci:
a1 + a 2 + ....... + a n b1 + b2 + ........ + bn a1b1 + a 2 b2 + ............ + a n bn

n
n
n
- forma integral:
Fie f, g : [a,b] R funcii monotone, de sens contrar. n aceste condiii:
b
b
b

1
f (x )dx g ( x )dx
. f ( x )g ( x )dx

b a a
a
a

Dac f, g sunt funcii monotone de acelai sens atunci:


b
b
b

1
f (x )dx g ( x )dx
. f ( x )g ( x )dx

ba a
a
a

- 49 -

CERCUL DE MATEMATICA
7. Inegalitatea Jensen
- forma elementar:
Dac f : I R este o funcie convex, atunci pentru orice x1, x2, .,xnI
i orice numere reale nenegative 1, 2, .,n, nu toate nule avem:
x + 2 x 2 + ........ + n x n 1 f ( x1 ) + 2 f ( x 2 ) + ........ + n f ( x n )

f 1 1
1 + 2 + .......... + n
1 + 2 + .......... + n
Dac 1= 2= =n inegalitatea se scrie:
x + x 2 + ......... + x n 1
f 1
[ f ( x1 ) + f ( x 2 ) + ........ + f ( x n )]
n

n
- forma integral:

:[m,M]

Fie f:[a,b] [m,M] o funcie integrabil i


continu, convex. n aceste condiii:
b
1 b

f (x )dx b a a ( f (x ))dx.
ba a

R o funcie

8. Inegalitatea Young
- forma elementar:
p

a
b
1 1
Dac a, b R; p, q >0 i + = 1 atunci ab
+
.
p q
p
q
- forma integral:
Fie f : R+ R+ o funcie continu, strict cresctoare i f(0)=0.
Pentru orice aR+ i bf(R+) are loc inegalitatea:
a

ab f (x )dx + f 1 ( y )dy.
APLICAII
1. Fie F : R R+ o primitiv a funciei continue f : R R. S se arate c
pentru orice nN* avem:
2
n
1

1
1
1
1
1 + + ......... + (F (n ) F (0 )) n f (x )dx + f ( x )dx + ............ + f ( x )dx .
n
21
n n1
2
0

prof. Daniel Sitaru


1

1
2. Fie f : [0,1] 0, integrabil. S se arate c ln
2

(1 f (x ))dx
0

f (x )dx

ln
0

1 f (x )
dx
f (x )

***

3. S se demonstreze c .

e x cos x dx e

.prof. Daniel Sitaru

- 50 -

CERCUL DE MATEMATICA
4. Fie f:[0,1] R o funcie monoton cresctoare i astfel nct

SSM
H

f ( x)dx = 0 .
1

S se arate c :. 1 + 3

1
x x

f ( x )dx 0

.prof. Daniel Sitaru

2
2
5. S se arate c e x dx e x dx 1 .
0
0

.prof. Daniel Sitaru


sin x
; x [1,2] A

1
1 1
6. Fie nN* , f : [1,2] R f ( x ) = x
, unde A = 1, , ,.............., .
n
2 3

; xA
5

2 1

S se arate c ln
dx
1 f (x )

ln f ( x )dx ln f ( x )dx .
1
1

.prof. Daniel Sitaru


7. S se arate c :
3 2 3 2
5 2 6 2 2 x dx 2 x dx 20 8 3.

2
2

.prof. Daniel Sitaru


Bibliografie
[1] Mitrinovic S. D. Analitic Inequalities Springer Verlag 1970
[2] Gh. Sirechi Calcul diferenial i integral Editura tiinific i
Enciclopedic
Bucureti 1985

- 51 -

EXAMENE
TESTE BACALAUREAT 2006

M1

prof.drd. Prajea Manuela


Subiectul I
1. Ct este tg1 + tg3 + tg5 + + tg179 ?
2. Ct este [(1 + i 3 )/2] 2006 ?
3. Cte triunghiuri se pot forma utiliznd segmente avnd lungimile din
mulimea {1,2,3} ?
4. Care este ecuaia dreptei de intersecie a planelor x-y+z=0 i x-2y-1=0 ?
5. Care este distana de la punctul A(1,0,1) la planul x+y+z=1 ?
6. Care este ecuaia tangentei la cercul x2+y2=8 ce trece prin punctul A(2,2)?
Subiectul II
1 0
}
Se consider mulimea G={A(x)M2 (C) | A(x) =
x 1
7. Ct este determinantul matricei A(x)?
8. Ct este inversa matricii A(x) ?
9. Cu ct este egal matricea B= 2A(x) A2(x) ?
10.

Care este soluia ecuaiei A(x)2006 = A(1) ?

11.

Ct este rangB, unde B=A4 + 3A2 + 2I2?


Se consider funcia f: , f(x) = e x e -x

- 52 -

12.

Ct este lim f(x) ?

13.

Ct este f(x) ?

14.

Care este semnul raportului

15.

Pe intervalul (-,0] , funcia f este convex sau concav?

16.

2
Ct este g e 1

f(x) f(y)
pentru x y R distincte ?
x y

, unde g este inversa funciei f ?

EXAMENE

SSM
H

Subiectul III
Se consider ecuaia : (P): a2-2b2 =1, (a,b)NxN , mulimile
a 2b
G={a+b 2 | a,bN, a2-2b2=1}, M={
| a, bN, a2-2b2=1}
b a
a 2b

i funcia f:GM, f(a+b 2 )=


b a
a) S se arate c

f(xy)=f(x)xf(y), oricare ar fi x, y G

b) S se deduc

f(xn)=(f(x))n pentru orice xG i orice nN*.

c) Dac XM, utiliznd b), s se determine Xn , nN*


d) Dac (a1,b1) e soluia ecuaiei (P) cu a1,b1N* i b1 minim artai c a1=3.

a 3 4 a n 1
e) Dac n =

, oricare ar fi nN*,n2, s se arate c (an,bn)
b
2
3
b
n
n 1
este soluie a ecuaiei (P).
f) Utiliznd c), d) i e) s se deduc an=[( 2 +1)2n +( 2 -1)2n]/2,
bn=( 2 +1)2n-( 2 -1)2n/2 2 ,

nN, n2.

g) S se arate c ecuaia (P) admite o infinitate de soluii.


Subiectul IV
x

tn
dt , x [0,1], n N
1+ t
0

Se consider integrala I n ( x) =
a) S se arate c 0

tn
t n , ( )t [0,1] .
1+ t

b) S se arate c 0 I n ( x)

1
( )n N .
n +1

c) S se deduc lim I n ( x) = 0, x [0,1] .


n

d) S se arate c I n ( x) + I n1 ( x) =
e) S se deduc I n ( x) =

xn
, ( ) n N , x [0,1] .
n

x n x n 1 x n2
x

+
... + ( 1) n1 + ( 1) n I 0 ( x), ( ) n N *
n n 1 n 2
1

f) S se deduc ln(1 + x) = lim ( x


n

x 2 x3
xn
+ ... + ( 1) n1 )
2
3
n

- 53 -

EXAMENE
TEST BACALAUREAT 2006

M1

prof.drd. Prajea Manuela


Subiectul I
1. Ct este sin

?
3

r
r r
r r
r
2. Ct este produsul scalar al vectorilor u = 2 i 3 j , v = 3 i + j ?
3. Ct este aR tiind c dreptele d: x -2y +1 =0 si d: ax +3y =0 sunt
paralele ?
4. Dac AB =5, BC =12, CA =13, ct este cosA?
5. Dac Mn = {zC | z n =z}, cte elemente are M5 M7 ?
6. Cte elemente are mulimea { cos (n 2 )| nN} ?
Subiectul II
7. Cte numere de forma a0b (a,b cifre) exist ?
) ) )
)
8. Ct este suma 1 + 3 + 5 + ... + 15 n grupul ( 16,+) ?
9. Cte elemente de ordinul 4 are grupul ( 16,+) ?
10.

Cte funcii injective diferite f:{1,2} {1,2,3} exist ?

11.

Cte soluii are ecuaia x 3 = x n inelul ( 16,+) ?

Se consider funcia f: , f(x) =arctgx.


12.

Ct este lim f(x) ?

13.

Ct este f(x) ?

14.

Cum este funcia f pe intervalul (0, ), convex sau concav ?

15.

Cte soluii are ecuaia f(x) =2 ?

16.

Care este aria suprafeei plane cuprins ntre graficul funciei f, axa

ox i dreptele x=0 si x=1 ?

- 54 -

EXAMENE

SSM
H

Subiectul III
Se consider polinoamele f= 8X3-6X-1, g=4X3-3X i mulimea

M = h (cos ) h Q[ X ])
9

a) S se arate c f nu are rdcini raionale.


b) S se determine ctul i restul mpririi lui f la g.

c) S se arate c f (cos ) = 0 .
9
d) S se arate c cos

nu aparine mulimii Q.
9

e) Dac hQ[X] i h(cos20o)=0. S se arate c h M f.


f) S se arate c M este spaiu vectorial real n raport cu operaiile uzuale.
Subiectul IV

Se consider funcia f : (0, ) R, f ( x ) = tg ( x) i I n = tg 2n xdx, n N


2
0

a) S se arate c f(x)=tg2x+1.
b) S se calculeze I0 i I1.
c) S se arate c In+In-1=

1
, oricare nN*.
2n 1

d) S se arate c irul (In)n0 este convergent i s se calculeze lim I n .


n

1 1
1
e) S se calculeze lim (1 + ... + ( 1) 2 n1
).
n
3 5
2n 1

f) S se arate c nu exist polinoamele f, gR[X] astfel nct :

f ( n)
1 1
1
= 1 + ... + (1) 2 n1
,
g ( n)
3 5
2n 1

n N*

- 55 -

EXAMENE
TEST BACALAUREAT - 2006

M2

prof. Dan Daniel


NOT. Toate subiectele sunt obligatorii. Se acord 10 puncte din oficiu.
Timp de lucru efectiv 3 ore.
SUBIECTUL I ( 30p )
Pentru ntrebrile 1-16 scriei doar rspunsurile pe foaia de
examen
Se consider funcia f : R R , f ( x ) = x 2 5 x + 4
(3p) 1. Pe ce mulime este f strict pozitiv ?
(3p)
(3p)
(3p)
(3p)

2.
3.
4.
5.

Care este mulimea valorilor funciei pe intervalul [2,5] ?


Dac graficul are vrful pe dreapta y = 2x + a, ct este a ?
Care sunt soluiile ecuaiei 2 2 x 5 2 x + 4 = 0 ?
Pentru ce valori ale lui x avem f(x+1)f(x)?

1
.
x 4
(3p) 6. Care este domeniul de definiie al funciei ?

Se consider funcia f : D R , f ( x ) =

(3p) 7. Ct este

f (x )dx ?
0

(3p) 8. Pe ce mulime este funcia cresctoare?


(3p) 9. Care este numrul punctelor de extrem ale graficului funciei f ?
(3p) 10. Ct este lim n( f (1) + f (3) + ... + f (2n + 1)) ?
n

SUBIECTUL II ( 20p )
Se dau punctele A(2,-1), B(3,1), C(2,2) ntr-un sistem de coordonate n
plan;
(4p) 11.

Gsii dou puncte D i D care formeaz cu A, B, C paralelograme.

(2p) 12.

Care este distana de la origine la AB?

(4p) 13.

Aflai aria patrulaterului OABC?

(4p) 14.

Care este ecuaia paralelei prin A la BC?

(4p) 15.

n ce puncte taie dreapta BC, axele de coordonate?

(2p)

- 56 -

16.

Gsii punctul care mparte segmentul AB n raportul

1
.
4

EXAMENE

SSM
H

Pentru subiectele III-IV se cer rezolvrile complete


SUBIECTUL III ( 20p )
1 3
0 0
1 0
Se consider matricele A =
, O2 =
, I 2 =
i mulimea.
0 1
0 0
0 1
(2p) a) S se calculeze determinantul matricei A .
(2p) b) S se calculeze rangul matricei A .
a b
(4p) c) S se determine a, b C , astfel nct A
= I 2
0 a
(4p) d) S se verifice c A2 2A + I2 = O2 .
(4p) e) S se calculeze A2005.
(4p) f) S se arate orice matrice care comut cu A se poate scrie n forma
aA+bI2.

SUBIECTUL IV ( 20p )
Se consider funcia f : R R , f ( x ) = arctg ( x + 1) arctg ( x 1) ;
(4p) a) S se calculeze f(x).
(4p)
f ( x) f (0)
b) S se calculeze lim
.
x 0
x

(4p) c) S se arate c 0 < f(x) , x R .


2
d) S se arate c f este strict cresctoare pe intervalul (-, 0] i strict
(4p)
descresctoare pe intervalul [0, ).
2

arctg x 2 , x 0
.
(2p) e) S se arate c f ( x ) =

,
x=0
2
1
(2p)
f) S se calculeze f ( x) dx .
0

- 57 -

EXAMENE
Test pentru examenul de Testare Naional
Prof. Antonie Rodica
C.N. Traian - Drobeta Turnu-Severin
*Toate subiectele sunt obligatorii. Se acord 10 puncte din oficiu.
*Timpul efectiv de lucru este de 2 ore.

PARTEA I (45p) Pe foaia de examen se trec numai rezultatele.


1
3p 1.a)Rezultatul calcului 0,25-1 . 0,(3) este..........
2
3p
b)Media geometric a numerelor 2- 3 si 2+ 3 este..........
3p
c)n intervalul (2- 3 ;2+ 3 ) exist..........numere ntregi.
3p 2.a)Descompunerea n factori primi a numrului 108 este....
3p
b)Numrul 108 are .........divizori naturali.
3p
c)Cel mai mic multiplu comun al numerelor 108 i 90 este ........
3p 3.a)2,5% din 5000ha reprezint..........ha.
3p
b)Dac 4 muncitori termin o lucrare n 9 zile , atunci 6 muncitori ar
termina aceeai lucrare n.........zile.
3p
c)Fie mulimea A=( 3; 66 ) I N.Probabilitatea ca alegnd un numr
la ntmplare din mulimea A el s fie numr prim este.........
4. ABC are AB=13cm, AC=15cm i BC=14cm.
3p
a)Perimetrul triunghiului este egal cu...cm.
3p
b)Aria triunghiului este egal cu..........cm2.
3p
c)Lungimea nalimii din vrful A este egal cu..........cm.
5.n cubul [ABCDA1B1C1D1] aria AB1C este egal cu 8 3 cm2.
3p
a)Lungimea segmentului[AC] este egal cu..........cm.
3p
b)Lungimea muchiei cubului este egal cu..........cm.
3p
c)Volumul cubului este egal cu..........cm3.
PARTEA a II-a(45p)-Pe foaia de examen scriei rezolvrile complete.
1.Numerele naturale a,b,c sunt direct proporionale cu 5,13 i 2.
3p
a)Artai c bc-ac i bc+ac sunt ptrate perfecte.
3p
b)Ce procent din a este c ?
4p
c)Determinai numerele a, b i c tiind c a2+b2+c 2=198.
x2 4x + 3
2.Fie expresia E(x)= 2
,x R.
x 4x + 5
4p
a)Calculai E(2+ 3 ).
4p
b)Rezolvai n R ecuaia E(x)=0.
3p
c)Determinai valoarea minim a expresiei E(x), cnd x este real.
4p
d)Determinai valorile ntregi ale lui x pentru care E(x) este ntreg.
3.Fie [ABCA1B1C1] o prism triunghiular regulat cu feele laterale
patrate i aria total 18(6+ 3 )cm2. Calculati:
5p
a)lungimea muchiei bazei prismei.
5p
b)volumul prismei.
5p
c)distana de la punctul A1 la planul (ABC1) .
5p
d)sinusul unghiului format de dreapta BC1 cu planul (AA1C).

- 58 -

EXAMENE

SSM
H

Test de admitere n facultate


- algebr i analiz matematic Prof. Popescu Eleodor
1. Dac polinomul f = 2 x 4 + 5 x 3 17 x 2 + mx + n se divide cu polinomul
g = 2 x 2 x 6 , atunci m + n este egal cu:
a)4
b)8
c)10
d)13
e)17
f) n.u.d
2. Numrul soluiilor ecuaiei 3 x 4 = 4 in R6 este egal cu:
a)1
b)2
c)3
d)4
e)5
f) n.u.d
3. Fie legea de compoziie , ( x, y ) x y = x + y xy . Suma
elementelor simetrizabile n raport cu legea dat este egal cu:
a)-1
b)0
c)1
d)2
e)4
f) n.u.d
2
x
1
4. Se consider funciile f , g : [ 1,1] , f ( x) =
, g ( x) = 2
. Aria
2
x +1
suprafeei cuprins ntre graficele funciilor f i g i dreptele x = -1, x = 1
este egal cu:

1
5
2
a)
b)
c)
d)
e) 2 3
f) n.u.d
6
2 3
2
3

5. Derivata de ordin 101, pentru f ( x) = sin x , calculat n


este egal cu:
3
1
3
1
3
a)
b)
c)0
d)
e)
f) n.u.d
2
2
2
2
6. Calculnd

1 + x dx obinem un numr din intervalul:

a)( 0, 2)
b)( 2, 4 )
c)( 4, 6 )
d)( 6, 8 )
e)( 8, 10 ) f) n.u.d
7. Dac notm cu x1 , x 2 , x3 , x 4 rdcinile ecuaiei x( x + 1)( x + 2)( x + 3) = 24 ,
atunci suma x1 + x 2 + x3 + x 4 este egal cu:
a)10
b) 3 + 3 3
c)12
d) 5 + 4 6
e) 6 + 3 6
f) n.u.d
8. Fie g inelul claselor de resturi modulo 9. Numrul soluiilor ecuaiei:
7 x + 3 = 2 n Z g este:
a)0
b)1
c)2
d)3
e)4
f) n.u.d
9. Lungimea graficului funciei f ( x) = ln x, x 3 , 8 este egal cu:

a)

3
2

2 1
b) 1 + ln
3 2

1 3
c) 1 + ln
2 2

d) ln( 2 +

3
)
2

3
e) ln 1 + 2
2

f) n.u.d

10. Expresia 3 5 2 + 7 3 5 2 7 este egal cu un numr din intervalul:


a)(0, 1]
b)(1, 2]
c)(2, 3]
d)(3, 4]
e)(4, 5]
f) n.u.d
11.

Dac lim n n 2 + n + n 2 = 1 , atunci + + este egal cu:


n

a)-2
12.

b)-1
c)0
d)2
e)3
f) n.u.d
Dac dreapta y = 2x + 3 este asimptota spre + pentru funcia
x + x2 1
f ( x) =
, atunci:
mx + n
a)m+n=0
b)m=n
c) m 2 = n
d) m 2 + n = 0 e) mn > 0
f) n.u.d

- 59 -

EXAMENE
13. Ecuaia x 3 + x + m = 0, m , are rdcinile x1 , x 2 , x3 . Dac x15 + x 25 + x35 = 10 ,
atunci m este egal cu:
a)-1
b)1
c)2
d)4
e)5
f) n.u.d
14. Numrul natural n care satisface egalitatea
1
1
1
+
+ ... +
= 55 log 3 2 este:
log 2 3 log 4 3
log 2n 3
a)4
b)5
c)7
d)9
e)10
f) n.u.d
Axy = 7 Axy 1
15. Dac perechea ( x0 , y o ) este o soluie a sistemului y
, atunci
6C x = 5C xy +1
suma x 0 + y o este egal cu:
a)12
b)14
c)16
d)18
e)20
f) n.u.d

sin , dacax R
16. Funcia f m ( x) =
admite primitive pe R dac m ia o
x
m, dacax = 0

anumit valoare din intervalul:


1
1 2
2 3
3 4
4
a) 0,
b) ,
c) ,
d) ,
e) ,1
f) n.u.d
2
2 3
3 4
4 5
5
17. Ecuaia x + ln x 2 = m are trei rdcini reale pentru orice valoare a lui m
din intervalul:
a)(-2, -1)
b)(-1, 0)
c)(0, 1)
d)(1, 2)
e)(2,3)
f) n.u.d
x + y + z = 1
18. Numerele reale x, y, z care verific sistemul:
sunt din
2
2 xy z = 1
intervalul:
a)[-3, -2]
b)[-2, -1]
c)[-1, 1]
d)[1, 2]
e)[2, 3]
f) n.u.d
19. Pe R se definete legea de compoziie: x y = xy + x + y, x, y . Dac
legea este asociativ i comutativ atunci expresia x + + ia valoarea:
a)3
b)5
c)7
d)9
e)11
f) n.u.d
x1 x2 x3

20. Fie matricea x 2 x3 x1 , unde x1 , x 2 , x3 sunt rdcinile ecuaiei:


x x x
1
2
3
3
2
x 2 x + 2 x + 17 = 0 . Calculnd det (A 2 ) obinem:
a)4
b)8
c)9
d)12
e)16
f) n.u.d
Not: Prescurtarea n.u.d s-a folosit pentru expresia nici-unul dintre
rspunsurile de la a), b), c), d), e).

- 60 -

EXAMENE
Probleme pregtitoare pentru lucrarea scris
la clasa a V-a

SSM
H

prof. Dina Bojneagu Sc.Gen.6


prof. Anghel Bdlu Sc.Gen.6
1. Cte pagini are o carte dac pentru numerotarea paginilor ei s-au folosit
441 de cifre?
2. Este posibil s punem 21 de bile n 6 cutii, astfel nct n fiecare cutie s
fie cel puin o bil i s nu existe dou cutii cu acelai numr de bile?
Dar 325 de bile n 25 de cutii?
3. Demonstrai c numerele naturale care se mpart la 72 i dau ctul egal
cu restul, sunt divizibile cu 73.
4. Un numr natural de trei cifre, scris n baza 10, mprit la rsturnatul
su d ctul 2 i restul 100. Aflai numrul tiind c diferena dintre
cifra sutelor i cea a unitilor este 4.
5. Scriei toate numerele naturale scrise n baza 10, de forma xy care sunt
cu 45 mai mare ca yx .
6. Pentru premierea elevilor, la un concurs de matematic s-au cumprat
14 obiecte cu preul de 500 lei, 300 lei i respectiv 100 lei. tiind c s-a
cheltuit suma de 2000 lei, cte obiecte de fiecare fel s-au cumprat ?
7. Dintr-un scule scoatem pe rnd bile. Dac scoatem cte dou, mai
rmne n scule o bil. Dac scoatem cte trei sau cte patru rmne
n scule tot o bil. Care este numrul minim de bile aflate n scule?
8. Suma a 6 numere prime consecutive este numr prim. Aflai numerele.
9. Aflai 3 numere naturale a, b, c tiind c ab = 144, bc = 240, ac = 60.
10. Aflai numrul elevilor clasei a V-a tiind c dac i aezm n rnd
cte 2, 3, 4 sau 6 rmne un elev singur. Dac i aezm n rnd cte 7
rmne un rnd cu 2 elevi.
11. Suma a dou numere este 44, iar diferena lor este a cincia parte din
numrul mic. Care sunt numerele?
12. Intr-un co sunt mai puin de 100 pere. Dac scoatem cte trei pere,
n co rmne o par, dac scoatem cte patru pere, n co rmn dou
pere, dac scoatem cte cinci pere, n co rmn trei pere, iar dac
scoatem cte ase pere, n co rmn patru pere. Cte pere au fost n
co?
42
13. Fie xyz cel mai mare numr cu proprietatea 2
. S se afle
x + y2 + z2
suma cifrelor sale.
2 1
14. Aflai numerele naturale a si b care verific egalitatea
+ =1
a b
1
9
9
9
15. Sa se calculeze :
2 3 1997
10 10 10
10

- 61 -

PROBLEME PROPUSE
Clasa a V -a
1. S se determine mulimea numerelor de forma n= abc despre care se stie ca
a=b+5 iar c=b3+1.
2. Aflai restul mpririi numrului 42006(42007+2) la 50.

prof. Marica tefan

prof. Iulia i Victor Sceanu

3. Exist ptrate perfecte, formate din cifre distincte oricare dou, care au cifra
sutelor 3, a zecilor 5 i suma tuturor cifrelor egal cu 15?

prof. Iulia i Victor Sceanu

4.

Determinai cifrele x, y, z dac 0,2( xy ) 3 + 0,3( yz ) 4 + 0,4( zx) 5 = 2,810


prof. Iulia i Victor Sceanu

5. n anul 1990, un btrn fiind ntrebat ci ani are, a rspuns: ,,n anul 2006
voi avea un numr de ani egal cu suma ptratelor cifrelor anului n care mam nscut. Ci ani va avea n anul 2006?

prof. Iulia i Victor Sceanu

6. Se calculeaz suma primelor n numere naturale impare


a) S se arate c suma este ptrat perfect, oricare ar fi n;
b) S se afle n, dac suma este 12321;
c) S se afle cel mai mic numr n, mai mare ca 1000, pentru care suma
este un cub perfect

prof. Eleodor Popescu

7. S se determine toate numerele abcd de patru cifre, care sunt divizibile cu 11


i au suma tuturor cifrelor egal cu 11

prof Dan Nedeianu

8. Demonstrai c nu exist numere naturale n care verific egalitatea


1 2 ... 2005 n 2004 + 2005 10 n + 2n 2 = 2006 2006 .

prof. Gh. Calafeteanu

9. Artai c numrul N = 9 + 92 + 93 ++ 92010 se divide cu 5 numere naturale


impare consecutive.

prof. Gh. Calafeteanu

10.Comparai numerele p = 212223245 i q = 1 + 22 + 23 +...+ 21034

prof. Gh. Calafeteanu

11.La un concurs de matematic se acord 15 puncte din oficiu, se dau 3


puncte pentru fiecare rspuns corect i se ia un punct pentru fiecare
rspuns incorect. Cte rspunsuri corecte a dat un concurent dac a primit
40 puncte dup 15 ntrebri?
12.Fie mulimea M = { 2n.5n+1 + 1 | n N }
a) Artai c 5001 M
b) Artai c M nu conine nici - un ptrat perfect.

prof. Vrzaru Mariana

prof. Vrzaru Mariana

2004
13.Demonstrai c fracia
este reductibil. Determinai cifrele a
ab05 (a + b + 2)
i b astfel nct fracia s se poat simplifica prin 6.

prof. Gh. Calafeteanu

14.Aflai restul mpririi numrului n = 2004 + 1232005 la 2006.

prof. Gh. Calafeteanu

15.Aflai dou numere naturale dac au suma 161 i suma rsturnatelor lor
este 503.

prof. Gh. Calafeteanu

- 62 -

PROBLEME PROPUSE
16.Cte fracii subunitare de forma

2n + 26
exist ? Justificai rspunsul !
2006

SSM
H

prof. Gh. Calafeteanu

17.S se demonstreze c numrul


N = 11001+22002+33003+44004+55005+66006+77007+88008+99009 nu este ptrat perfect.

prof. Gh. Calafeteanu

18.Se d numrul A = 2006 + 12+123+1234++123.2005


a) Stabilii paritatea lui A,
b) Aflai ci divizori are al aptelea termen din sum,
c) Artai c A nu este ptrat perfect.

prof. Gh. Calafeteanu

Clasa a VI a
1. S se determine cifrele a<b<c<d cu proprietatea c numrul dcba abcd este
numr ale crui cifre sunt exact a,b,c,d ntr-o anumit ordine.

prof. Ciniceanu Gheorghe

2. S se determine numerele prime p, q, r tiind c: p|q+r , q|r+p , r|3p+q.

prof. Marica tefan

3. S se arate c numrul 100300 +202305 +304509 se divide cu 3.

prof. Ciniceanu Gheorghe

4. O echip de 10 muncitori trebuie s execute o lucrare n 20 de zile. Dupa 8


zile de la inceperea lucrrii 6 muncitori sunt mutai pe o perioad de 5 zile la
o alta lucrare i apoi revin la echip i lucreaz pn la terminarea lucrrii.
n cte zile s-a excutat de fapt lucrarea?

prof. Eleodor Popescu

5. S se determine restul mparirii lui N = a1


002
...4
0b + a0b (k N * fixat) la 11.
4
3
2 k cifre

prof Dan Nedeianu

2005 + 2 2005 + 3 2005 + ... + 2005


1003
x

6. Rezolvai n N ecuaia

x +1

= 2005
prof.Ovidiu Ticui

7. Rezolvai n N inecuaia
2005 + 2005 2 + 2005 4 + ... + 2005 2 2005
x
x
x

+
+ ... +
2006
1 2 2 3
2005 2006
2006(2
1)

prof.Ovidiu Ticui

Clasa a VII-a

1. n ABC cu m( A )=90o, notm cu O mijlocul lui BC, cu R, r respectiv raza


cercului circumscris i raza cercului nscris, iar cu R1, R2 respectiv razele
cercurilor circumscrise AOB i AOC. Artai c dac avem relaia:
1
1
4
, atunci ABC este isoscel.
+
=
R1 R2 R + r
prof. Iulia i Victor Sceanu

2. n ABC cu m( A )=90o i m( B )=60o fie D mijlocul lui BC, DM BC, DN AD,


unde M, N (AC). Artai c cercurile circumscrise lui ABDM, ADN i CDM
sunt egale.

prof. Iulia i Victor Sceanu

- 63 -

PROBLEME PROPUSE
3. n ABC considerm bisectoarele (AD, (BE, (CF concurente n I. Artai c:
IA IB IC

8.
ID IE IF

prof. Iulia i Victor Sceanu

4. S se arate c dac ntr-un triunghi lungimile medianelor sunt proporionale


cu numerele 5, 3 3 , i 7, atunci lungimile laturilor sunt proporionale cu
numerele 55 , 11 i 127 .
prof. Eleodor Popescu

5. Pe latura [DC ] a paralelogramului ABCD se consider un punct E i fie F


punctul comun dreptelor BE i AD. Dac AD 2 = AC DF demonstrai c
dreapta AE este bisectoarea unghiului CAD.

prof. Dan Nedeianu

6. Fie

a = ( 2 1) 2005 ( 3 2 ) 2005 ... ( 2005 2004 ) 2005 i


b = ( 2 + 1) 2005 ( 3 + 2 ) 2005 ... ( 2005 + 2004 ) 2005 .
S se arate c a + b 2

prof.Ovidiu Ticui

7. Fie m = 2 + 4 + 6 + ... + 4008 + 2005 .


S se rezolve inecuaia

x 2 2 x 2005 + 2005 + 2005 m


prof.Ovidiu Ticui

Clasa a VIII-a
1. Rezolvai n N ecuaia xyz + xy + yz + xz + x + y + z + 1 = 6

prof. Iulia i Victor Sceanu

2. Dac n N, artai c

4 12 24
2n 2 + 2n (n + 1) 2
+ +
+ ... +

3 5
7
2n + 1
2

prof. Iulia i Victor Sceanu

3. Dac n N* rezolvai n R ecuaia cu necunoscuta x


n
n +1
n+2
= n3+3n2+2n.
[3(n3+3n2+2n)x+3n2+6n+2]
+
+
nx
+
1
(
n
+
1
)
x
+
1
(
n
+
2
)
x
+
1

prof. Iulia i Victor Sceanu

4. S se arate c triunghiurile ale cror laturi a, b, c ndeplinesc condiia:

a 2 2a 3n + n 2 + 7 n + 4 + b 2 2b 2n + n 2 + 4n + 1 + c 2 2c n + n 2 + n 3(n+1)
au unghiurile de msur constant, oricare ar fi n N*.

prof. Iulia i Victor Sceanu

5. Fie numerele reale x1, x2,, xn>1, astfel nct


S se arate c

x1 + x2 + ... + x n

x1
x
x
+ 2 ++ n = n+1.
x1 1 x 2 1
xn 1

x1
x2
xn
+
++
.
x1 1
x2 1
xn 1
prof. Iulia i Victor Sceanu

6. Dac numerele reale a i b satisfac relaiile -1 < a < 0 i 0 < b < 1, atunci s
a 2002 + b 2000
a 2002 + b 2001
se afle care dintre numerele = 2000
i

=
este mai mare.
a
+ b 2001
a 2000 + b 2002

prof. Eleodor Popescu

- 64 -

PROBLEME PROPUSE
3 x 2 + y 3 = 27

7. Aflai parametrul real a, dac sistemul


are soluie unic.
x y + 2006 = a

SSM
H

prof. Dan Nedeianu

Clasa a-IX-a
1. Fie ABC un triunghi oarecare cu lungimile laturilor a, b i c. Fie C(O,r) un
cerc de centru mobil i raz constant care ntlnete laturile triunghiului n
punctele A1, A2, A3, A4, A5 i A6 unde A1, A2(BC), A3, A4(AC), A5, A6(AB).
Notm cu = m(<A1OA2), =m(<A3OA4) i =m(<A5OA6).
a) Artai c acos /2 +bcos /2 +ccos /2 este constant.
b) Daca [BA1][A2C] i [CA3][A4A], demonstrai c centrul cercului
circumscris triunghiului ABC coincide cu punctul O.
prof. Gabriela & Lucian Bondoc

2. S se arate c

sin 15
cos15

=2
0
sin 5
cos 50

a)

b)

1
1

=2 2
0
sin 15
cos15 0
prof. Buzatu Otilia

tg 45 0 x =

3. S se arate c

cos x sin x
1
=
tg 2 x
cos x + sin x cos 2 x
prof. Buzatu Otilia

4. S se arate c expresia

sin 2a x sin a
. nu depinde de x.
1 + cos 2a x cos a
prof. Buzatu Otilia

sin 7 x sin 4 x
.
sin 2 5 x sin 2 6 x
2

5. S se simplifice fracia

prof. Buzatu Otilia

6. S se arate c

x R+* avem

1+ x + x
3

3
2
x+x
2

prof. Buzatu Otilia

7. S se arate c dac x0 atunci ( x 2) 16 x > 0 .


4

prof. Buzatu Otilia

8. S se arate c x 4 x +

1
0.
2
prof. Buzatu Otilia

9. S se verifice identitatea

sin( 2a + b) sin( 2b + a)
sin 2 a + sin 2 b
+
4 cos(a + b) =
sin a
sin b
sin a sin b
prof. Buzatu Otilia

10.S se calculeze tg

tiind c sin + cos = i c (0, ) .


2
5
4
prof. Buzatu Otilia

- 65 -

PROBLEME PROPUSE
11.S se rezolve ecuaia :

x + 6 1 x 2 = 1 dac x 1 .
prof. Buzatu Otilia

12.Sa se rezolve ecuatia

x 4 x + x + 6x + 2 = 0
4

prof. Buzatu Otilia

(2 + m)

13.Dac 0 < m < 1, s se rezolve ecuaia

x 3

+ ( 2 m)

x 2 5 x + 6

=2

prof. Buzatu Otilia

14.ntr-un patrulater convex lungimile laturilor sunt numere subunitare iar


lungimile diagonalelor sunt notate cu e si f. Sa se arate ca:
i)
e + f > ef
ii)
e2 + f 2 > e2 f 2
prof. Eleodor Popescu

15.Fie a, b, c, d numere reale pozitive, fixate. S se determine minimul funciei

a
b
c
d
f : (0, ) R , f ( x) =
+
+ 2 +
2
2
2
sin x cos x tg x ctg 2 x
prof. Dan Nedeianu

[ x ] = [ y ]
16.S se rezolve in R sistemul
[ y ] = [x ]

prof. Daniel Sitaru

17.S se demonstreze c dac ntr-un triunghi, lungimile bisectoarelor interioare


sunt proporionale cu lungimile laturilor corespunztoare , atunci triunghiul
este echilateral.
prof. Daniel Sitaru

18.S se rezolve n numere naturale distincte ecuaia x2 + y2 + z2 = 495


prof. Ungureanu Octavian Mihai

Clasa a-X-a
1. Sa se rezolve sistemul de ecuatii

x 2 + y 2 + z 2 = 29

x + y + z = 9
xy = 12

prof. Buzatu Otilia

2. S se demonstreze c

1
1
1
log 5 6 + log 6 7 + log 7 5 > 1
2
3
4
prof. Eleodor Popescu

3. Fie r R, r>1, x, y, z C astfel ca x = r 2 1 , y = 2r , z = r 2 + 1 i x + y + z = 0.


Demonstrai c exist a, b R a.i. ax 2 +by 2 = 0
prof. Dan Nedeianu
(Generalizare O.L.M. Bucuresti 1996)

- 66 -

PROBLEME PROPUSE

SSM
H

4. Fie z1 , z 2 C , astfel nct z1 , z 2 , z1 + z 2 , z1 - z 2 C \ R .


a) Considerm f : R R, f ( x ) = z1 x z 2 x . Aflai valoarea maxim a lui f.
b) Aflai x R pentru care f atinge valoarea maxim .

prof. Giugiuc Constantin

5. Considerm n planul complex ABC neechilateral cu A( z1 ), B( z 2 ), C ( z 3 ) astfel


nct 2 z 2 z 3 = z1 z 2 + z1 z 3 . Artai c patrulaterul determinat de
punctele B, C, G, I este trapez, unde G este centrul de greutate iar I centrul
cercului nscris ale ABC.
prof. Giugiuc Leonard

6. Fie A o mulime infinit iar f : A A o funcie injectiv i nesurjectiv.


Artai c () g : A A astfel nct g o f = 1A .
prof. Giugiuc Leonard

7. Fie A o mulime infinit iar f : A A o funcie surjectiv i neinjectiv.


Artai c () g : A A astfel nct g o f = 1A .

prof. Diana Trilescu

8. Fie A = {z C z < 1} ,iar f : C A, f ( z ) =

z
. Artai c f este bijecie.
1+ z
prof. Diana Trilescu

9. Fie x, y, z unghiurile formate de diagonala unui paralelipiped dreptunghic cu


trei muchii diferite care pleac din acelai vrf. S se arate c
(sin x + sin y + sin z + 1) 2 4(sin 2 x + sin 2 y + sin 2 z )
prof. Daniel Sitaru

10.S se arate c:
55 ln( e sin 1.01) ln( e sin 1.02)... ln( e sin 1.55) ln( e 5 sin 1.01sin 1.02... sin 1.55 )
prof. Daniel Sitaru

11.Se consider cercurile de ecuaii:


z z1 = r1 , z z 2 = r2 , z1 = m + in, z 2 = p + iq , m, n, p , q , r1 , r2 R . S se arate c
cercurile sunt ortogonale dac i numai dac

r1 + m p
r +qn
= 2
r2 + n q r1 + m p
prof. Daniel Sitaru

Clasa a-XI-a
1. Fie functia u:R R pentru care u( x +

1
1
) = u( y + ) x, y R * si u (0) = 0.
y
x

Artai c :
a) u este o funcie par.
b) Gsii funciile continue cu aceast proprietate .
prof. Gabriela & Lucian Bondoc

2. Fie A M 2 ( R ). Artai c dac A2 =0 atunci det(I-A)= 2 , R .

prof. Buzatu Otilia

3. S se calculeze

x +1 + x 1
.
x
3

lim
x 0

prof. Buzatu Otilia

- 67 -

PROBLEME PROPUSE
4. S se afle cel mai mare termen al irului x n =

n2
.
n 3 + 200
prof. Buzatu Otilia

1
lim (cos ) x
x
x

5. S se calculeze

prof. Buzatu Otilia

6. Fie a, b R i irul xn definit prin x n+1 = b xn , x1 = a . S se afle limita irului.


3

prof. Buzatu Otilia

(n + 1) .
n [(n + 1)!]2
n

7. Sa se calculeze lim

prof. Eleodor Popescu

8. Fie k>0 fixat i matricea

A=y
z

y
z
x

x M 3 ( R) . S se afle max( det A ) , cnd


y

x, y, z verific relaia x 2 + y 2 + z 2 = k.
prof. Dan Nedeianu

9. Fie f : [a, b] R, Im f R \ Q . Cercetai dac funcia este continu.

prof. Gimoiu Iuliana

a b c
a'

10.Fie A, B M 3 (C ) , unde: A = c a b , iar B = c'


b c a
b'

a1 b1

a) Artai c ()a1 , b1 , c1 C astfel nct A B = c1 a1


b c
1
1
a1 + b1 + c1 = (a + b + c ) (a '+b'+c')

2
2
2
, unde
a1 + b1 z + c1 z = a + bz + cz a '+b ' z + c' z

2
2
2
a1 + b1 z + c1 z = a + bz + cz a '+b ' z + c' z

(
(

)(
)(

)
)

b' c '

a ' b' .
c' a '
c1

b1 i au loc relaiile:
a1

z C i 1 + z + z 2 = 0 .

b) Calculai A n , n N * .
prof. Giugiuc Leonard

11.Fie A = (a ij )1i , j n , B = (bij )1i , j n , C = (c ij )1 i, j n aij = max(i, j ), bij = min(i, j ),


,
i j, i j
cij =
S se arate c det[( AB C )( BC A)(CA B )] 0
2i 1, i = j

prof. Daniel Sitaru

12.

Fie a R , f : [0, a] R , o funcie continu astfel nct f(0)=f(a). S se


*
+

arate c exist b [ 0 , 2 ] astfel nct

- 68 -

f (b ) = f (

a + 2b
)
2

prof. Daniel Sitaru

PROBLEME PROPUSE
l og

13.Fie a,b,c (1; ). Calculati lim

x 0

l og

log

log

log

log

cb
cos x
a b
cos x

SSM
H

prof. Daniel Sitaru

Clasa a-XII-a
1. Fie f : R R , f(x) =

x 4 + x 2 + 2 i F primitiv a lui f care se anuleaz n 0.

S se demonstreze c
1

2. S se calculeze

x x2 +1
+ ln( x+
2

F(x) >

x2 +1) ,

x>0.

prof. Ciniceanu Gheorghe

(1 + e )(1 + x )dx
x

prof. Buzatu Otilia


6

3. S se arate c

16
x

dx 9
3 4 x+2
prof. Buzatu Otilia

4. Sa se caluculeze integrala

3 cos 2 x + 4 sin 2 x + 5 cos x + 6



dx , x o, .
4 cos x + 2 sin x + 1
2

prof. Eleodor Popescu


a

5. S se arate c exist a>0 astfel ca

x + 2x + 2
2

x + 2 x + 2 + x 4x + 5
2

dx =

1
2

prof Dan Nedeianu


2

6. S se calculeze

max(sin x; arcsin(sin x))dx


0

prof. Daniel Sitaru

7. S se calculeze

sin 2005 x + cos 2 x


0 sin 2005 x + cos 2005 x + 1dx
prof. Daniel Sitaru

8. Se consider familia de drepte d a : (3 a) x + (2 + 7a) y + 5a 1 = 0, a R .


a) S se calculeze

Id

aR

b) Fie M = {d a | a R} . S se defineasc pe M o lege de compoziie


intern , astfel nct grupurile abeliene (M, ) i (R,+) s fie
izomorfe.

prof. Daniel Sitaru

- 69 -

Petre Sergescu
PETRE C. SERGESCU
- un matematician severinean de renume mondial prof. Gheorghe Calafeteanu
coala Constantin Negreanu
S-a nscut la 5/17 decembrie 1893 n Drobeta Turnu Severin, ntr-o
familie de intelectuali. Dup mam, Sergescu este strnepot al preotului
revoluionar Radu Sapca ( Popa Sapca).
La Severin urmeaz clasele primare i cursurile liceului Traian,
avnd ca profesori de matematici, crora le-a pstrat frumoase amintiri, pe
C.I.Coculescu, Perte Marinescu i N. Muzicescu. Deoarece n clasa I
(primar) i pierde vederea unui ochi, ntr-un joc de copii, cu bulgri de
zpad, toat viaa va lucra folosind lupa. Rezultatela excepionale la
nvtur i obinerea de premii la concursurile Gazetei Matematice au
fcut s fie remarcat de ctre Gheorghe ieica i ali profesori ai
Universitii bucuretene. A absolvit att secia real, ct i cea modern a
liceului i a susinut un dublu bacalaureat n tiine i litere, dup care a
urmat facultatea de matematici din Bucureti (1912-1916), unde a avut
profesori mari de matematici ca : Gheorghe ieica, Dimitrie Pompeiu, Al.
Davidoglu, Traian Lalescu, Nicolae Coculescu i David Emanuel. n paralel
cu facultatea de matematic a urmat cursurile facultii de filozofie i
Conservator de Muzic.
n timpul studeniei, ca preedinte al Centrului studenesc Bucureti,
a fost unul dintre animatorii studenimii pentru lupta n vederea realizrii
unitii politice a rii, organiznd manifestaii i congrese studeneti.
n 1917 a fost arestat n timpul unei ore de curs la Liceul Traian din
Turnu Severin, unde i ncepuse cariera, de ctre jandarmii germani pentru
c i manifestase public patriotismul. A fost inut ca ostatic politic timp de
18 luni ntr-un lagr bulgar ( Golemo Konare), mpreun cu fotii si
profesori de liceu : Gh. Oprescu, Paulian, Vrcol i Rzmeri. Cu toate c
avea tatl pe patul de moarte, nu i s-a permis s-l vad dect cu condiia s
semneze o declaraie, spunnd c pentru actele sale era pltit de
propaganda rus. Cu tot riscul i sacrificiul, Petre Sergescu a refuzat s
semneze o astfel de declaraie mincinoas.
n 1919 Sergescu obine o burs i pleac la Paris pentru a studia, n
continuare, matematicile. n afara cursurilor de la Sorbona a urmrit, n
1992, toate leciile cursului de filozofie matematicii predate la College de
France de ctre P. Boutroux.
ntors n ar cu nc o licen n matematici, trece i examenul de
capacitate pentru nvmntul secundar, reuind ntiul, cu media 10.
La 28 noiembrie 1923 obine doctoratul n matematici cu o tez avnd
subiectul Asupra nucleelor simetrizabile, n faa unei comisii formate din
David Emanuel, ieica i Lalescu, iar n 1924 este numit profesor suplinitor
la Politehnica din Bucureti.
n 1924 este numit profesor agregat de geometrie analitic la
Universitatea din Cluj , unde rmne pn n 1943 ( ca i profesor titular).
Din noiembrie 1943 este numit profesor de geometrie analitic la
coala Politehnic din Bucureti, n 1945 este ales rector al acestei
instituii, iar n 1946 este ales ca preedinte al Academiei Internaionale de

- 70 -

Petre Sergescu
Istorie a tiinei. n 1947 este ales secretar pe via al acestui for
internaional i secretar general al Uniunii Internaionale de Istorie a
tiinei, iar din 1952 ndeplinete i funcia de cercettor la Centrul Naional
de Cercetare tiinific din Paris i director al Arhivelor Internaionale de
Istorie a tiinei.
n vara anului 1953, n timp ce se ntorcea cu vaporul n Frana de la
cel de-al VIIlea Congres Internaional de Istoria tiinelor, inut la
Ierusalim, a suferit, pe Mediteran un oc cerebral, apoi, n decembrie,o
hemiplegie, care se amelioreaz n urma tratamentului, dar rmne
suferind. n seara zile de 20 decembrie 1954 i propune s ncheie a doua zi
un articol despre viaa i opera lui Dimitrie Pompeiu (care murise la 7
octombrie 1954), dar pentru el nu a mai fost a doua zi, s-a stins n somn, n
noaptea de 20 spre 21 decembrie 1954, la Paris.
Academicianul matematician Paul Montel a terminat articolul despre
Pompeiu i l-a publicat n revista L enseignement mathematiques din
Geneva, alturi de necrologul lui Sergescu.
Elogiile aduse savantului Petre Sergescu de ctre marii matematicieni
ai lumii au aprut n revistele tiinifice franceze,elveiene, germane, engleze
i americane, unde alturi de Paul Montel s-au nscris ilutrii F. S.
Bodenheimer, Georges Bouligant, Sophie Picard, Waclaw Sierpinski, Rene
Taton, etc, iar cunoscutul istoric al tiinei George Sarton i-a dedicat ultima
sa carte aprut n 1957.
Petre Sergescu a fost ngropat n cimitirul oamenilor celebrii,
Montmorency, n marginea de nord a Parisului unde o piatr funerar
amintete numele su, data i locul naterii i al morii i una din frazele
sale preferate, fraz ce conine ntreaga sa pasiune, modestie i druire
Jai ce que jai donne ( Am ceea ce am dat).
n Temoignage marele Sierpinski spunea c l-a cunoscut pe
Sergescu n 1927 la Congresul matematicianilor polonezi, la Lwow, iar dup
aceea s-a nscut ntre ei o simpatie reciproc care s-a transformnat ntr-o
prietenie pentru toat viaa. Sierpinski a fost prezent la primele dou
congrese ale matematicienilor romni la Cluj i Turnu Severin i
mrturisete c a vzut cu proprii si ochi ct munc, trud i energie a
depus Petre Sergescu pentru ca aceste congrese s fie reuite .
Dintr-o scrisoare a doamnei Maria Kasterska Sergescu reiese c
universitile americane, cnd trimiteau matematicieni tineri pentru
specializare la Paris, le recomandau s treac , n primul rnd, pe la
Sergescu, spre a-i cere sfaturi i ndrumri asupra cursurilor ce trebuia
urmrite.
Petre Sergescu a contribuit foarte mult la stabilirea relaiilor de
prietenie i de cunoatere reciproc dintre matematicieni strini i romni.
Astfel, datorit eforturilor sale au participat la Congresele noastre naionale
matematicieni celebri, ca Arnaud Denjoy, Paul Montel, Vito Volterra, W.
Serpinski, Destouches, A. Reymond, Aldo Mieli, Mauro Picone, B. Hostinsky,
etc. De asemenea, muli matematicieni strini au colaborat la revista
Mathematica din Cluj, pentru care Sergescu a fcut deosebite sacrificii
materiale.
Paul Montel spunea despre el : Viaa lui Petre Sergescu ne ofer
imaginea unor nalte caliti tiinifice mpletite cu mari virtui morale (....) A

SSM
H

- 71 -

Petre Sergescu
dus o via ordonat i modest, care rmne pentru noi un exemplu i un
model .
Amintiri frumoase avem i de la academicianul Octav Onicescu, care
relateaz : ... Acele ntlniri ale noastre erau mblsmate de atmosfera de
puritate intelectual i moral care au fcut farmecul lui Petre Sergescu dea lungul ntregii sale vieii (...) matematicianul filozof , generos n toate
manifestrile sale, care era prietenul meu Petre Sergescu , ... acest om ales,
druit ntreg pentru problemele rii i ale tiinelor noastre .

Prima ediie a Concursului interjudeean


Petre Sergescu-Drobeta Tr. Severin
21-22 ianuarie 2005
n cadrul unei excelente colaborri ntre Colegiul Naional Traian i
Filiala Mehedini a Societii de tiine Matematice din Romnia, n memoria
marelui matematician Petre Sergescu s-a desfurat n zilele de 21-22
ianuarie 2005 prima ediie a concursului interjudeean de matematic
purtndu-i numele.
La concurs au participat peste 400 de elevi iubitori de matematic din
Mehedini, Cara Severin, Dolj.
Concursul a fost nsoit de o sesiune de referate i note matematice
susinute de domnii profesori, dupa programul de mai jos:
[1]
[2]
[3]
[4]
[5]
[6]
[7]
[8]
[9]
[ 10 ]
[ 11 ]
[ 12 ]
[ 13 ]

- 72 -

Despre contributii oltene la dezvoltarea stiintei


Prof.pensionar MARICA STEFAN Drobeta Tr.Severin
Numarabilitatea discontinuitatilor unor clase de functii reale
Prof.dr.Gheorghe Cainiceanu CNT
Teorema Grassman si aplicatii
Prof.drd.PRAJEA MANUELA CNT
Teorema lui Van Aubel si aplicatii la rezolvarea problemelor de geometrie
Prof.ANTONIE RODICA CNT - Prof.PAPONIU DANA CNT
Petre Sergescu un mare matematician severinean de renume mondial
Prof.VARZARU MARIANA Lic.Auto Drobeta Tr.Severin
Prof.CALAFETEANU GH.Sc.Gen.9 Drobeta Tr.Severin
Proportia divina
Prof.MARIS MARINELA Sc.Gen.15 - Prof.TARALESCA ELENA Sc.Gen.15
Relatii metrice de tip Menelaus
Prof.CALAFETEANU GH. Sc.Gen.9 - Prof.VARZARU MARIANA Lc.Auto
Petre Sergescu si invatamantul mehedintean
Prof.FLORESCU VIOLETA Sc.Gen.Petre Sergescu
Prof.MALINEANU GABRIELA Sc.Gen.Petre Sergescu Drobeta Tr.Severin
Metode de rezolvare efectiva a ecuatiilor diferentiale
Prof.MOGONEA NATALIA LAVINIA Sc.Cazanesti
Petre Sergescu elevul inscris pe tabla de onoare a Colegiului Traian
Prof.CAMENITA PIPINA - Prof.PAUNESCU EUGEN
Prof.POPESCU ILEANA Sc.Gen.3 Drobeta Tr.Severin
Probleme elementare din operele unor personalitati
Prof.HINOVEANU SORIN Sc.Gen.6 Drobeta Tr.Severin
Morfisme de grupuri
Prof.MARASCU CORNEL Sc.Gen.6
Matematica si poezia

Petre Sergescu
[ 14 ]
[ 15 ]
[ 16 ]
[ 17 ]
[ 18 ]
[ 19 ]
[ 20 ]
[ 21 ]
[ 22 ]
[ 23 ]
[ 24 ]
[ 25 ]
[ 26 ]
[ 27 ]

Prof.GAGEA GHEORGHITA Sc.Gen.6


Prietenul meu,Fibonacci
Prof.BOJNEANGU DINA Sc.Gen.6
Siruri recurente de ordinul doi
Prof.BADALUTA ANGHEL Sc.Gen.6
O proprietate derivata de asemanare
Prof.CIOCAN ION - Prof.VASILUTA GHEORGHE Sc.Gen.6
Constructii cu rigla si compasul
Prof.VASILCANU FLORENTINA Lic.D-l Tudor
Prof.VASILCANU OCTAVIAN Sc.Gen.6
Corpuri geometrice rotunde-probleme deosebite
Prof.GABRIELA FARAGO - Prof.GHERGHINA RAESCU
Prof.ALEXANDRU FARAGO Lic.Traian Lalescu Orsova
Petre Sergescu o mare personalitate severineana
Prof. CIUPAGEA MARIA Sc.Gen.Petre Sergescu
Cum au aparut radicalii
Prof.PATRUTESCU SORIN Lic.I.St. Paulian Drobeta Tr.Severin
O generalizare a inegalitatii C.S.B
Prof.GIUGIUC LEONARD MIHAI Lic. D-l Tudor
Asimptotele primitivelor functiilor periodice continue si necontinue
Prof.NEDEIANU DAN - Prof.GIUGIUC LEONARD Lic.D-l Tudor
Cardioida
Prof.GIUGIUC CONSTANTIN C.N.T
Revista de matematica a elevilor un proiect educational in scoala actuala
Prof.MIRULESCU MARITA
Prof.HUMITA DORINA Liceul Pedagofic C.D.Loga Caransebes
Teorema lui LAPLACE
Prof.BIZDOACA CLAUDIA Fundatia Gh. Titeica - Prof. NANUTI DAN CNT
Clase de functii reale
Prof.PUPAZA ECATERINA Liceul Decebal Drobeta Tr.Severin
Teorema impartirii cu rest (Inele de polinoame)
Prof.GRIGORAS ANISOARA Sc.Gen.14 - Prof.MANDRES LUMINITA Sc.Gen.6

SSM
H

Iata care au fost rezultatele concursului (premii si mentiuni) :


Clasa V-a
Numele i prenumele
Asproniu Robert
Curelea Raul
Fril Remus
Clin Lucian
Constantinescu Robert
into Mdlina
Dragotescu Teodor
Guran Volintiru Maria
Arbna Gigel
Grecu Andra
Nicolicioiu Andrei
Srbu Adrian
Boescu Anca
Costea Maria
Prunescu Flavius
Stnic Alida
Pileag Anemona
enea Andrei
Codu Silviu
Grosu Vlad
Olaru Mihaela

Scoala
CNT
ieica
c2
Craiova
c2
CNT
c14
c2
ieica
ieica
ieica
ieica
CNT
CNT
CNT
c6
CNT
Craiova
c14
CNT
CNT

Pct.
21
21
19
18
17
17
16
16
14
14
14
14
13
13
13
13
12
12
11
11
11

Numele i prenumele
Cpitnescu Mdlina
Toader Simona
Clein Andrei
Diaconeasa Lucian
Drguin Stelian
Pavel Cristian
Ptruescu Alina
Roca Ioana
Vasiu Mihnea
Bnu Albert
Beliu Ruxandra
Cicic Carla
Ciocoteal Alexandru
Filipescu Alexandru
Limbosu Maria
Lungu Amelin
Popescu Florentina
Preoteasa Ana Carina
Tufaru Danusia
Vian Luana
Zugravu Rozalia

Scoala
CNT
c2
CNT
c2
c5
CNT
ieica
CNT
CNT
Craiova
CNT
ieica
CNT
c14
CNT
CNT
c1
ieica
c6
CNT
CNT

Pct.
10
10
9
9
9
9
9
9
9
8
8
8
8
8
8
8
8
8
8
8
8

- 73 -

Petre Sergescu
Clasa a VI-a
Numele i prenumele
Mitroi Bianca
Agape Mihai
Zorocliu Andra
Teil Bianca
Schneider Valeriu
Cepe Cristian
Milici Alina Simona
Manta Hanelore
Popescu Eduard
Furcu Ioana
Drpe Cristian
Pan Cristina
Drghia Miruna
Radu Cornel
Caplea Luminia
Minescu Mdlina
Budu Anca

coala
Craiova
CNT
CNT
ieica
Craiova
Decebal
c5
ieica
CNT
CNT
ieica
CNT
CNT
ieica
CNT
CNT
CNT

Pct.
18
17
13
12
12
12
11
11
10
10
10
8
8
7
7
6
6

Numele i prenumele
Conea Sorina
Cheredi Ioan
Arcu Anamaria
Sitaru Bogdan
Poenaru Alexandra
Drcea Vrbete Elvis
Andreca Mihai
uculanu Andreea
Stnioar Sarah
Popa Bogdan
Gugulici Larisa
Gantolea Adina
Fsui Vali Ionela
Cioclei Ciprian
Botoanu Cristi
Alexandrescu Iulia

coala
c6
ieica
c14
c6
c14

coala
Craiova
Craiova

Numele i prenumele
Croitoru Rzvan
Nef Nicoleta
Huza Alexandra
Megan Livia
Milu Lucian
Crng Alisa
Ecobici Gabriela
Moatr Alexandra
Nicoar Clin
Rdu Marian

coala
CNT
c2
CNT

CNT
Craiova
ieica
CNT
CNT
CNT

Pct.
21
21
19
17
14
11
11
10
10
8

CNT
c2

Pct.
8
8
7
7
7
5
5
5
5
5

coala
Craiova
Craiova
Craiova
CNT
CNT
CNT
Craiova
CNT

Pct.
19
14
9
5
5
5
4
3

Numele i prenumele
Mercioni Marina
Popescu Alina
Rducu Alexandru
Rou Maria
Boblc Oana
Creu Adina
Nica Flavius
Trocan Irina

coala
Orova
CNT
CNT
CNT
CNT
Odobleja
CNT
CNT

Pct.
3
3
3
3
2
2
2
2

coala
Reia
CNT
Craiova
CNT
CNT
CNT
CNT
CNT
Craiova
CNT
CNT
Odobleja

Pct.
18
15
15
14
10
9
9
9
8
6
5
5

Numele i prenumele
Puia Constantin
Racheru Adrian
Sandu Oana
unea George
Mariescu Radu
Stoica Raluca
Axinte Livia
Creu Andrei
Ghinea Constantin
Hinoveanu Ctlin
Nistor Alexandru
Pandioniu Georgiana

coala
Craiova
CNT
CNT
CNT
CNT
CNT
CNT
CNT
Dl.Tudor
CNT
CNT
CNT

Pct.
4
4
4
4
3
3
2
2
2
2
2
2

Odobleja

CNT
CNT
CNT
c14
c2
ieica
c14
CNT
ieica
c14

Pct.
5
5
5
4
4
4
4
3
3
3
3
3
3
3
3
3

clasa a VII-a
Numele i prenumele
Danciu Diana
Pdureanu Victor
Milcu Roxana
Vlad Adina
Pi Rada Andrei
Alexandru Bogdan
Marin Andreca Cristina
Clinovici Paul
eitan Mihaela
Carapencea Constantin

Caransebe
Caransebe

Caransebe

Craiova
Odobleja
CNT
Caransebe

Clasa a VIII-a
Numele i prenumele
Prodescu Corneliu
Matican Bogdan
Ivanov Bogdan
Nicolescu Alexandra
Oprea Radu
Rogobete Roxana
Pufu Cristian
Marinescu Iris

Clasa a IX-a
Numele i prenumele
Popovici Doru
Coand Oana
Goea Ioan Victor
Tigora Andrei
uselea Robert
Botea Silvia
Ciniceanu Andrei
Prundeanu Andreea
Aurariu Rzvan
Rveanu Ioana
Gaml Andreea
Popescu Anca

- 74 -

Petre Sergescu
Tu Leontin
Gomoi Mihai
Hane Anca

CNT
CNT
CNT

5
4
4

Punescu Adrian
Trac Mihai

CNT
CNT

2
2

SSM
H

Clasa a X-a
Numele i prenumele
Dinu Leonard
Bobii Ruxandra
Cucu Silviu
Rapcea Mihai
Boce Bogdan
Dgdi Monica
Rou tefan

coala
Craiova
ieica
Reia
CNT
CNT
CNT
CNT

Pct.
17
15
13
12
11
10
10

Numele i prenumele
Bica Denisa
Turturea Roxana
Viau Mihai
Bun Mdlina
Pleniceanu Bogdan
Sptariu Rzvan
Murean Viorel

coala
CNT
CNT
Odobleja
Reia
CNT
CNT
Caransebe

Pct.
9
8
8
7
6
5
5

coala
CNT
ieica
CNT
CNT
Reia
CNT
CNT
CNT

Pct.
21
9
8
7
5
5
5
5

Numele i prenumele
Chilom George
Blteanu Valeriu
Bolocan Anca
Buoniu Paula
Grindeanu Andreea
Jianu Amalia
Mateiescu Alexandru
Nicoar erban

coala
CNT
CNT
CNT
Orova
CNT
CNT
CNT
CNT

Pct.
4
3
3
3
3
3
3
3

coala
Craiova
Craiova
Craiova
CNT
Craiova
CNT

Pct.
20
18
16
16
15
12

Numele i prenumele
Brlan Lucian
Irimia Florin
Mazilu Mircea
Nicolicea Alin
Stanomir Oana
Vasilovici Andreea

coala
CNT
CNT
CNT
CNT
CNT
CNT

Pct.
11
10
9
9
9
9

Clasa a XI-a
Numele i prenumele
Ungureanu Andrei
Bzvan Eduard
Bcrin Dan
Mirea Mihai
Chi Andrei
Gdea Marius
Golenteanu Magda
Luca Alexandru

Clasa a XII-a
Numele i prenumele
Diaconu Jean
Clin Georgel
Munteanu Alin
naru Mihai
Ticu Codru
Boronti Dana

Profitm de apariia noului numr al Revistei de Matamatic a


Mehediniului pentru a anuna c a doua ediie a concursului se va
desfura n zilele de 20-21 ianuarie 2006 la Colegiul Naional Traian.
Listele cu elevii nscrii pe coli vor fi trimise la CNT pn pe data de
13.01.2006. Taxa de participare este de 15 RON de elev, banii vor fi achitai
odat cu nscrierea; de asemenea se va desfura sesiunea de comunicri
tiinifice i referate a cadrelor didactice, taxa de nscriere este de 20 RON
pentru fiecare autor. Colegii care doresc nscrierea la sesiune vor aduce
referatul pn pe 15.01.2006; tehnoredactarea este optional i se va
prezenta mpreun cu discheta n Word XP.

- 75 -

PREMIANTI
Olimpiada Judeteana de Matematica
martie 2005

CLASA a V-a
GROSU VLAD
CIOCOTEALA ALEX.
NISTOR MIHAELA
GHINEA ANDREI
Trancota Alexandru
Grecu Anda
Buta Simona
Curelea Raul
Nicolicioiu Andrei
Sarbu Adrian
Burdescu Ana-Maria
Cicic Carla
Costea Maria
Craciun Marieta
Lupitu Gabriela
Panescu Dragos
Papuc Roxana
Rachitan Laura
Sanislav Alexandru
Zugravu Rozalia
Bobalca Andrei
Bostina Petruta
Carciu Bogdan
Clein Andrei
Pisleaga Anemona

CNT
Titeica
Strehaia
Titeica
Titeica
Titeica
Odobleja
Titeica
CNT
Odobleja
CNT
CNT
Titeica
CNT
Titeica
CNT
Titeica
Odobleja
Titeica
CNT
CNT

CNT
CNT
GEN.14
GEN.14
M
M
M
M
M
M
M
M
M
M
M
M
M
M
M
M
M
M
M
M
M

Premiul. I
Premiul. II
Premiul. II
Premiul. III

Ciniceanu Gh.
Ciniceanu Gh.
Ionic C-tin
Ionic C-tin

Sandulescu Adela
Visan Luana
Botea Anca
Codau Silviu
Dragotescu Tudor
Limbosu Maria
Pacioaga Florentin
Pilsu Irina
Rolea Andreea
Ungureanu Bianca
Nau Florin Marius
Andreescu Madali
Arbanas Gigel
Craioveanu Maria
Diaconescu Lucia
Ionascu Laura
Nanuti Gheorghe
Oprita geanina
Popescu Violeta
Radulescu silvia
Trusca Bogdan

CNT
CNT
Gen.2
Gen.14
Gen.14
CNT
P. Sergescu
Odobleja
Decebal
Orsova
Jidostita
CNT
Titeica
Jidostita
Gen.2
Gen.14
Titeica
CNT
Vj.Mare
Gen.6
D-l Tudor

M
M
M
M
M
M
M
M
M
M
M
M
M
M
M
M
M
M
M
M
M

CLASA a VI-a
TESILA BIANCA
RADU CORNEL
POPA BOGDAN
Tuculanu Andreea
Agape Mihai
Budu Anca
Caplea Luminita
Darpes Cristian
Doroiman Costinela
Gugulici Alexandra
Botoaca Madalina
Meleanca Inana
Popescu Eduard
Botosanu Cristian
Pacala Razvan
Cheresdi Ioan
Conea Sorina
Milici Simona
Papa Florin
Soare Razvan
Tuce Ovidiu

- 76 -

CNT
CNT
CNT
CNT
CNT
Odobleja
Gen.2
Gen.2
Gen.14
CNT
Titeica
Titeica
Titeica
Gen.6
Gen.5
CNT
Orsova
Titeica

TITEICA
TITEICA
GEN.14
M
M
M
M
M
M
M
M
M
M
M
M
M
M
M
M
M
M

Premiul.I
Premiul.II
Premiul.III

Zman Irina
Zman Irina
Palac Vladimira

Zorocliu Andra
Andreca Mihai
Ceanca Valentina
Crumpei Lucretia
Tita Andreea
Degeratu Silvius
Minescu Madalina
Poenaru Alexandra
Savulescu Roxana
Burileanu Daniel
Cepeti Cristian
Cotorogea Daniel
Mazi Adelina
Aldescu Liviu
Copaceanu Adela
Dadalau Iulia
Gosa Viorica
Dulan Ana

CNT
CNT
Gen.6
Gen.13
CNT
Vj.Mare
CNT
Gen.14
Gen.7
Titeica
Decebal
Gen.2
Titeica
Gen.2
Gen.2
Titeica
Gen.2
Gen.2

M
M
M
M
M
M
M
M
M
M
M
M
M
M
M
M
M
M

PREMIANTI

SSM
H

CLASA a VII-a
CALINOVICI PAUL
CROITORU RAZVAN
CARAPENCEA C-Tin
NICOARA CALIN
STRETCU OTILIA
Iorga Cristian
Petrescu Bianca
Damian Beatrice
Braila Cristiana
Ciotarla Danut
Crivac Cristina
Pit Andrei
Baltarete Ileana
Banciu Alexandra
Bragaru Evelina
Cornea Marian
Duta Adrian
Marin Andreca
Nef Nicoleta Iulia
Pana Dragos
Seitan Mihaela

Gen.2
Odobleja
Titeica
CNT
CNT
Titeica
CNT
CNT
CNT
Titeica
Gen.15
CNT
Titeica
Gen.2
Titeica
CNT

CNT
CNT
CNT
CNT
CNT
M
M
M
M
M
M
M
M
M
M
M
M
M
M
M
M

Premiul.I
Premiul.I
Premiul.I
Premiul.II
Premiul.III

Paponiu Dana
Paponiu Dana
Paponiu Dana
Paponiu Dana
Stretcu Daniel

Capraru Alexandru
Darpes Ionut
Raduica Valentin
Torsin Ligia
Bota Catalina
Ecobici Gabriela
Radut Marian Cos
Cucerzan Melania
Gae Georgiana
Marinescu Mihai
Nuta Lucian
Pupaza Elena
Sosu Cristian
Iordache Dana
Pirvulescu Dan

CNT
CNT
Gen.6
Orsova
Gen.7
CNT
Gen.2
Titeica
D Tudor
Gen.2
Titeica
CNT
CNT
Titeica
Vj.Mare

M
M
M
M
M
M
M
M
M
M
M
M
M
M
M

CLASA a VIII-a
RADUCU ALEXANDRU
ROGOBETE ROXANA
NICOLESCU ALEXA
ECHIM IOANA
Vasilache Madalina
Dumitrescu Ozana
Tigora Anca
Popescu Alina
Purcaru Alin
Ciouca Eugen
Oprea Radu
Savoiu Aurel
Voicu Andreea
Bratu Simona
Antonie Raul
Chircu Liviu
Robu Alexandru
Tuce Andreea
Trocan Irina
Nistor Ovidiu

Gen.14
Orsova
Gen.14
CNT
Gen.7
Gen.2
CNT
Gen.2
Odobleja
Gen.11
Gen.2
Gen.6
CNT
Titeica
CNT
Gen.2

CNT
CNT
CNT
CNT
M
M
M
M
M
M
M
M
M
M
M
M
M
M
M
M

Premiul.I
Premiul.II
Premiul.II
Premiul.III
Sarbu Cosmina
Bechir Adriana
Garjoaba Simona
Papala Doriana
Bobalca Oana
Buciu Alex
Ciorobea Mihai
Dinca Radu
Paponiu Elena
Trancota Andrei
Ursachi Sandel
Valceanu Georgeta
Lata Adriana
Lincan Dan
Munteanu Corina
Stefanoiu Anca

Prajea Manuela
Prajea Manuela
Prajea Manuela
Ionic C-tin
P.Sergescu
CNT
Gen.2
Gen.3
CNT
Vj.Mare
CNT
P.Sergescu
Gen.6
Vj.Mare
Vj.Mare
P.Sergescu
Gen.2
Decebal
Titeica
CNT

M
M
M
M
M
M
M
M
M
M
M
M
M
M
M
M

CLASA a IX-a
TIGORA ANDREI
COANDA OANA
GOGOLOIU GABRIELA
PRUNDEANU ANDRE
TUTA LEONTIN
Ciocea Andreea
Hinoveanu Catalin
Mituca Atena
Botea Silvia
Raveanu Ioana
Stoica Raluca
Suselea Robert

CNT
CNT
CNT
CNT
CNT
CNT
CNT

CNT
CNT
TITEICA
CNT
CNT
M
M
M
M
M
M
M

Premiul.I
Premiul.II
Premiul.III
Premiul.III
Premiul.III

Saftoiu Mihai
Ciniceanu Andrei
Trusculescu Daniel
Misconiu Danut
Rachieru Adrian
Slama Victor
Vasilcanu Dragos

Ciniceanu Gh.
Ciniceanu Gh.
Popescu Rodica
Ciniceanu Gh.
Ciniceanu Gh.
CNT
CNT
Titeica
Odobleja
CNT
CNT
CNT

M
M
M
M
M
M
M

- 77 -

PREMIANTI
Belbu Loredana
Gamala Andreea
Lica Stefania
Mariescu Radu
Nistor Irina

CNT
CNT
Orsova TL
CNT
Titeica

M
M
M
M
M

Adamescu Paula
Folcutescu Sebastia
Gal Raluca
Trasca Mihai

Titeica
Titeica
CNT
CNT

M
M
M
M

CLASA a X-a
RAPCEA MIHAI
BOBITI RUXANDRA
DAGADITA MONICA
Bocse Bogdan
Sirbu Ovidiu
Balaceanu Bogdan
Mihart Liliana
Rosu Stefan
Turturea Roxana
Busuioc Lucian
Plesca Stefan
Plotogea Catalin
Viasu Mihai
Zbarcea Razvan

CNT
Titeica
Titeica
CNT
CNT
CNT
CNT
Titeica
CNT
Odobleja
CNT

CNT
TITEICA
CNT
M
M
M
M
M
M
M
M
M
M
M

Premiul.I
Premiul.II
Premiul.III

Baltateanu Robert
Bica Denisa
Plesu Georgiana
Voichita Viorel
Cercel roxana
Coada Daniela
Cojocaru Claudia
Nastasie Liliana
Nicolaescu Mihaela
Vamvore Florin

Paponiu Dana
Zman Irina
Paponiu Dana
Titeica
CNT
Strehaia
Strehaia
CNT
Titeica
CNT
CNT
Titeica
Titeica

M
M
M
M
M
M
M
M
M
M

CLASA a XI-a
UNGUREANU ANDREI
BAZAVAN EDUARD
PASOV IULIA
Chilom George
Anghelescu Georgiana
Bazavan Cristina
Bolocan Anca
Busoniu Andra
Golenteanu Magdalena
Tartea Gabriela
Bacarin Dan

CNT
Titeica
CNT
CNT
Orsova TL
CNT
CNT
CNT

CNT
TITEICA
CNT
M
M
M
M
M
M
M
M

Premiul.I
Premiul.II
Premiul.III

Mirea Teodor
Sirbu Florin
Stoica George
Baltateanu Valeriu
Craciunescu Mirela
Dobjanschi Cristian
Pandioniu Mihaela
Vajaica Raluca

Prajea Manuela
Stretcu Daniel
Popescu Eleodor
CNT
CNT
CNT
CNT
CNT
CNT
Titeica
CNT

M
M
M
M
M
M
M
M

CLASA a XII-a
TANTARU MIHAI
TUDOR ALEXANDRU
DUMITRU IULIA
MAZILU MIRCEA
Caragea Alexandru
Irimia Florin
Mazalu Calin
Motoi Oana
Osain Alin
Banila Madalina
Cercel Clementin

- 78 -

CNT
CNT
CNT
CNT
CNT
D-l Tudor
Titeica

CNT
CNT
CNT
CNT
M
M
M
M
M
M
M

Premiul.I
Premiul.II
Premiul.III
Premiul.III
Mitroi Dan
Iovanovici Tveta
Moraru Andreea
Cilibia Marius
Dagadita Alin
Pacala Catalin
Ularu Loredana

Ciniceanu Gh.
Ciniceanu Gh.
Ciniceanu Gh.
Prajea Manuela
CNT
Orsova TL
Odobleja
CNT
CNT
CNT
D-l Tudor

M
M
M
M
M
M
M

PREMIANTI
Olimpiada Naional de Matematica
Elev
UNGUREANU ANDREI
RAZVAN CROITORU
BAZAVAN EDUARD
CARAPENCEA CONSTANTIN
TIGORA ANDREI
PASOV IULIA
BOBITI RUXANDRA

Premiu
I MEC
M MEC
M MEC
M MEC

Medalie
AUR
AUR
ARGINT
ARGINT
BRONZ
BRONZ
BRONZ

SSM
H

Profesor
PRAJEA MANUELA
PAPONIU DANA
STRETCU DANIEL
PAPONIU DANA
CAINICEANU GH.
POPESCU ELEODOR
ZAMAN IRINA

Calificai n Lotul naional lrgit


UNGUREANU ANDREI
BAZAVAN EDUARD

Calificai n lotul speranelor olimpice


CROITORU RAZVAN
CARAPENCEA CONSTANTIN
CONCURSUL INTERJUDETEAN ION CIOLAC
- MAI 2005 GROSU VLAD
NICOLESCU ALEXANDRA
TIGORA ANDREI
RAPCEA MIHAI
CARAPENCEA CONSTANTIN
CROITORU RAZVAN
RADUCU ALECSANDRU
PASOV IULIA

Premiul I
Premiul I
Premiul II
Premiul II
Meniune
Meniune
Meniune
Meniune

CAINICEANU GH.
PRAJEA MANUELA
CAINICEANU GH.
PAPONIU DANA
PAPONIU DANA
PAPONIU DANA
PRAJEA MANUELA
POPESCU ELEODOR

CONCURSUL INTERJUDETEAN
SCOALA CU CEAS RAMNICU-VALCEA MARTIE 2005

NICOARA CALIN
PAPA FLORIN

Premiul III
Meniune

PAPONIU DANA
PAPONIU DANA

- 79 -

PREMIANTI
CONCURSUL INTERJUDETEAN GHEORGHE TITEICA
RAMNICU VALCEA MAI 2005
GROSU VLAD

Meniune
Meniune

ECHIPAJ VII-VIII

CROITORU RAZVAN
CARAPENCEA CONSTANTIN
NICOLESCU ALEXANDRA
ECHIPAJ IX-X
TIGORA ANDREI
BOBITI RUXANDRA
RAPCEA MIHAI
ECHIPAJ XI-XII
PASOV IULIA
BAZAVAN EDUARD
TANTARU MIHAI

Meniune

Meniune

CAINICEANU GH.
PAPONIU DANA
PAPONIU DANA
PRAJEA MANUELA
CAINICEANU GH.
ZAMAN IRINA
PAPONIU DANA
POPESCU ELEODOR
STRETCU DANIEL
CAINICEANU GH.

CONCURSUL ANUAL AL REZOLVITORILOR


GAZETEI MATAMATICE

CARAPENCEA CONSTANTIN

Meniune

PAPONIU DANA

AMERICAN MATHEMATIC COMPETITION


CALIFICATI LA AIME

TIGORA ANDREI
UNGUREANU ANDREI
MATEISESCU ALEXANDRU
VATA RADU
PASOV IULIA
DUMITRU IULIA
NAIDAN ANDREEA
NICOARA SERBAN
TRAILESCU ALIN

IX
XI
XI
XII
XI
XII
XII
XI
XII

124P
132P
112,5P
111,5P
107,5P
105,5P
104,5P
104P
101P

CAINICEANU GH.
PRAJEA MANUELA
NANUTI DAN
PRAJEA MANUELA
POPESCU ELEODOR
CAINICEANU GH.
CAINICEANU GH.
POPESCU ELEODOR
CAINICEANU GH.

S-AU MAI REMARCAT OBINND


YOUNG STUDENT CERT OF ACHIEVEMENT

NICOARA CALIN
BOCSE BOGDAN
RAPCEA MIHAI

BICA DENISA
BOTEA SILVIA
CAINICEANU ANDREI
DAGADITA MONICA
GEORGESCU ALIN
HINOVEANU CATALIN
IORDACHE NORICA
MARIESCU RADU

- 80 -

VII
X
X

104P
96P
95P

PAPONIU DANA
PAPONIU DANA
PAPONIU DANA

ELEVI CU PESTE 100P LA AMC10


X
IX
IX
X
IX
IX
IX
IX

111P
104P
114,5P
107,5P
108P
109P
109P
111,5P

OLARU ALEXANDRU
PRUNDEANU ANDREEA
RAVEANU IOANA
SBARCEA RAZVAN
SUSELEA ROBERT
TOMESCU LIANA
TURTUREA ROXANA

IX
IX
IX
IX
X

108,5P
102P
104P
104P
102P
104,5P
105,5

REZOLVITORI
Colegiul National Traian
Prof. Paponiu Dana clasa a VI-a Ionescu Larisa (15), Bondoc Ana-Maria
(26), Tita Andreea (13), Popescu Eduard(13);
Prof. Cainiceanu Gheorghe clasa a V-a Boescu Anca(9), Mituca Anda(6) .
clasa a IX-a Iordache Norica (8), Pitulicu Lorena (8), Mariescu Radu (8),
Racheru Adrian (8), Abagiu Aurel(8), Nitulescu Maria (8), Mituca Atena(8),
Ganda Irina(8), Badea Silviu(8), Georgescu Alin(8), Cucu Irina (9), Gamala
Andreea (9), Modalca Denisa(8), Prundeanu Andreea(9), Saftoiu Mihai(8),
Steanta Ana-Marina(8), Cotet Alexandra(8), Bojinovici Sergiu(8), Ionica
Adina(8), Stavaru Daniel(8), Cretu Andrei(8), Nistor Alexandru(8), Paunescu
Georgiana(8), Ghiga Octavian(8), Gomoi Mihai(8), Susnea George(8), Porojan
Otilia(8), Vladu Margareta(8), Tomescu Liana(9), Buncianu Ilie(8), Coanda
Oana(9), Urse Ruxandra(8) clasa a XII-a Rugina Alexandru(10);
Prof. Antonie Rodica clasa a V-a Andreescu Madalina(10), Bologa
Ionela(10), Ciuca Cosmina(10), Costea Alexandra(10), Lupitu Gabriela(10),
Maritoiu Oana(10), Moldoveanu Cristina(10), Murdarea Madalina(10), Radu
Ramona(10), Sandulescu Adela(10), Trancota Alexandru(10).
coala Constantin Negreanu
Prof. Calafeteanu Gheorghe - clasa a V-a Calafeteanu Liviu George, Oel
Diana Andra, Hornoiu Eugen Alexandru, Cepoi Mihaela, Cojocaru Liviu
tefan, Surdulescu Simona Roxana, Olaru Eugen Remus, Cotolan Violeta
Silvia, Popescu Dinu, Borz Diana, Tril Alexandra, Blan Dumitru Mihai,
Epuran Vlad, tefan Raluca Rodica, Stan Alexandru; clasa a VI-a Curea

SSM
H

Loredana, Crgioiu Andrei, Goliciu Patricia, Nedelcea Sorin, Ptracu


Andreea, Nicolae Marius, Popescu Florentina Geanina, Roiu Ileana, Rianu
Alina, Gheran Sperana, Ceauc Alexandra, Calot Elena, Cornea Sorin,
Indrecan Robert, Halng Gabrieala; clasa a VII-a Irimia Laura, Poogeanu
Diana, Iordache Ana-Maria, Homeag Diana, Bratu Ana-Maria, Broiu Felix,
Epure Andrei; clasa a VIII-a Gal Oana Roxana, Anghioiu Rozvita, Petrescu
Andreea, Andrei Mdlina, Popescu Anca;
Liceul Teoretic Gh.ieica
Prof. Zman Irina Cheredi Ioan, Croitoru Raluca, Firulescu Mihai, Pcal
Rzvan, Radu Cornel, Toderic Dan.

- 81 -

COLABORATORI
Antonie Rodica
Bdlu Anghel
Bloi Valeria
Bojneagu Dina
Bondoc Gabriela
Bondoc Lucian
Buzatu Carmen Victoria
Buzatu Otilia
Calafeteanu Gheorghe
Ciniceanu Gheorghe
Coand Oana
Constantin Magdalena
Cucu Irina
Dan Daniel
Draga Ttucu Mariana
Draga Ttucu Porfirel
Gimoiu Iuliana
Giugiuc Constantin
Giugiuc Leonard

Lupu Adrian
Marica tefan
Moclea Adriana
Nedeianu Dan
Ptrcoiu Constantin
Pi-Rada Ionel-Vasile
Pi-Rada Marica
Popescu Eleodor
Prajea Manuela
Prejneanu Vasile Doru
Sceanu Iulia
Sceanu Victor
Sitaru Daniel
Ticui Ovidiu
Tigora Andrei
Trilescu Diana
Ungureanu Andrei Bogdan
Vrzaru Mariana
Zman Irina

Colectivul de redacie:
Gheorghe Ciniceanu
Manuela Prajea
Dan Daniel

Popescu Eleodor
Marcel Popescu
Dan Nedeianu

Decembrie 2005

- 82 -

Das könnte Ihnen auch gefallen